You are on page 1of 385

Fundamentals

of
Sleep Technology Workbook
Rita Brooks, MEd, R EEG/EP T, RPSGT, RST, CNIM
Director of Diagnostic Services and Clinical Informatics
Capital Health System
Trenton, New Jersey Cynthia Mattice, MS, RPSGT, RST
Director of Sleep Operations
The Sleep Clinic
Oklahoma City, Oklahoma Teofilo Lee-Chiong, MD
Professor of Medicine
National Jewish Health
Denver, Colorado
Professor of Medicine
University of Colorado Denver
Aurora, Colorado
Chief Medical Liaison
Philips Respironics
Murrysville, Pennsylvania
Associate Editor

Richard S. Rosenberg, PhD


Department of Psychology
California State University

Long Beach, California


Acquisitions Editor: Jamie Elfrank Product Development Editor: Andrea Vosburgh Production Project
Manager: David Saltzberg Design Coordinator: Holly Reid McLaughlin Manufacturing Coordinator: Beth
Welsh Marketing Manager: Stephanie Manzo Prepress Vendor: SPi Global

Copyright © 2015 American Association of Sleep Technologists
All rights reserved. This book is protected by copyright. No part of this book may be reproduced or
transmitted in any form or by any means, including as photocopies or scanned-in or other electronic copies,
or utilized by any information storage and retrieval system without written permission from the copyright
owner, except for brief quotations embodied in critical articles and reviews. Materials appearing in this
book prepared by individuals as part of their official duties as U.S. government employees are not covered
by the above-mentioned copyright. To request permission, please contact Wolters Kluwer Health at Two
Commerce Square, 2001 Market Street, Philadelphia, PA 19103, via email at permissions@lww.com, or via
our website at lww.com (products and services).

9 8 7 6 5 4 3 2 1

Printed in China

This work is provided “as is,” and the publisher disclaims any and all warranties, express or implied,
including any warranties as to accuracy, comprehensiveness, or currency of the content of this work.

This work is no substitute for individual patient assessment based upon healthcare professionals'
examination of each patient and consideration of, among other things, age, weight, gender, current or prior
medical conditions, medication history, laboratory data and other factors unique to the patient. The
publisher does not provide medical advice or guidance and this work is merely a reference tool. Healthcare
professionals, and not the publisher, are solely responsible for the use of this work including all medical
judgments and for any resulting diagnosis and treatments.

Given continuous, rapid advances in medical science and health information, independent professional
verification of medical diagnoses, indications, appropriate pharmaceutical selections and dosages, and
treatment options should be made and healthcare professionals should consult a variety of sources. When
prescribing medication, healthcare professionals are advised to consult the product information sheet (the
manufacturer's package insert) accompanying each drug to verify, among other things, conditions of use,
warnings and side effects and identify any changes in dosage schedule or contradictions, particularly if the
medication to be administered is new, infrequently used or has a narrow therapeutic range. To the maximum
extent permitted under applicable law, no responsibility is assumed by the publisher for any injury and/or
damage to persons or property, as a matter of products liability, negligence law or otherwise, or from any
reference to or use by any person of this work.

LWW.com
CONTRIBUTORS

Debbie Akers, RRT, RPSGT, RST


Operations Coordinator
Eastern Virginia Medical School
Norfolk, Virginia

Jon W. Atkinson, BS, RPSGT


Consultant
Ohio Sleep Consulting and Recording Services, LLC
Lancaster, Ohio
Sleep Health Management Resources, Inc.
Stillman Valley, Illinois
Vice President
SleeProz, Inc.
Stillman Valley, Illinois

Joyce Black, BS, RPSGT, CRT


Sr. Clinical Sales Trainer
Philips Respironics
Murrysville, Pennsylvania

Lisa M. Bond, RST, RPSGT


Scoring Technologist
Advanced Sleep Management
Bryan, Texas

Rita Brooks, MEd, R EEG/EP T, RPSGT, RST, CNIM


Director of Diagnostic Services and Clinical Informatics
Capital Health System
Trenton, New Jersey

Christopher A. Cook, MBA, RPSGT, RST


Technical Director
South Florida Sleep Centers
United Sleep Diagnostics, Inc.
American Institute for Sleep Performance, Inc.
Fort Lauderdale, Florida

Laree Fordyce, RPSGT, RST, CCRP


Clinical Sleep Educator/Technical Director
Sound Sleep Solutions
Calgary, Alberta
Canada

Charlotte Fromer, RPSGT, RST


Clinical Coordinator
American Sleep Medicine
Skokie, Illinois

Joanne Hebding, RPSGT, RST


Manager
Tallahassee Sleep Disorders Center
Tallahassee, Florida

Teofilo Lee-Chiong, MD
Professor of Medicine
National Jewish Health
Denver, Colorado
Professor of Medicine
University of Colorado Denver
Aurora, Colorado
Chief Medical Liaison
Philips Respironics
Murrysville, Pennsylvania

Steven H. Lenik, BS, RCP, RPFT, RPSGT, RST


Technical Manager
Desert Pulmonary & Sleep Consultants, PLC
Gilbert, Arizona

Laura A. Linley, RST/RPSGT


VP of Clinical Operations
Advanced Sleep Management, LLC
Richardson, Texas

Cynthia Mattice, MS, RPSGT, RST


Director of Sleep Operations
The Sleep Clinic
Oklahoma City, Oklahoma

Shalanda L. Mitchell, RPSGT, RST


Clinical Services Manager of Hampton Roads Sleep/Therapy
SleepMed, Inc.
Hampton Roads, Virginia

Richard S. Rosenberg, PhD


Department of Psychology
California State University
Long Beach, California

Connstance Shivers-Smith, BA, RPSGT, RST


Polysomnographic Specialist
Sleep Disorders Center
Saint Luke's Hospital of Kansas City
Kansas City, Missouri

Karen I. Smith, BA, RST, RPSGT


Consultant
Capitola, California

Tim A. Statza, BA, CRT, RPSGT, RST


Sleep Center Supervisor
Children's Hospital of Wisconsin
Milwaukee, Wisconsin

Roxanne Taylor, BA, RST, RPSGT


Supervisor
Sleep Center at Rocky Mountain Heart & Lung
A Department of Kalispell Regional Medical Center
Kalispell, Montana

Melinda Trimble, RPSGT, RST, LRCP


Clinical Specialist
Philips Respironics
Springdale, Arkansas

Christina L. Troxell, RPSGT, RST, MLT (ASCP) Midwest Regional Sales


Manager
SOMNOmedics America
Washington, Illinois

Harry R. Whitmore, RPSGT, RST


Chief Sleep Research Technologist
Sleep, Metabolism and Health Center
University of Chicago
Chicago, Illinois
PREFACE

This workbook, intended to complement the second edition of Fundamentals of


Sleep Technology, is specifically designed to meet the educational needs of
teachers, students, and practitioners of sleep technology. The workbook focuses
on both essential theoretical information and practical skills that everyone
involved with the profession of sleep technology is required to know. We hope
that this workbook will serve as a useful learning tool for working sleep
technologists as well as for those studying for registry examinations.

The workbook is written by and for sleep technologists and includes numerous
exercises and discussion questions to expand the knowledge of the learner.
Content focuses on testing processes and data analysis, basic anatomy and
physiology, normal sleep, and sleep disorders in adult and pediatric patients.
Many discussion questions include references to relevant published research and
other scientific papers.

Ongoing changes in definitions, sleep scoring criteria, practice guidelines,


regulatory policies, and emerging technology continue to significantly transform
the neuroscience of sleep as well as the clinical practice of sleep medicine and
technology. All efforts have been made to update material that has changed since
the textbook was published. Significant changes to material in this workbook
will be posted in the companion eBook version, where videos, presentations, and
modular learning tools will also be made available for workbook users.

The editors extend their appreciation and thanks to all the authors of this
workbook with special thanks to Dr. Richard Rosenberg, whose expertise and
experience proved invaluable to this project. We also thank Katelyn Fullerton,
AAST Coordinator; John Noel, AAST Specialty Society Manager; Andrea
Vosburgh, Product Development Editor at Wolters Kluwer Health; and the Board
of Directors of the American Association of Sleep Technologists for their
support throughout this process.

Finally, this workbook would not have been possible without the support of the
following individuals—it is to them that we dedicate this workbook:

Ed Mattice and Keely Mattice


Richard Brooks
Dolores Grace Zamudio and Zoe Lee-Chiong
Contents

SECTION I
Anatomy and Physiology
1 Sleep Across the Life Cycle
ROXANNE TAYLOR
2 Anatomy and Physiology of the Biopotentials of Sleep
SHALANDA L. MITCHELL
3 Respiratory Anatomy and Physiology
LAURA A. LINLEY
4 Oxygen and Gas Exchange in the Body
STEVEN H. LENIK
5 Cardiac Anatomy and Physiology
JON W. ATKINSON
6 General Human Physiology for the Sleep Technologist
ROXANNE TAYLOR

SECTION II
Sleep Disorders and Disorders that Affect Sleep
7 Circadian Rhythms and Circadian Rhythm Disorders
CHARLOTTE FROMER
8 Narcolepsy
CYNTHIA MATTICE
9 Insomnia
CONNSTANCE SHIVERS-SMITH
10 Parasomnias
MELINDA TRIMBLE and CONSTANCE SHIVERS-SMITH
11 Movement Disorders
MELINDA TRIMBLE
12 Periodic Limb Movement Disorder and Restless Legs Syndrome
CHARLOTTE FROMER
13 Obstructive Sleep Apnea
JOYCE BLACK
14 Central Sleep Apnea
JOANNE HEBDING
15 Seizures and Sleep
MELINDA TRIMBLE
16 Sleep and Medical Disorders
CYNTHIA MATTICE
17 Psychiatric Disorders That Affect Sleep
KAREN I. SMITH
SECTION III
Patient Care and Assessment
18 Patient and Employee Safety
DEBBIE AKERS
19 Medications and Their Effects on Sleep and Sleep Disorders
DEBBIE AKERS

SECTION IV
Polysomnography
20 Digital Polysomnography
CHRISTINA L. TROXELL
21 Recording the Biopotentials of Sleep
CHRISTINA L. TROXELL
22 Patient Interviewing and Assessment
JOANNE HEBDING
23 Patient Preparation
SHALANDA L. MITCHELL
24 Polysomnographic Recording Procedures
KAREN I. SMITH
25 Cardiac Arrhythmias
JON W. ATKINSON
26 Adult Sleep Scoring
HARRY R. WHITMORE
27 Report Generation
JON W. ATKINSON

SECTION V
Interventions and Therapeutics
28 Titration of Continuous Positive Airway Pressure and Application and
Adjustment of Positive Airway Pressure Devices
JOYCE BLACK
29 Developing and Maintaining Therapeutic Compliance
JOYCE BLACK
30 Oxygen Administration in the Sleep Center
STEVEN H. LENIK
31 Advanced PAP Therapies
LISA M. BOND
32 Dental Sleep Medicine
LAREE FORDYCE
33 Treatment of Insomnia
ROXANNE TAYLOR

SECTION VI
Ancillary Procedures
34 Multiple Sleep Latency Test and Maintenance of Wakefulness Test
CHARLOTTE FROMER
35 Actigraphy
KAREN I. SMITH
36 Portable Monitoring
LAREE FORDYCE
SECTION VII
Pediatrics
37 Pediatric Polysomnography
LAREE FORDYCE
38 Pediatric Scoring
TIM A. STATZA
39 Sleep-Related Breathing Disorders in Children
CHRISTOPHER A. COOK
40 Nonrespiratory Pediatric Sleep Disorders
CHRISTOPHER A. COOK
41 Interventions in the Pediatric Sleep Laboratory
LAREE FORDYCE

APPENDICES
I Algorithms
RICHARD S. ROSENBERG
II Glossary
RITA BROOKS
Index
SECTION I
Anatomy and Physiology
CHAPTER
1
Sleep Across the Life Cycle
ROXANNE TAYLOR

NOTE: This chapter corresponds to Chapter 3 in Fundamentals of Sleep


Technology, 2nd edition.

1. According to the two-process model, sleep-waking and alertness are


influenced by a circadian factor (Process C) and a Process S, which is also
known as the:
A. Homeostatic sleep drive
B. Sleep inertia effect
C. Ultradian process
D. Fatigue factor

2. An EEG rhythm with a frequency of 8 to 13 Hz, a sinusoidal morphology,


and maximum amplitude in the occipital region is:
A. Theta activity
B. Slow waves of youth
C. Hypnagogic hypersynchrony
D. Alpha rhythm

3. Recordings of the electroencephalogram, chin electromyogram, and


______________ are required in order to differentiate sleep from wake and
to identify the different stages of sleep.
A. Galvanic skin response
B. Eye movements
C. Blood pressure
D. Electrocardiography

4. An EEG pattern seen at sleep onset in infants and children but rarely seen
after age 12 is:
A. Frontal slow-wave activity
B. Three per second spike and wave activity
C. Sleep spindles
D. Hypnagogic hypersynchrony

5. The best predictor of EEG patterns in neonates is:


A. Gestational age
B. Birth order
C. Chronologic age
D. Conceptional age

6. At what age do all three of the key features of EEG activity during NREM
sleep (spindles, slow waves, and K complexes) first present in infants?
A. 1 month
B. 3 months
C. 6 months
D. 1 year

7. One difference in sleep stage scoring for adults using the AASM Scoring
Manual and stage scoring for infants using the Anders and Parmelee Manual
is that the latter scores:
A. Rollover minutes
B. Indeterminate sleep
C. High-voltage slow active sleep
D. Unrecognized sleep

8. A mother comes to the sleep center with her 5-year-old daughter. She is
concerned that her daughter, who will be starting kindergarten in the fall, is
still napping. The best course of action is to:
A. Begin a small dose of a stimulant such as modafinil
B. Evaluate the child for pediatric narcolepsy
C. Reassure the parent that this is normal for age
D. Recommend cognitive behavioral therapy for the child

9. The most prominent change in circadian sleep pattern in the transition from
childhood to adolescence is:
A. Early morning awakening
B. Increased sleep fragmentation
C. Postprandial lethargy
D. Delay of the sleep phase
10. The typical NREM–REM sleep cycle in young adults lasts:
A. 90 to 110 minutes
B. 60 to 70 minutes
C. 20 to 40 minutes
D. 90 to 110 seconds

11. A change in sleep stages with aging that is most evident in elderly men is:
A. Decreased stage N3 sleep
B. Elimination or near elimination of REM sleep
C. Decreased amounts of stage N1 sleep
D. Decreased amounts of stage N2 sleep

12. Altered circadian rhythms in older adults are due to changes in the biologic
clock, decreased melatonin secretion with aging, and:
A. Weakened environmental factors such as limited daylight exposure
B. Excessive exposure to bright light in the morning
C. Use of laptops and video games with bright screens at night
D. Incentives for early afternoon meals at several restaurant chains

DISCUSSION QUESTIONS

13. A recent study demonstrated a statistically significant increase in teenage


driver crash rates in a district with early school start times as compared to a
neighboring school district with later start times. Discuss how age-related
changes in sleep patterns might contribute to this problem.

14. Although there are differences in slow-wave activity between young and
elderly sleepers, both groups respond to frequent naps with similar changes
in slow-wave activity. Since slow-wave activity is thought to reflect
homeostatic sleep processes, this suggests that the homeostatic sleep process
is intact in the elderly. Discuss prominent age-related changes in sleep and
their relationship to the two-process theory.

ANSWERS

1. A, Homeostatic sleep drive.


See Fundamentals of Sleep Technology, 2nd edition, Chapter 3, page 16.

2. D, Alpha rhythm.
See Fundamentals of Sleep Technology, 2nd edition, Chapter 3, page 16.

3. B, Eye movements.
See Fundamentals of Sleep Technology, 2nd edition, Chapter 3, page 16.

4. D, Hypnagogic hypersynchrony.
See Fundamentals of Sleep Technology, 2nd edition, Chapter 3, page 17.

5. D, Conceptional age.
See Fundamentals of Sleep Technology, 2nd edition, Chapter 3, page 17.

6. C, 6 months.
See Fundamentals of Sleep Technology, 2nd edition, Chapter 3, page 17.

7. B, Indeterminate sleep.
See Fundamentals of Sleep Technology, 2nd edition, Chapter 3, page 18.

8. C, Reassure the patient that this is normal for age.


See Fundamentals of Sleep Technology, 2nd edition, Chapter 3, page 19.

9. D, Delay of the sleep phase.


See Fundamentals of Sleep Technology, 2nd edition, Chapter 3, page 20.

10. A, 90 to 110 minutes.


See Fundamentals of Sleep Technology, 2nd edition, Chapter 3, page 21.

11. A, Decreased stage N3 sleep.


See Fundamentals of Sleep Technology, 2nd edition, Chapter 3, page 23.

12. A, Weakened environmental factors such as limited daylight exposure.


See Fundamentals of Sleep Technology, 2nd edition, Chapter 3, page 23.

13. “An increased pressure to sleep may explain the increased rate of teen crashes
when school start times are more than an hour earlier. In addition, early start
times such as are seen in Virginia Beach conflict with neurophysiology. For a
teen arising at 06:00 to achieve at least 9 hours of sleep, he or she would have
to go to bed by 21:00. Beyond the impracticality of getting a high school
student in bed by 21:00, teen delayed circadian rhythms work against such an
early bedtime. These teens may suffer from circadian delays in addition to
sleep deprivation, which may place them at a heightened risk for crashes.
Sleep deprivation may also be related to increased risk-taking proclivity,
which might relate to increased crash rates.” (Vorona RD, Szklo-Coxe M, Wu
A, et al. Dissimilar teen crash rates in two neighboring southeastern Virginia
cities with different high school start times. J Clin Sleep Med
2011;7(2):145–151.)

14. “There was no age difference between relative TST and SE during the naps,
and both age groups responded to low sleep pressure with lower SE, longer
sleep latencies, and more wakefulness after lights out during the recovery
night. Concerning the EEG spectra, which allow quantification of the sleep
homeostatic process, a very similar EEG delta power decline during the first
NREM cycle was found in both age groups during the recovery night. If we
assume that the SWA level at the beginning of the sleep episode is one of the
most reliable physiologic markers for accumulated homeostatic sleep
pressure, we do not have strong evidence for an age difference in homeostatic
delta response to low sleep pressure, similar to the findings of Campbell and
Feinberg. In that study, neither age-related differences in the mean EEG delta
response (0.3 to 3.0 Hz) during the postnap night nor a change in the period-
amplitude incidence of EEG delta power were found.” (Munch M, Knoblauch
V, Blatter K, et al. Is homeostatic sleep regulation under low sleep pressure
modified by age? Sleep 2007;30(6):781–792.)
CHAPTER
2
Anatomy and Physiology of the
Biopotentials of Sleep
SHALANDA L. MITCHELL

NOTE: This chapter corresponds to Chapter 5 in Fundamentals of Sleep


Technology, 2nd edition.

1. The biopotentials that are recorded during electroencephalography (EEG) are


the result of:
A. Enzymatic degradation
B. Thermal consequences of metabolic activity
C. Ion movement across membranes
D. DNA to RNA transcription

2. Specialized muscle fibers in the heart carry signals from the pacemaker
located in the ______________ to the muscles of the atrium and ventricles.
A. Suprachiasmatic nucleus (SCN)
B. Sinoatrial (SA) node
C. Cardioversion locus
D. Synaptic cleft

3. Proper cleaning and preparation of the skin prior to application of electrodes


is essential to reduce:
A. Wheatstone bridging
B. Soiling of the electrodes
C. Risk of electrocution
D. Impedance

4. Electroencephalographic (EEG) waveforms are categorized based on wave


shape, location on the scalp, amplitude, and:
A. Impedance
B. Size of the voltage recorded
C. Sleep stage
D. Frequency

5. Which electroencephalographic (EEG) waveform is seen during relaxed


wakefulness with the eyes closed?
A. Sawtooth waves
B. Theta waves
C. Alpha waves
D. Occipital slow waves of youth

6. If there is damage to the sinoatrial (SA) node, the heart rate can be regulated
by the atrioventricular (AV) node or the bundle of His. When this happens,
the heart rate is typically:
A. Slower
B. Irregular
C. Faster
D. Irregularly irregular

7. The P wave represents:


A. Activity of the parietal cortex
B. Depolarization of the cardiac atria
C. Activation of thalamocortical loops
D. Pulmonary artery distention

8. Gas exchange in the lung takes place in the:


A. Trachea
B. Bronchus
C. Alveolar sacs
D. Respiratory bronchioles

9. Peripheral sensors that monitor oxygen, carbon dioxide, and acidity levels in
the blood include the:
A. Aortic and carotid bodies
B. Pulmonary mechanoreceptors
C. Baroreceptors
D. Medulla

10. Brain nuclei involved in the control of respiration are located primarily in
the:
A. Hypothalamus
B. Thalamus
C. Limbic system
D. Pons and medulla

MATCH THE DESCRIPTION WITH THE TERM

11. Electrical representation of the depolarization of the A. Retinohypothalamic


cardiac ventricles tract
12. A change in the potential of the cell membrane that B. QRS complex
makes it more positive
13. Fibers that transmit signals from the retina to the C. Delta waves
suprachiasmatic nuclei
14. Electroencephalographic (EEG) activity with a D. Depolarization
frequency of 0.5 to 2 Hz and amplitude of at least
75 mV

DISCUSSION QUESTIONS

15. Describe the generation of the electroencephalographic (EEG) slow-wave


activity during sleep.

16. What causes heart rate variability during sleep?


ANSWERS

1. C, Ion movement across membranes.


See Fundamentals of Sleep Technology, 2nd edition, Chapter 5, page 37.

2. B, Sinoatrial (SA) node.


See Fundamentals of Sleep Technology, 2nd edition, Chapter 5, page 38.

3. D, Impedance.
See Fundamentals of Sleep Technology, 2nd edition, Chapter 5, page 39.

4. D, Frequency.
See Fundamentals of Sleep Technology, 2nd edition, Chapter 5, page 41.

5. C, Alpha waves.
See Fundamentals of Sleep Technology, 2nd edition, Chapter 5, page 41.

6. A, Slower.
See Fundamentals of Sleep Technology, 2nd edition, Chapter 5, page 43.

7. B, Depolarization of the cardiac atria.


See Fundamentals of Sleep Technology, 2nd edition, Chapter 5, page 44–45.

8. C, Alveolar sacs.
See Fundamentals of Sleep Technology, 2nd edition, Chapter 5, page 45.

9. A, Aortic and carotid bodies.


See Fundamentals of Sleep Technology, 2nd edition, Chapter 5, page 46.

10. D, Pons and medulla.


See Fundamentals of Sleep Technology, 2nd edition, Chapter 5, page 46.

11. B, QRS complex.

12. D, Depolarization.

13. A, Retinohypothalamic tract.

14. C, Delta waves

15. “All slow waves in the sleep EEG are manifestations of the same fundamental
cellular phenomenon: the slow oscillation of cortical neurons between a
depolarized upstate and a hyperpolarized downstate. Slow waves originate in
specific cortical regions and travel over the brain, each with a distinct site of
origin and pattern of propagation.” (Tononi G. Slow wave homeostasis and
synaptic plasticity. J Clin Sleep Med. 2009;5:S16–S19.)

16. “Cardiac autonomic tone, as measured by heart rate variability (HRV), has
been identified as a physiologic mechanism through which sleep disturbances
and disorders may potentially influence morbidity and mortality. Heart rate
variability varies as a function of sleep such that power in the high frequency
band (HF-HRV), interpreted as a measure of parasympathetic tone, correlates
with the depth of NREM sleep and is highest in stage N3. Conversely, REM
sleep and lighter stages of NREM sleep are characterized by decreased power
in the HF-HRV band. Heart rate variability may also vary as a function of
sleep disorders, including insomnia and sleep-disordered breathing (SDB),
and experimental sleep restriction.” (Israel B, Buysse DJ, Krafty RT, et al.
Short-term stability of sleep and heart rate variability in good sleepers and
patients with insomnia: for some measures, one night is enough. Sleep.
2012;35(9):1285–1291.) ADDITIONAL READING FOR DISCUSSION
QUESTIONS
Tononi G. Slow wave homeostasis and synaptic plasticity. J Clin Sleep Med.
2009;5:S16–S19.
Stein PK, Pu Y. Heart rate variability, sleep and sleep disorders. Sleep Med Rev.
2012;16(1):47–66.
Israel B, Buysse DJ, Krafty RT, et al. Short-term stability of sleep and heart rate
variability in good sleepers and patients with insomnia: For some measures, one
night is enough. Sleep. 2012;35(9):1285–1291.
CHAPTER
3
Respiratory Anatomy and Physiology
LAURA A. LINLEY

NOTE: This chapter corresponds to Chapter 6 in Fundamentals of Sleep


Technology, 2nd edition.

1. Sensory signals from the medulla, carotid bodies, and aortic arch provide
feedback information to the respiratory system in the brainstem about levels
of:
A. Lung stretch and inflation
B. Oxygen, carbon dioxide, and pH
C. Fatigue of the respiratory muscles
D. Level of stress requiring increased energy production

2. The functions of the upper airway include filtering, warming, and


______________ of incoming air.
A. Humidification
B. Desaturation
C. Resaturation
D. Desalinization

3. Upper airway collapse, such as that seen in obstructive sleep apnea, typically
takes place at the level of the:
A. Alveolar sac
B. Trachea and bronchus
C. Diaphragm
D. Larynx and pharynx

4. Gas exchange between the lungs and the vascular system occurs at the:
A. Alveoli and capillaries
B. Bronchioles and veins
C. Lobes and aorta
D. Trachea and carotids
5. During sleep, neural output to all skeletal muscles is reduced with the
exception of the:
A. Pectoralis
B. Iliopsoas
C. Brachioradialis
D. Diaphragm

6. The motor nerve controlling the diaphragm is the:


A. Intercostal
B. Diaphragmatic
C. Radial
D. Phrenic

7. Tidal volume of the lung is:


A. Total lung capacity
B. Volume of air moved in a single unforced breath
C. Amount of air moved in 1 minute of breathing
D. Physiologic dead space

8. The relationship between the hemoglobin saturation (the percentage of the


capacity of hemoglobin to carry oxygen) and the pressure of oxygen in the
blood is graphed as the:
A. Oxygen/carbon dioxide correlation
B. Inverted u-shaped curve
C. Oxyhemoglobin dissociation curve
D. Apnea/hypopnea index

9. The work of breathing increases with greater:


A. Oxygen saturation
B. Blood flow
C. Airway resistance
D. Upper airway diameter

10. The respiratory system normally responds most robustly to:


A. Hypoxia
B. Hypercarbia
C. Hypocarbia
D. Anemia
11. The basic respiratory rhythm is generated in the:
A. Hypothalamus
B. Pons
C. Medulla
D. Reticular activating system

12. The reduction in respiratory rate at sleep onset is due to decreased sensitivity
to:
A. Basic respiratory rhythm generators
B. Heart rate
C. Carbon dioxide levels
D. Upper airway resistance

13. Respiratory rate is normally slowest during:


A. NREM sleep
B. REM sleep
C. Quiet wakefulness
D. Indeterminate sleep

14. The sleep stage with the lowest variability in respiratory rate is:
A. N1 sleep
B. N2 sleep
C. N3 sleep
D. REM sleep

15. A sigh or hyperventilation can cause a brief fall in PCO2 level resulting in:
A. Pneumothorax
B. Central apnea
C. Dead space
D. Complex sleep apnea

DISCUSSION QUESTIONS

16. Increased levels of carbon dioxide in the blood stimulate breathing. Central
sleep apnea is a result of decreased drive to breathe. Could increasing CO2
improve central sleep apnea?

17. Abnormalities of the motor control of pharyngeal muscles may be involved


in the pathogenesis of obstructive sleep apnea. How do inputs to the upper
airway muscles change with sleep stage? How does this contribute to the
increased risk of upper airway collapse during sleep?

ANSWERS

1. B, Oxygen, carbon dioxide, and pH.


See Fundamentals of Sleep Technology, 2nd edition, Chapter 6, page 47.

2. A, Humidification.
See Fundamentals of Sleep Technology, 2nd edition, Chapter 6, page 48.

3. D, Larynx and pharynx.


See Fundamentals of Sleep Technology, 2nd edition, Chapter 6, page 49.

4. A, Alveoli and capillaries.


See Fundamentals of Sleep Technology, 2nd edition, Chapter 6, page 49.

5. D, Diaphragm.
See Fundamentals of Sleep Technology, 2nd edition, Chapter 6, page 51.

6. D, Phrenic.
See Fundamentals of Sleep Technology, 2nd edition, Chapter 6, page 51.

7. B, Volume of air moved in an unforced single breath.


See Fundamentals of Sleep Technology, 2nd edition, Chapter 6, page 53.

8. C, Oxyhemoglobin dissociation curve.


See Fundamentals of Sleep Technology, 2nd edition, Chapter 6, page 55.

9. C, Airway resistance.
See Fundamentals of Sleep Technology, 2nd edition, Chapter 6, page 57.

10. B, Hypercarbia.
See Fundamentals of Sleep Technology, 2nd edition, Chapter 6, page 58.
11. C, Medulla.
See Fundamentals of Sleep Technology, 2nd edition, Chapter 6, page 59.

12. C, Carbon dioxide levels.


See Fundamentals of Sleep Technology, 2nd edition, Chapter 6, page 60.

13. A, NREM sleep.


See Fundamentals of Sleep Technology, 2nd edition, Chapter 6, page 61.

14. C, N3 sleep.
See Fundamentals of Sleep Technology, 2nd edition, Chapter 6, page 62.

15. B, Central apnea.


See Fundamentals of Sleep Technology, 2nd edition, Chapter 6, page 62.

16. “The role of abnormal CO2 homeostasis in the development of sleep-


ventilatory instability is well established. Central apneas during sleep in
patients with idiopathic central sleep apnea can be triggered by abrupt
hyperventilation, and such patients chronically hyperventilate, maintaining
their PaCO2 close to the threshold for apnea during sleep as the result of
increased chemical respiratory drive. These patients have significantly lower
mean transcutaneous and end-tidal CO2 during sleep and lower arterial
PaCO2 levels while awake, typically 2 to 3 mm Hg difference from controls,
and have increased wake ventilatory responses to CO2. Patients with
congestive cardiac failure and Cheyne-Stokes respiration have lower PaCO2
levels than those without…. Increasing CO2 in a variety of ways has been
used to treat central sleep apnea and periodic breathing. This includes direct
inhalation of CO2 and increasing dead space.” (Thomas RJ, Daly RW, Weiss
JW. Low-concentration carbon dioxide is an effective adjunct to positive
airway pressure in the treatment of refractory mixed central and obstructive
sleep-disordered breathing. Sleep 2005;28(1):69–77.)

17. “The change in activity from wake to sleep has not been reported for all the
muscles, but those that have been studied generally show decreased activity
in non-REM sleep with further decreases in REM. The mechanical
consequences of this decreased activity have been demonstrated by Goh et al.,
who showed that the net dilating force acting on the UA during breathing (as
measured by the amplitude of the pressure swings in the isolated UA) is
decreased from wake to non-REM sleep and is frequently absent in REM. In
addition to the decreases in phasic volume changes in sleep, it was also
shown that baseline pressure in the isolated UA became more positive from
wake to sleep, indicating a net decrease in volume probably mediated by
decreased tonic UA muscle activity.” (Horner RL. Motor control of the
pharyngeal musculature and implications for the pathogenesis of obstructive
sleep apnea. Sleep 1996;19(10):827–853.)
CHAPTER
4
Oxygen and Gas Exchange in the Body
STEVEN H. LENIK

NOTE: This chapter corresponds to Chapter 7 in Fundamentals of Sleep


Technology, 2nd edition.
RESPIRATION/VENTILATION

1. The difference between respiration and ventilation is which of the following?


A. Respiration occurs during inhalation, whereas ventilation occurs during
exhalation
B. Respiration is the amount of air exchanged, and ventilation is the flow of
air
C. Ventilation is mechanical gas movement; respiration is physiological gas
exchange
D. Ventilation occurs during sleep, and respiration occurs while awake

GAS TRANSFER

2. The process by which oxygen enters the bloodstream from the alveoli is
known as:
A. Diffusion
B. Transpiration
C. Ventilation
D. Osmosis

3. Which process removes carbon dioxide produced from cell metabolism?


A. Diffusion
B. Blood circulation
C. Ventilation
D. All of the above

OXYGEN AVAILABILITY

4. Which of the following phenomena is responsible for decreased oxygen


availability at higher altitude?
A. Concentration of oxygen decreases due to less molecules.
B. Reduced temperature results in lower diffusion rate.
C. Pressure drops as the mass of air above decreases.
D. None of the above

PARTIAL PRESSURE

5. At a barometric pressure of 760 torr (or mm Hg), what is the partial pressure
of oxygen in atmospheric air?
A. 21%
B. 160 torr
C. 14.7 psi
D. None of the above

OXYGEN-HEMOGLOBIN DISSOCIATION
CURVE

6. Which of the following decreases hemoglobin's ability to bind oxygen?


A. Increased CO2
B. Decreased temperature
C. Increased pH
D. Decreased hematocrit

7. Define oxygen saturation.


A. Amount of oxygen bound to hemoglobin
B. Quantity of oxygen available to the blood
C. Portion of hemoglobin's capacity to carry oxygen
D. Percent of oxygen pressure on the hemoglobin

OXYGEN CONTENT

8. Oxygen content of the blood is determined by:


A. Total amount of hemoglobin
B. Partial pressure of oxygen (pO2)
C. Oxygen saturation (SaO2)
D. All of the above

HYPOXEMIA/HYPOXIA

9. Tissue hypoxia is caused by:


A. Low amounts of hemoglobin
B. Low partial pressure of oxygen (pO2)
C. Poor circulation
D. Poisoning
E. All of the above

CORRECTING HYPOXIA

10. The best treatment for low SaO2 in the sleep lab is:
A. Supplemental oxygen via nasal cannula
B. Continuous positive airway pressure (CPAP) to open the upper airway
C. Bilevel positive airway pressure (BPAP) with timed back-up-rate
D. None of the above
E. All of the above
EXAMPLE NO. 1: JANE DOE

11. Ms. Doe is seen in the sleep center for shortness of breath on exertion and
nocturnal desaturation from an overnight oxygen saturation test. She has
been observed to snore and to “hold her breath” during sleep.
Polysomnography demonstrated no evidence of obstructive apneas, and only
a few mild hypopneas (<5 per hour of sleep) are noted. Her baseline SaO2 is
91%, and drops to an average of 88% on falling asleep and further to 78%
during REM sleep. What is the appropriate treatment?

EXAMPLE NO. 2: JOHN DOE

12. Mr. Doe presents with loud snoring, excessive daytime sleepiness and
frequent dyspneic episodes, especially upon waking. Home overnight
oximetry demonstrates an SaO2 nadir of 80%, and 72% of the night was
spent below 90% saturation. Polysomnography shows repeated obstructive
and mixed apneas and obstructive hypopneas. Baseline SaO2 is 96%, but
falls to 79% during episodes of apneas. What is the appropriate treatment?

SAMPLE PROBLEM NO. 1

13. Given the data below, calculate the CaO2 and explain the data's significance.
pO2 = 92 torr SaO2 = 96%
Hb = 14.2 g/dL
FiO2 = 30% (or 0.30)

SAMPLE PROBLEM NO. 2


14. Given the data below, calculate the CaO2 and explain the data's significance.
pO2 = 65 torr SaO2 = 99%
Hb = 9.8 g/dL
FiO2 = 21% (or 0.21 or room air)

SAMPLE PROBLEM NO. 3

15. Given the data below, calculate the CaO2 and explain the data's significance.
pO2 = 154 torr SaO2 = 74%
Hb = 16.3 g/dL
FiO2 = 28% (or 0.28)

ANSWERS

1. C, Ventilation is mechanical gas movement; respiration is physiological gas


exchange.
While ventilation refers to the movement of gas in and out of the lungs, it does not describe the process
of gas exchange. Respiration is the process of oxygen and carbon dioxide exchange between the alveoli
and blood, the consumption of oxygen by the tissues, and carbon dioxide production and removal from
the body.
See Fundamentals of Sleep Technology, 2nd edition, Chapter 7, page 64.

FURTHER READING ON VENTILATION AND RESPIRATION: Nilsson GE. Respiratory


Physiology of Vertebrates. Cambridge, UK: Cambridge University Press, 2010. ISBN 978-0-521-
70302-4.
Randall D. Eckert Animal Physiology. New York: W.H. Freeman and Co., 2002. ISBN 0-7167-3863-5.
Hogan MC. Respiration. In: McGinley M, Cleveland CJ, eds. Encyclopedia of Earth. Washington, DC:
National Council for Science and the Environment, 2011.

2. A, Diffusion.
Gas molecules naturally travel from a region of higher pressure to one of lower pressure by the process
of diffusion. Since pressure of oxygen in the alveoli is relatively higher (about 147 torr) compared to
that in blood (about 50 to 70 torr), oxygen readily diffuses from the alveoli to blood. Conversely, carbon
dioxide pressure in the blood is relatively higher (about 40 torr) compared to alveoli (almost 0 torr on
inspiration), so carbon dioxide diffuses into the alveoli from the bloodstream.
See Fundamentals of Sleep Technology, 2nd edition, Chapter 7, page 66.

3. D, All of the above.


Tissue cells consume oxygen in order to fuel metabolic processes. Carbon dioxide (CO2) is produced as
a byproduct. In order to eliminate CO2, gas is first transferred from cells to blood by diffusion. Blood
carrying CO2 is pumped to the lungs by the heart (circulation and perfusion). In the lungs, CO2 is
transferred from blood to the alveoli by diffusion, and finally exhaled (ventilation). The process is
illustrated in Figure 4-1.

Figure 4-1 Interchange between metabolism, circulation, and ventilation.

See Fundamentals of Sleep Technology, 2nd edition, Chapter 7, page 66.

FURTHER READING ON GAS EXCHANGE: Piiper J, Dejours P, Haab P, et al. Concepts and
basic quantities in gas exchange physiology. Respir Physiol. 1971;13:292–304.
Wasserman K, Whipp B, Koyal S, et al. Anaerobic threshold and respiratory gas exchange during
exercise. J Appl Physiol. 1973;35:236–243.
Krogh A, Krogh M. Rate of diffusion into lungs of man. Skand Arch Physiol. 1910;23:236–247.
Crank J. The Mathematics of Diffusion. Oxford, UK: Clarendon Press, 1956.

4. C, Pressure drops as the mass of air above decreases.


Atmospheric pressure is created by the mass of air particles pushing downward due to gravitational
attraction. Since the atmosphere is limited to about 62 miles above the earth's surface (with three
quarters of the air located within 36,000 feet), the higher one climbs, the less air is above him. Less air
pushing down results in lower pressures. Nevertheless, the concentration (portion) of oxygen remains
constant at about 21% of the total air regardless of altitude (Fig. 4-2).

Figure 4-2 Atmospheric pressure.

See Fundamentals of Sleep Technology, 2nd edition, Chapter 7, page 65–66.

FURTHER READING ON OXYGEN AVAILABILITY: Randall D. Eckert Animal Physiology. New


York: W.H. Freeman and Co., 2002. ISBN 0-7167-3863-5.
Boron WF, Boulpaep EL. Medical Physiology, 2nd ed. Philadelphia, PA: Saunders, 2008.
Costanzo LS. Physiology, 4th ed. Philadelphia, PA: Saunders Elsevier, 2010.
Ivanov KP, Derry AN, Vovenko EP, et al. Direct measurements of oxygen tension at the surface of
arterioles, capillaries and venules of the cerebral cortex. Pflugers Arch. 1982;393:118–120.
Koeppen BM, Stanton BA. Berne and Levy Physiology, 6th ed. Philadelphia, PA: Mosby Elsevier,
2008.
Piiper J, Scheid P. Diffusion limitation of O2 supply to tissue in homogeneous and heterogeneous
models. Respir Physiol. 1991;85:127–136.
Pittman RN. Microvessel blood oxygen measurement techniques. In: Baker CH, Nastuk WL, eds.
Microvascular Technology. Orlando, FL: Academic Press, 1986:367–389.
Popel AS. Theory of oxygen transport to tissue. Crit Rev Biomed Eng. 1989;17:257–321.
Riley RL. Gas exchange and transportation. In: Ruch TC, Patton HD, eds. Physiology and Biophysics,
19th ed. Philadelphia, PA: Saunders, 1965:761–787.

5. B, 160 torr.
Partial pressure of a specific gas is the pressure (force) it exerts as a percentage of the whole
atmosphere. If the total atmosphere has a pressure of 760 torr, and if oxygen constitutes 21% of the
atmosphere, then 0.21 × 760 torr = 159.6 torr (rounded to 160 torr). Hence, the oxygen pressure alone is
160 torr whereas other gases (chiefly nitrogen) are responsible for the other 600 torr of atmospheric
pressure. This partial pressure is indicated by a small “p” in front of the gas name: For example pO2
refers to the partial pressure of oxygen, and pCO2 is the partial pressure of carbon dioxide.
See Fundamentals of Sleep Technology, 2nd edition, Chapter 7, page 65–66.

FURTHER READING ON PARTIAL PRESSURE: Henrickson C, Cliff Notes Chemistry Quick


Review, 2nd Edition, Wiley Publishing, Inc., Hoboken, NJ, 2011
Dutton, F.B. “Dalton's Law of Partial Pressures.” Journal of Chemical Education Aug 1961.
Frostburg State University's “General Chemistry Online”
http://antoine.frostburg.edu/chem/senese/101/index.shtml (accessed Aug 2014) Flemming C. Medical
Biophysics, 6th ed. 2008:200.
Perry RH, Green DW, eds. Perry's Chemical Engineers' Handbook, 7th ed. McGraw-Hill, 1997. ISBN
0-07-049841-5.
Smith, F.L., and Harvey, A.H., “Avoid Common Pitfalls when using Henry's Law,” Chem. Eng. Prog.
103(9), 33-39 (Sept. 2007).
Jacquez J. Respiratory Physiology. McGraw-Hill, American Physiological Society, Bethesda, MD,
1979:156–175

6. A, Increased CO2.
Almost all oxygen carried by the blood (about 98%) is bound to hemoglobin. Each gram of hemoglobin
can carry about 1.34 mL of oxygen. Several factors determine how much oxygen the hemoglobin
actually does carry: A higher pO2 will increase binding by causing more contact between oxygen
molecules and hemoglobin molecules in a given time i.e., high pressure results in more diffusion; so
will an increased temperature (for the same reason). A higher pCO2, on the other hand, will hinder
oxygen binding by displacing the latter (imagine trying to get a good seat at a crowded concert).
Increased CO2 will also decrease pH as CO2 spontaneously combines with water to form carbonic acid:

See Fundamentals of Sleep Technology, 2nd edition, Chapter 7, page 67.

7. C, Portion of hemoglobin's capacity to carry oxygen.


Each gram of hemoglobin carries about 1.34 mL of oxygen. If a gram of hemoglobin is only carrying
1.00 mL of oxygen, then it's carrying 1.00 ÷ 1.34 or 75% of its potential capacity. This (75%) is the
oxygen saturation (SaO2). Increasing oxygen pressures (pO2) will increase oxygen saturation, and vice
versa. This relationship is not linear, as shown in Figure 4-3. As oxygen saturation nears 100%, further
increases in PO2 will cause little change in SaO2. At lower saturations, in contrast, small decrements in
oxygen pressure will result in large drops in saturation. Thus, oxygen is easily removed from
hemoglobin (and utilized by the cells) at the level of the tissue where pO2s are naturally low, and
hemoglobin is more easily saturated with oxygen when it is in contact with alveoli, where pO2s are far
above 70 torr.
Figure 4-3 Oxy-hemoglobin dissociation curve.

See Fundamentals of Sleep Technology, 2nd edition, Chapter 7, page 67.

FURTHER READING ON OXYHEMOGLOBIN DISSOCIATION CURVE: Jacquez J.


Respiratory Physiology. McGraw-Hill, 1979:156–175.
Coetzee A, Swanepoel C. The oxyhemoglobin dissociation curve before, during and after cardiac
surgery. Scand J Clin Lab Invest Suppl. 1990;203: 149–153. doi:10.3109/00365519009087504. PMID
2089610.
Schmidt-Nielsen. Animal Physiology: Adaptation and Environment. Cambridge University Press, 1997.
ISBN 0521570980.
Harvey, R. [Ed.] Lippincott's Illustrated Reviews: Biochemistry 5th ed., Lippincott Willliams &
Wilkins, Philadelphia, 2010.
Rodman T, Close HP, Cathcart R. The oxyhemoglobin dissociation curve in the common
hemoglobinopathies. Am J Med. 1959;27(4)558–566.

8. D, All of the above.


Oxygen content of the blood (CaO2) is the total amount of oxygen available to all tissues, and is
determined by several factors, namely (a) the total amount of hemoglobin (Hb) capable of carrying
oxygen (note: abnormal hemoglobins and hemoglobin bound to other molecules may not be available),
(b) oxygen saturation, and (c) partial pressure of oxygen. About 98% of oxygen is carried by
hemoglobin, and 2% is dissolved in the blood plasma (0.003 mL of oxygen per 100 mL of blood for
every torr of oxygen pressure). Thus, total oxygen content is:
In a person who has 15 g of hemoglobin and an arterial blood pO2 of 95 mm Hg (normal), based on
the oxy-hemoglobin dissociation curve above, this pO2 corresponds to an oxygen saturation of 97%.
Hence, CaO2 is (15 g/dL × 1.34 mL/g × 97/100) + (0.003 mL/torr/dL × 95 torr) or 19.78 mL/dL (try it!).
Normal CaO2 is 16 to 20 mL/dL of blood.
Multiplying this value by 5 L of blood in the body (for a typical adult male) gives a total of 989 mL
of oxygen (almost a liter) in this person's blood.
Another person with a hemoglobin of 10 g, pO2 of 85 mm Hg, and SaO2 of 96% (still looks good!)
will have a calculated CaO2 of 13.12 mL/dL [(10 g/dL × 1.34 mL/g × 96/100) + (0.003 mL/torr/dL × 85
torr)] or about 27% below normal. Despite normal blood gases, it is readily apparent that this person is
quite deprived of oxygen.
See Fundamentals of Sleep Technology, 2nd edition, Chapter 7, page 67.

FURTHER READING ON OXYGEN CONTENT: McLelland, SA, Oxygen Delivery and


Haemoglobin, Contin Educ Anaesth Crit Care Pain (2004) 4 (4): 123-126
Stoelting RK, Hillier SC: Pharmacology & Physiology in Anesthetic Practice. 4th ed, 2006 Lippincott
Williams & Wilkins, Philadelphia Morgan, Jr., M. S. Mikhail, & M. J. Murray (Eds.), Clinical
anesthesiology (4th ed., pp. 200-202). New York: Lange Medical Books/McGraw-Hill Medical
Publishing Division.
Miller, RD, ed. Anesthesia. 6th ed. Elsevier/Churchill Livingstone, 2005, Philadelphia ISBN 0443066

9. E, All of the above.


Hypoxemia refers to low oxygen levels in the blood. It can be caused by exposure to low levels of
oxygen (either reduced concentration [FiO2] or pressure [pO2]), lung disease (such as asthma or
emphysema) or asphyxiation (e.g., strangulation or obstructive sleep apnea). It manifests as low blood
pO2 and SaO2. It is important to realize that hypoxemia is only one cause of tissue hypoxia. Even if
blood gases are within acceptable levels, hypoxia can be present due to anemia or unavailable
hemoglobin, such as in carbon monoxide poisoning.
Hypoxia can also occur despite normal or high blood pO2s when circulation is poor. Blood cannot
get to the tissues in a timely manner either because cardiac output is decreased, blood is shunted from an
organ or blood, or oxygen demand is increased (think exercise, fever and cancer as examples).
An additional cause of hypoxemia is histotoxic hypoxia, which occurs when tissue mitochondria are
damaged or poisoned, and are, thus, unable to metabolize oxygen via the normal Kreb's cycle (e.g.,
cyanide poisoning). Again, blood gas findings can be completely normal. Beware when interpreting
blood gases as the sole indicator of hypoxia—hypoxemia is not the only cause.
See Fundamentals of Sleep Technology, 2nd edition, Chapter 7, page 67–68.

FURTHER READING ON HYPOXEMIA/HYPOXIA: West JB. Pulmonary Pathophysiology: The


Essentials, 8th ed. Lippincott Williams & Wilkins, 2012, Philadelphia ISBN 978-1-4511-0713-5.
Anderson KN, ed. Mosby's Medical, Nursing and Allied Health Dictionary, 6th ed. C.V. Mosby, 2002,
Reed Elsevier, London ISBN 978-0-323-01430-4.
Hess DR, MacIntyre NR, Mishoe SC, et al., eds. Respiratory Care: Principles and Practice, 2nd ed.
Jones and Bartlet Learning, 2012, Burlington, MA ISBN 978-0-7637-6003-8.
Samuel J, Frankling C. Hypoxemia and hypoxia. In: Myers JA, Millikan KW, Saclarides TJ, eds.
Common Surgical Diseases, 2nd ed. 2008:391–394, Springer Science and Business Media, New York
ISBN 978-0-387-75245-7.

10. E, All of the above.


The treatment of hypoxia depends upon the cause of hypoxia—this concept is fundamental to sleep
disordered breathing therapy. There are multiple causes of tissue hypoxia, and only hypoxic (or anoxic)
hypoxia is associated with low SaO2. A further consideration is that most sleep labs do not measure true
arterial oxygen saturation (SaO2), but rather pulse oximetry (SpO2), a sometimes unreliable proxy. With
pulse oximetry, all measured hemoglobin is assumed to be capable of carrying oxygen; additionally,
pulse oximetry does not take into account changes in pCO2, pH, body temperature or 2,3-DPG levels (a
blood chemical that facilitates release of oxygen from hemoglobin). Readings from pulse oximeters are
also subject to artifact resulting from poor circulation, poor placement, or movement.
How should one respond if low SaO2 is related to blood hypoxemia? If the cause of hypoxemia is
upper airway obstruction (OSA), removal of this obstruction is the solution. Continuous positive airway
pressure (CPAP) remains the most effective method for treating OSA. In some instances, repositioning
the patient's neck may provide some benefit, particularly if it is hyper- or hypo-extended.
If the cause of hypoxemia is an underlying lung disease, supplemental oxygen may be indicated.
Increasing inspired oxygen levels (FiO2) will also increase pO2 (pO2 = FiO2 × PB1) as well as the rate
of diffusion and the amount of oxygen binding (SaO2), thus making more oxygen available to the
tissues.
Patients with severe anemic, circulatory or histotoxic hypoxia should be transferred from the sleep
lab to an emergency room or hospital for more intensive medical management.
See Fundamentals of Sleep Technology, 2nd edition, Chapter 7, page 68.

FURTHER READING ON CORRECTING HYPOXIA: West JB. Pulmonary Pathophysiology: The


Essentials, 8th ed. Lippincott Williams & Wilkins, 2012. ISBN 978-1-4511-0713-5.
Barrett KE, et al. Ganong's Review of Medical Physiology, 24th ed. New York: The McGraw-Hill
Companies, 2012.
Tintinalli JE, et al. Tintinalli's Emergency Medicine: A Comprehensive Study Guide, 7th ed. New York:
The McGraw-Hill Companies, 2011.
Porter RS ed., Acute hypoxemic respiratory failure (AHRF, ARDS). The Merck Manuals: The Merck
Manual for Healthcare Professionals. Merck & Co., 2011, New York.
Kent BD, et al. Hypoxemia in patients with COPD: cause, effects, and disease progression. Int J Chron
Obstruct Pulmon Dis. 2011;6:199–208.
Nussbaumer-Ochsner Y, et al. Sleep and breathing in high altitude pulmonary edema susceptible
subjects at 4,559 meters. Sleep 2012;35:1413.
EXAMPLE NO. 1
11. A trial of nocturnal oxygen supplementation via nasal cannula is indicated.
Oxygen therapy is started at 1 L/min. Her SaO2 is 93% while awake, but
falls to 85% during sleep. Supplemental oxygen is increased to 2 L/min and
results in a sleep SaO2 of 88%. Ten minutes later (allowing time for
equilibration and stabilization), supplemental oxygen is further increased to 3
L/min. Oxygen saturation increases to 92%. Patient is maintained at 3 L/min
O2 therapy until she again goes into REM sleep. Oxygen saturation plunges
to 86%, and an additional liter per minute of supplemental oxygen is added.
Baseline SaO2 remains above 90%, and according to the sleep laboratory's
protocol, treatment with 4 L/min of O2 via nasal cannula is adequate for this
patient (Table 4-1).
Table 4-1

EXAMPLE NO. 2
12. After 2 hours of baseline testing, patient is started on continuous positive
airway pressure (CPAP) therapy, and the latter is titrated to 9 cm H2O.
Apneas, hypopneas and snoring are nearly eliminated, and patient's lowest
SaO2 at this CPAP setting is 94%. Hypopneas recur during REM sleep,
prompting an increase in pressure to 10 cm H2O; this results in resolution of
obstructive events as well as maintenance of a stable SaO2 above 92%.
Supplemental oxygen therapy is not indicated for this patient (Table 4-2).

Table 4-2

SAMPLE PROBLEM NO. 1


13. To calculate CaO2:
Substituting,
Thus,

Although all of these values are normal, it should be noted that the patient is receiving supplemental
oxygen enriched to 30%, and one should expect much higher values for SaO2 and pO2. This suggests
that the patient has an underlying hypoxic hypoxia, such as a diffusion abnormality or
ventilation/perfusion disorder. Knowledge of the patient's medical history is vital in understanding the
cause of hypoxia.
SAMPLE PROBLEM NO. 2
14. To calculate CaO2:

Substituting,
Therefore,

The SaO2 is high but the pO2 is low, suggesting an artifact or measurement error (one would expect
an SaO2 of about 92 for this level of pO2). If measurements are accurate, increased body temperature,
high (alkaline) pH, or very low pCO2 (hyperventilation) should be considered as possible causes, all
increasing oxygen binding at lower pressures.
Oxygen concentration (CaO2) is low at 13.2 mL/dL (down about 27%), indicating a moderate hypoxia,
a major cause of which is anemia (normal Hb is 12 to 16 g/dL). This, however, does not explain the low
pO2. This patient may have a coexistent lung disease complicating the identification and treatment of
hypoxia. Supplemental oxygen may improve hypoxemia (pO2) and help treat the lung disease, but will
not correct the underlying anemic hypoxia.
Using pure 100% oxygen (FiO2 = 1.00), saturation can't get any higher than it already is (99%).
Even if pO2 rises to 400 torr,

The oxygen content is still 21% below normal.


Sample Problem No. 3
15. To calculate CaO2:

Substituting,
Hence,

With the patient on supplemental oxygen therapy, his/her pO2 is elevated (as expected, which is a
favorable response), but SaO2 remains low (the SaO2 should be above 99% for this pO2). However,
due to the normal hemoglobin, the patient's oxygen content (CaO2) is adequate at this level and they are
not hypoxic, despite the low SaO2. The cause of the underlying hypoxemia is unclear, and a thorough
medical history and examination is indicated. A hemoglobin abnormality that is preventing full
saturation may be present in this patient.

1P = Barometric Pressure
B
CHAPTER
5
Cardiac Anatomy and Physiology
JON W. ATKINSON

NOTE: This chapter corresponds to Chapter 8 in Fundamentals of Sleep


Technology, 2nd edition.
For questions 1 to 4, use Figure 5-1 to match the heart chamber to its function:

Figure 5-1 Adapted from WPClipart.com.


1. Chamber labeled "A" A. Receive deoxygenated blood from the body
2. Chamber labeled "B" B. Send deoxygenated blood to the lungs
3. Chamber labeled "C" C. Receive oxygenated blood from the lungs
4. Chamber labeled "D" D. Send oxygenated blood to the body

5. The three layers of tissue in the heart, from innermost to the surface, are:
A. Myocardium, endocardium, epicardium
B. Endocardium, myocardium, epicardium
C. Epicardium, myocardium, endocardium
D. Endocardium, epicardium, myocardium

6. The normal electrical conducting pathway of the heart proceeds along which
of the following pathways?
A. Sinoatrial node, atrioventricular bundle, atrioventricular node, left and
right bundle branches, Purkinje fibers
B. Sinoatrial node, atrioventricular node, atrioventricular bundle, left and
right bundle branches, Purkinje fibers
C. Atrioventricular node, atrioventricular bundle, sinoatrial node, left and
right bundle branches, Purkinje fibers
D. Purkinje fibers, left and right bundle branches, atrioventricular bundle,
atrioventricular node, sinoatrial node

For questions 7 and 8: There are two phases in the cardiac cycle
7. The first phase is called ______________, which is associated with
contraction of the myocardium.

8. The second phase is called ______________ and refers to the relaxation of


the cardiac muscle.

9. The heart has an intrinsic mechanism for controlling its rate. This function is
normally under control of the:
A. Atrioventricular (AV) node pacemaker
B. Ventricular pacemaker
C. Sinoatrial (SA) node pacemaker

10. The heart also has external neural influences on the heart rate via the
autonomic nervous system. The ______________ portion of the autonomic
nervous system slows the heart rate down.

11. Slowing of the heart rate is caused by the release of which neural transmitter
substance?
A. Acetylcholine
B. Adenosine
C. Norepinephrine
D. Serotonin

12. The speeding up of the heart rate is caused by the influence of the
______________ portion of the autonomic nervous system.

13. This acceleration of the heart rate is caused by release of the neurotransmitter
substance:
A. Acetylcholine
B. Adenosine
C. Norepinephrine
D. Serotonin

14. Cardiac output is defined as ______________ multiplied by the heart rate (in
minutes).

15. ______________ refers to the venous return of blood to the heart.


16. The resistance against which the heart must pump blood is called
______________.

17. An increase in peripheral resistance to blood flow will cause the cardiac
output to ______________.

18. Normal cardiac output is:


A. 2 to 3 liters per minute (lpm)
B. 5 to 6 lpm
C. 10 to 12 lpm
D. 15 to 20 lpm

19. Normal blood pressure is:


A. 100/60 mm Hg
B. 120/80 mm Hg
C. 140/100 mm Hg
D. 160/120 mm Hg

20. In recording and reporting blood pressure, the upper number represents the
pressure within the blood vessels during ventricular contraction and is
termed ______________ pressure.

21. In recording and reporting blood pressure, the lower number represents the
pressure within the blood vessels between ventricular contractions and is
called ______________ pressure.

22. For the cardiac pump to be efficient, it is important that all the myocardial
cells contract simultaneously. This phenomenon is accomplished by a
special anatomical feature located at the ends of the myocardial cells called
______________. This anatomical feature allows electrical coupling
through low-resistance contacts called ______________.

23. Cardiac action potentials associated with maintaining myocardial contraction


are due to influx of which ion across the plasma membrane?
A. Calcium, Ca2+
B. Chloride, Cl−
C. Potassium, K+
D. Sodium, Na+

ANSWERS
1. A, Right atrium, labeled “A,” receives deoxygenated blood from the body.
See Fundamentals of Sleep Technology, 2nd edition, Chapter 8, page 69.

2. C, Left atrium, labeled “B,” receives oxygenated blood from the lungs.
See Fundamentals of Sleep Technology, 2nd edition, Chapter 8, page 69.

3. D, Left ventricle, labeled “C,” sends oxygenated blood to the body.


See Fundamentals of Sleep Technology, 2nd edition, Chapter 8, page 69.

4. B, Right ventricle, labeled “D,” sends deoxygenated blood to the lungs.


See Fundamentals of Sleep Technology, 2nd edition, Chapter 8, page 69.

5. B, Endocardium, myocardium, epicardium.


See Fundamentals of Sleep Technology, 2nd edition, Chapter 8, page 69.

6. B, Sinoatrial node, atrioventricular node, atrioventricular bundle, left and right


bundle branches, Purkinje fibers.
See Fundamentals of Sleep Technology, 2nd edition, Chapter 8, page 70.

7. Systole.
See Fundamentals of Sleep Technology, 2nd edition, Chapter 8, page 72.

8. Diastole.
See Fundamentals of Sleep Technology, 2nd edition, Chapter 8, page 72.

9. C, Sinoatrial (SA) node pacemaker.


See Fundamentals of Sleep Technology, 2nd edition, Chapter 8, page 70.

10. Parasympathetic.
See Fundamentals of Sleep Technology, 2nd edition, Chapter 8, page 71.

11. A, Acetylcholine.
See Fundamentals of Sleep Technology, 2nd edition, Chapter 8, page 71.

12. Sympathetic.
See Fundamentals of Sleep Technology, 2nd edition, Chapter 8, page 71.

13. C, Norepinephrine.
See Fundamentals of Sleep Technology, 2nd edition, Chapter 8, page 71.

14. Stroke volume.


See Fundamentals of Sleep Technology, 2nd edition, Chapter 8, page 73.
15. Preload.
See Fundamentals of Sleep Technology, 2nd edition, Chapter 8, page 73.

16. Afterload.
See Fundamentals of Sleep Technology, 2nd edition, Chapter 8, page 73.

17. Decrease.
See Fundamentals of Sleep Technology, 2nd edition, Chapter 8, page 73.

18. B, 5 to 6 lpm.
See Fundamentals of Sleep Technology, 2nd edition, Chapter 8, page 73.

19. B, 120/80 mm Hg.


See Fundamentals of Sleep Technology, 2nd edition, Chapter 8, page 73.

20. Systolic.
See Fundamentals of Sleep Technology, 2nd edition, Chapter 8, page 73.

21. Diastolic.
See Fundamentals of Sleep Technology, 2nd edition, Chapter 8, page 73.

22. Intercalated discs; gap junctions.


See Fundamentals of Sleep Technology, 2nd edition, Chapter 8, page 75.

23. A, Calcium, Ca++.


See Fundamentals of Sleep Technology, 2nd edition, Chapter 8, page 74.
CHAPTER
6
General Human Physiology for the
Sleep Technologist
ROXANNE TAYLOR

NOTE: This chapter corresponds to Chapter 9 in Fundamentals of Sleep


Technology, 2nd edition.

1. Production and secretion of certain hormones display a circadian


rhythmicity. The production of melatonin, in contrast, is primarily
influenced by:
A. Onset of sleep
B. Level of exercise
C. Exposure to light
D. Level of stress

2. Impaired kidney function may change the pH balance of the blood, and this
in turn may result in the development of:
A. Narcolepsy
B. Insomnia
C. Nocturnal eating disorder
D. Sleep-disordered breathing

3. A common complaint of a patient with gastroesophageal reflux disorder


during sleep is:
A. Restless legs symptoms
B. Nightmares
C. Awakenings associated with coughing
D. Dreaming about a particularly spicy meal

4. Many patients report onset of narcolepsy symptoms following an infection.


This suggests that the destruction of orexin-containing neurons in the
hypothalamus responsible for narcolepsy is the result of:
A. Autoimmune destruction
B. Prion disease
C. Elevated body temperature during the illness
D. Programmed cell death

5. Urinary incontinence that occurs during sleep is called:


A. Leaking
B. Neurogenic bladder
C. Enuresis
D. Sphincter dyscontrol

DISCUSSION QUESTIONS

6. Describe the pathway involved in the suppression of melatonin by light. How


is this process preserved in some blind persons? Discuss how changes in
exposure to light affect melatonin production and sleepiness during a normal
day and during shift work.

ANSWERS

1. C, Exposure to light.
See Fundamentals of Sleep Technology, 2nd edition, Chapter 9, page 79.

2. D, Sleep-disordered breathing.
See Fundamentals of Sleep Technology, 2nd edition, Chapter 9, page 82.

3. C, Awakenings associated with coughing.


See Fundamentals of Sleep Technology, 2nd edition, Chapter 9, page 84.

4. A, Autoimmune destruction.
See Fundamentals of Sleep Technology, 2nd edition, Chapter 9, page 85.

5. C, Enuresis.
See Fundamentals of Sleep Technology, 2nd edition, Chapter 9, page 86.
6. “A distinct, non–image-forming subset of retinal ganglion cells containing the
light-sensitive pigment, melanopsin, functions in circadian entrainment by
transducing light into neural impulses that project via the retinohypothalamic
tract (RHT) directly to the SCN, the mammalian master biologic clock.
Retinal afferents to SCN modulate endogenous pacemaker activity and set
the biologic clock in phase with the 24-hour LD cycle. Retinal light exposure
tonically activates SCN neurons through release of multiple
neurotransmitters, including the excitatory amino acid glutamate from RHT
nerve endings, resulting in greater SCN neuronal activity during the
illuminated portion of the LD cycle. GABAergic efferents from SCN inhibit
the activity of the paraventricular hypothalamic nucleus (PVN), ensuring the
maintenance of an inverse activation state relative to SCN. Diminished
nocturnal retinal light exposure lowers tonic stimulation of the SCN and, in
turn, GABAergic inhibition of the PVN. The surge in PVN neuronal
discharge is transmitted via a circuitous pathway that projects caudally to the
upper thoracic intermediolateral cell column, then ascends rostrally as
preganglionic sympathetic fibers to the superior cervical ganglion (SCG),
and finally continues as postganglionic fibers (nervi conarii) to innervate the
pineal gland. The release of norepinephrine from the postganglionic
noradrenergic fibers of SCG promotes the synthesis of melatonin from
serotonin in the pineal gland. Melatonin is secreted into the bloodstream and
detected by the melatonin 1 and 2 receptors (MT-1 and MT-2) in the SCN,
further reducing and modulating its electrical activity in a negative feedback
loop. In the presence of intact retinal photoreceptors, light inhibits, while
darkness promotes, melatonin secretion from the pineal gland. Under natural
conditions, plasma melatonin concentration surges after dusk and returns to
low daytime values with the onset of dawn.” (Shirani A, St. Louis EK.
Illuminating rationale and uses for light therapy. J Clin Sleep Med.
2009;5(2):155–163.)
SECTION II
Sleep Disorders and Disorders that
Affect Sleep
CHAPTER
7
Circadian Rhythms and Circadian
Rhythm Disorders
CHARLOTTE FROMER

NOTE: This chapter corresponds to Chapter 10 in Fundamentals of Sleep


Technology, 2nd edition.

1. Circadian rhythms are physiologic patterns that:


A. Parallel the 24-hour cycle of day and night
B. Parallel the 36-hour cycle of day and night
C. Are the opposite of the 24-hour cycle of day and night
D. Are the opposite of the 36-hour cycle of day and night

2. Provide three examples of human circadian rhythms.


A. ___________________________
B. ___________________________
C. ___________________________

3. The ______________, also known as tau, is the length of the rhythm. It is


usually very close to 24 hours.

4. The ______________ is the magnitude of the rhythm from its peak to nadir.

5. The ______________ refers to the circadian position at any specific instant


of time.

6 Different phase markers are used depending on what type of circadian rhythm
is measured. Identify the phase marker for these circadian rhythms.
CR of core temperature = ______________ and ______________
temperature CR of hormonal rhythms = ______________, ______________,
or ______________ of hormonal secretion

7. Where is the master circadian clock located? ______________ of the


______________.

8. Information about external lighting conditions is conveyed to the


suprachiasmatic nucleus (SCN) from the:
A. Retina through the retinohypothalamic tract
B. Retinohypothalamic tract through the retina
C. Retina through the anterior hypothalamus
D. Retina through the retinohypothalamic SCN

9. The cells in the retina responsible for light and dark information are called
intrinsically photosensitive retinal ______________.

10. The retinal ganglion cells responsible for circadian rhythm entrainment differ
from cells that transmit rod and cone input to the brain for visual image
formation. True or false?

11. What pathways are thought to transmit both photic (light) and nonphotic
information to the circadian clock?
A. Intergeniculate leaflet of the thalamus and the midbrain raphe nuclei
B. Retina and anterior hypothalamus
C. Ganglion cells and retina

12. Secondary efferents from the hypothalamus, basal forebrain, and midline
thalamus project to many brain regions. Name five of them.

13. Over what functions, besides endocrine and body temperature regulation,
does the suprachiasmatic nucleus exert control?
A. Sleep–wake cycle, metabolism, autonomic regulation, psychomotor and
cognitive performance, attention, sight, smell, taste
B. Sleep–wake cycle, metabolism, autonomic regulation, balance, attention,
memory and emotion
C. Sleep–wake cycle, metabolism, autonomic regulation, psychomotor and
cognitive performance, attention, memory and emotion
D. Sleep apnea, metabolism, autonomic regulation, psychomotor and
cognitive performance, attention, memory

14. The light–dark cycle helps to keep a person entrained to the


______________-hour cycle of the external environment.
A. 36
B. 26
C. 24
D. 18

15. If the light–dark cycles are absent, circadian rhythms become


______________. Usually common in ______________.

16. Phase shifting is the process of entraining rhythms to a different time. Name
three circumstances where this may occur.

17. The German term “time giver” or ______________ acts as a time cue to
stimulate a resetting of circadian rhythms.

18. A phase response curve is described as the relationship between:


A. Circadian rhythms, phase shifting, and zeitgebers
B. Light–dark cycle, phase shifting, and zeitgebers
C. Circadian rhythms, free running, and zeitgebers
D. Circadian rhythms, phase shifting, and circadian pacemaker

19. Explain the difference between phase delay and phase advance.

20. SWD or shift work disorder can occur from working what types of shifts?

21. In addition to reduced sleep quality and quantity, what are the symptoms of
shift work disorder?
A. Excessive sleepiness during wake time, nonrestorative sleep, shortened
sleep–wake cycles
B. Increased REM sleep, nonrestorative sleep, shortened sleep duration
C. Excessive sleepiness during wake time, nonrestorative sleep, shortened
sleep duration

22. Driving home for a night shift worker is dangerous because of what circadian
phenomenon?

23. Name five problems unrelated to sleep that shift workers can be susceptible
to.

24. What disorder is caused by rapid crossing of time zones in air travel in the
east or west direction?

25. Symptoms of jet lag disorder include:


A. Sleep loss, excessive sleepiness, decrease in alertness, periodic limb
movements
B. Sleep gain, excessive sleepiness, increase in alertness, gastrointestinal
issues
C. Sleep loss, decreased sleepiness, decrease in alertness, increased slow-
wave sleep
D. Sleep loss, excessive sleepiness, decrease in alertness, gastrointestinal
issues

26. Jet lag disorder is usually transient. True or false?


27. Exposure to inappropriately timed zeitgebers can prolong jet lag. True or
false?

28. Travel to the east is easier to adjust to than is westward travel. True or false?

29. Phase delays occur more slowly than do phase advances. True or false?

30. Symptoms may become chronic for those who frequently fly across multiple
time zones. True or false?

31. Sleep–wake re-entrainment occurs at a rate of approximately one time zone


per day. True or false?

32. Free running occurs when a patient's sleep and wakefulness occur at a
progressively:
A. Earlier clock time each successive day
B. Later clock time each successive week
C. Later clock time each successive day
D. Earlier clock time each successive week

33. The majority of free-running patients have some degree of blindness. True or
false?

34. Which rare disorder is characterized by the absence of a regular sleep–wake


pattern?

35. DSPD is an acronym for a disorder in which sleep and wake cycles are
shifted later than normal, with typical bedtimes between 2 AM and 6 AM.
Name this disorder.

36. ASPD is a disorder in which habitual wake and sleep times occur
substantially earlier than normal, with typical wake times between 2 AM and
4 AM. Name this disorder.

37. A proper evaluation by a clinician is necessary to diagnose circadian rhythm


sleep disorders because they can mimic what other sleep disorders? Name
five.

38. A ______________ is an inexpensive tool, along with the patient's history,


often used to aid in the diagnosis of circadian rhythm sleep disorders.

39. The use of ______________, which is normally worn over a 2-week period,
is to collect objective activity data.

40. Because measuring the activity of the master circadian clock or


suprachiasmatic nucleus cannot be done directly, markers are often used.
These markers may include:
A. Rhythms of core body temperature, rest–activity, and hormones
B. Rhythms of core body temperature, sleep–wake, and hormones
C. Rhythms of core body temperature, rest–activity, and excessive
sleepiness
D. Rhythms of hypersomnia, rest–activity, and hormones

41. The dim light melatonin onset can be used as a circadian phase marker. True
or false?

42. Melatonin is a reliable biomarker for sleep–wake rhythms but can be subject
to masking by ______________.

43. Under what circumstance would polysomnography be appropriate in patients


suspected of having a circadian rhythm sleep disorder?

44. Influence of zeitgebers on intrinsic circadian rhythms cannot be decreased by


using constant routine protocols. True or false?

45. Because light exposure can suppress the circadian rhythm of melatonin,
constant routines involve providing constant dim light conditions. True or
false?

46. Forced desynchrony protocols control light–dark cycle in a laboratory
setting, so the internal circadian clock cannot adjust. True or false?

47. Except for ______________, the first line of treatment for circadian rhythm
sleep disorders is often phototherapy.

48. Sometimes treatment for a circadian rhythm sleep disorder can be as simple
as ______________.

49. For patients diagnosed with shift work disorder, what are some light-
blocking devices that can be used during the daytime? Name two.

50. ______________ can be administered in the early evening for patients with
delayed sleep phase disorder.

51. A treatment for certain circadian rhythm sleep disorders, ______________,


involves a gradual shift of the sleep period by 1 or 2 hours each day.

ANSWERS

1. A, Parallel the 24-hour cycle of day and night.


See Fundamentals of Sleep Technology, 2nd edition, Chapter 10, page 93.

2. Sleep–wake cycle, rhythm of core body temperature, hormonal rhythms.


See Fundamentals of Sleep Technology, 2nd edition, Chapter 10, page 93.

3. Period.
See Fundamentals of Sleep Technology, 2nd edition, Chapter 10, page 93.

4. Amplitude.
See Fundamentals of Sleep Technology, 2nd edition, Chapter 10, page 93.
5. Phase.
See Fundamentals of Sleep Technology, 2nd edition, Chapter 10, page 93.

6. Minimum, maximum; peak, mean, or nadir.


See Fundamentals of Sleep Technology, 2nd edition, Chapter 10, page 93.

7. Suprachiasmatic nuclei, anterior hypothalamus.


See Fundamentals of Sleep Technology, 2nd edition, Chapter 10, page 94.

8. A, Retina through the retinohypothalamic tract.


See Fundamentals of Sleep Technology, 2nd edition, Chapter 10, page 94.

9. Ganglion cells.
See Fundamentals of Sleep Technology, 2nd edition, Chapter 10, page 94.

10. True.
See Fundamentals of Sleep Technology, 2nd edition, Chapter 10, page 94.

11. A, Intergeniculate leaflet of the thalamus and the midbrain raphe nuclei.
See Fundamentals of Sleep Technology, 2nd edition, Chapter 10, page 94.

12. Neocortex, hippocampus, basal ganglia, anterior pituitary, hypothalamus,


reticular formation, pineal gland.
See Fundamentals of Sleep Technology, 2nd edition, Chapter 10, page 94.

13. C, Sleep–wake cycle, metabolism, autonomic regulation, psychomotor and


cognitive performance, attention, memory and emotion.
See Fundamentals of Sleep Technology, 2nd edition, Chapter 10, page 94.

14. C, 24.
See Fundamentals of Sleep Technology, 2nd edition, Chapter 10, page 94.

15. Free running, blindness.


See Fundamentals of Sleep Technology, 2nd edition, Chapter 10, page 95.

16. Jet lag, night shift work, daylight savings time.


See Fundamentals of Sleep Technology, 2nd edition, Chapter 10, page 95.

17. Zeitgeber.
See Fundamentals of Sleep Technology, 2nd edition, Chapter 10, page 95.

18. A, Circadian rhythms, phase shifting, and zeitgebers.


See Fundamentals of Sleep Technology, 2nd edition, Chapter 10, page 95.
19. In phase delay, sleep is shifted to a later time, whereas phase advance involves
a shift in sleep to an earlier time.
See Fundamentals of Sleep Technology, 2nd edition, Chapter 10, page 95.

20. Overnight shift (both fixed and rotating), early morning shift.
See Fundamentals of Sleep Technology, 2nd edition, Chapter 10, page 96.

21. C, Excessive sleepiness during wake time, nonrestorative sleep, shortened


sleep duration.
See Fundamentals of Sleep Technology, 2nd edition, Chapter 10, page 96.

22. Acute reduction in alertness that usually occurs in early morning hours.
See Fundamentals of Sleep Technology, 2nd edition, Chapter 10, page 96.

23. Gastrointestinal issues, cardiovascular disease, hypertension, high cholesterol,


obesity, reproductive difficulties, family and marital issues.
See Fundamentals of Sleep Technology, 2nd edition, Chapter 10, page 96.

24. Jet lag disorder.


See Fundamentals of Sleep Technology, 2nd edition, Chapter 10, page 97.

25. D, Sleep loss, excessive sleepiness, decrease in alertness, gastrointestinal


issues.
See Fundamentals of Sleep Technology, 2nd edition, Chapter 10, page 97.

26. True.
See Fundamentals of Sleep Technology, 2nd edition, Chapter 10, page 97.

27. True.
See Fundamentals of Sleep Technology, 2nd edition, Chapter 10, page 98.

28. False.
See Fundamentals of Sleep Technology, 2nd edition, Chapter 10, page 98.

29. False.
See Fundamentals of Sleep Technology, 2nd edition, Chapter 10, page 97.

30. True.
See Fundamentals of Sleep Technology, 2nd edition, Chapter 10, page 97.

31. True.
See Fundamentals of Sleep Technology, 2nd edition, Chapter 10, page 97.

32. C, Later clock time each successive day.


See Fundamentals of Sleep Technology, 2nd edition, Chapter 10, page 98.

33. True.
See Fundamentals of Sleep Technology, 2nd edition, Chapter 10, page 99.

34. Irregular sleep–wake rhythm.


See Fundamentals of Sleep Technology, 2nd edition, Chapter 10, page 99.

35. Delayed sleep phase disorder.


See Fundamentals of Sleep Technology, 2nd edition, Chapter 10, page 100.

36. Advanced sleep phase disorder.


See Fundamentals of Sleep Technology, 2nd edition, Chapter 10, page 101.

37. Narcolepsy, idiopathic hypersomnia, insufficient sleep syndrome, insomnia,


poor sleep hygiene, medical and psychiatric disorders.
See Fundamentals of Sleep Technology, 2nd edition, Chapter 10, page 102.

38. Sleep–wake diary.


See Fundamentals of Sleep Technology, 2nd edition, Chapter 10, page 102.

39. Wrist actigraphy.


See Fundamentals of Sleep Technology, 2nd edition, Chapter 10, page 102.

40. A, Rhythms of core body temperature, rest–activity, and hormones.


See Fundamentals of Sleep Technology, 2nd edition, Chapter 10, page 102.

41. True.
See Fundamentals of Sleep Technology, 2nd edition, Chapter 10, page 103.

42. Light.
See Fundamentals of Sleep Technology, 2nd edition, Chapter 10, page 103.

43. If the possibility of comorbid sleep apnea, periodic limb movements,


paradoxical insomnia, or narcolepsy/idiopathic hypersomnia exists.
See Fundamentals of Sleep Technology, 2nd edition, Chapter 10, page 102.

44. False.
See Fundamentals of Sleep Technology, 2nd edition, Chapter 10, page 103.

45. True.
See Fundamentals of Sleep Technology, 2nd edition, Chapter 10, page 103.

46. False.
See Fundamentals of Sleep Technology, 2nd edition, Chapter 10, page 103.

47. Free-running disorder.


See Fundamentals of Sleep Technology, 2nd edition, Chapter 10, page 95.

48. Exposure to natural sunlight.


See Fundamentals of Sleep Technology, 2nd edition, Chapter 10, page 104.

49. Sunglasses, blackout shades.


See Fundamentals of Sleep Technology, 2nd edition, Chapter 10, page 104.

50. Melatonin.
See Fundamentals of Sleep Technology, 2nd edition, Chapter 10, page 104.

51. Chronotherapy.
See Fundamentals of Sleep Technology, 2nd edition, Chapter 10, page 104.
CHAPTER
8
Narcolepsy
CYNTHIA MATTICE

NOTE: This chapter corresponds to Chapter 11 in Fundamentals of Sleep


Technology, 2nd edition.

1. Symptoms of narcolepsy typically develop during the ______________


decade of life.
A. First
B. Second
C. Third
D. Fourth

2. Compared to the general population, risk of narcolepsy among first-degree


relatives is:
A. Twice as high
B. Five times higher
C. Ten to forty times higher
D. One hundred times higher

3. The standard diagnostic evaluation of a patient suspected of narcolepsy


without cataplexy requires:
A. Serum hypocretin levels
B. Multiple sleep latency testing
C. Stanford sleepiness scale
D. Brain imaging

4. Over the course of an entire 24-hour day, the total amount of sleep obtained
by narcoleptic patients is:
A. Twice the normal amount for age
B. About 12 hours on average
C. Less than half of the usual amount for age
D. About the same as the usual amount for age
5. A teenager reports that she unexpectedly won an award for best article in her
school paper. She was surprised and pleased but suddenly felt weak in the
knees and unable to get up from her chair to accept the award. She was fully
conscious during this event. Which of the following is the most likely
diagnosis?
A. Cataplexy
B. Generalized seizure
C. Panic attack
D. Vasovagal attack

6. Treatment of narcolepsy involves:


A. Gene replacement therapy
B. Neurosurgery
C. Relief of symptoms
D. Brief course of immunosuppression

7. Many medications for cataplexy are also used to treat:


A. Schizophrenia
B. Depression
C. Epilepsy
D. Generalized anxiety disorder

8. Polysomnographic features of narcolepsy include:


A. Shortened latency to stage R sleep
B. Prolonged latency to stage R sleep
C. Rapid eye movements in all sleep stages
D. Sawtooth waves during stage N3 sleep

9. A medication prescribed to increase alertness in narcolepsy patients is:


A. Sodium oxybate
B. Diazepine
C. Eszopiclone
D. Zolpidem

10. Reduced sleep latency during the Multiple Sleep Latency Test (MSLT) is
defined as a mean sleep latency:
A. Less than 4 minutes
B. Less than 8 minutes
C. Less than 12 minutes
D. Within 30 seconds of lights out

Consider the following history of a sleep center patient: A 17-year-old boy


repeatedly falls asleep in class. Past medical history is unremarkable, and
physical examination is normal. During school days, he falls asleep quickly at
his usual bedtime at midnight and wakes up at 6 AM. In contrast, bedtimes and
wake times are generally later at 2 to 3 AM and 10 to 11 AM on weekends. He
often wakes during the night and has always been a restless sleeper. For the past
few years, he has wakened with frightening dreams sometime involving a troll
from a storybook that his mother read to him when he was a child. This troll had
enormous sharp teeth and asked him to answer ambiguous riddles. The penalty
for an incorrect response was death. Once, after pondering a riddle, the patient
woke abruptly and felt unable to move for several minutes. On several
occasions, just as he was falling asleep, he experienced a very realistic sensation
of the troll nibbling on his feet. Problems with daytime sleepiness began
approximately a year ago after a week off of school due to an upper respiratory
tract illness. When he returned to class, he found himself unable to stay awake
during classes. He naps for 45 minutes regularly after school at 4 PM. He reports
several episodes of feeling weak when laughing. On one occasion, when
watching the Colbert Report, he found a piece on “Truthiness” particularly funny
and his knees buckled, causing him to slump to the floor.
Match the symptoms with the description in the history:
11. Cataplexy A. He dreamt that he was being chased by a troll from a
storybook that his mother read to him when he was a
child.
12. Sleep B. On several occasions, just as he was falling asleep, the
paralysis patient had a very realistic sensation of the troll nibbling
on his feet.
13. Vivid dreams C. When watching the Colbert Report, he found a piece on
"Truthiness" particularly funny and his knees buckled.
14. Excessive D. After pondering the riddle, the patient woke abruptly and
daytime found he was unable to move for an extended period of
sleepiness time.
15. Hypnagogic E. When he returned to class, he found himself unable to stay
hallucinations awake during math, which was previously his favorite
class.
He is prescribed modafinil (200 mg) to be taken each morning. He reports that
this medication helps him stay awake in school but that he still has to take naps
in the afternoon. What other recommendations are appropriate to help him stay
awake during the day?
For questions 16 through 20 indicate if the treatment for this patient is
appropriate or inappropriate.
16. Imipramine hydrochloride (Tofranil) for treatment of cataplexy as needed

17. Increase dose of modafinil to 600 mg per day


18. Monitor efficacy of medication by admitting the patient to hospital for 24-
hour EEG monitoring

19. Encourage sleep hygiene by instructing the patient to avoid naps in the
afternoon

20. Improve sleep “structure” by encouraging regular bed and wake times on
weekdays and weekends and adequate time for sleep

DISCUSSION QUESTIONS:

21. Polysomnography and multiple sleep latency testing are not routinely
indicated for diagnosing narcolepsy in patients with documented symptoms
of cataplexy. Why are these tests not necessary?

22. Hypocretin analysis is useful in the diagnosis of narcolepsy but requires


lumbar puncture to obtain cerebrospinal fluid for testing. This is an invasive
procedure with some risk. Discuss conditions when this diagnostic test
would be useful.

23. Modafinil is an effective treatment for narcolepsy. Weigh the risks and
benefits of prescribing this drug to an adolescent patient.

ANSWERS

1. B, Second.
See Fundamentals of Sleep Technology, 2nd edition, Chapter 11, page 108.

2. C, Ten to forty times higher.


See Fundamentals of Sleep Technology, 2nd edition, Chapter 11, page 108.

3. B, Multiple sleep latency testing.


See Fundamentals of Sleep Technology, 2nd edition, Chapter 11, page 112.

4. D, About the same as the usual amount for age.


See Fundamentals of Sleep Technology, 2nd edition, Chapter 11, page 112.

5. A, Cataplexy.
See Fundamentals of Sleep Technology, 2nd edition, Chapter 11, page 111.

6. C, Relief of symptoms.
See Fundamentals of Sleep Technology, 2nd edition, Chapter 11, page 113.

7. B, Depression.
See Fundamentals of Sleep Technology, 2nd edition, Chapter 11, page 114.

8. A, Shortened latency to stage R sleep.


See Fundamentals of Sleep Technology, 2nd edition, Chapter 11, page 111.

9. A, Sodium oxybate.
See Fundamentals of Sleep Technology, 2nd edition, Chapter 11, page 114.

10. B, Less than 8 minutes.


See Fundamentals of Sleep Technology, 2nd edition, Chapter 11, page 112.

11. C, When watching the Colbert Report, he found a piece on “Truthiness”


particularly funny and his knees buckled.
See Fundamentals of Sleep Technology, 2nd edition, Chapter 11, page 111.
12. D, After pondering the riddle, the patient woke abruptly and found he was
unable to move for an extended period of time.
See Fundamentals of Sleep Technology, 2nd edition, Chapter 11, page 111.

13. A, He dreamt that he was being chased by a troll from a storybook that his
mother read to him when he was a child.
See Fundamentals of Sleep Technology, 2nd edition, Chapter 11, page 111.

14. E, When he returned to class, he found himself unable to stay awake during
math, which was previously his favorite class.
See Fundamentals of Sleep Technology, 2nd edition, Chapter 11, page 111.

15. B, On several occasions, just as he was falling asleep, the patient had a very
realistic sensation of the troll nibbling on his feet.
See Fundamentals of Sleep Technology, 2nd edition, Chapter 11, page 111.

16. Not appropriate. Treatment is for cataplexy and has no effect on sleepiness.
See Fundamentals of Sleep Technology, 2nd edition, Chapter 11, pages 113–114.

17. Not appropriate. Dose exceeds recommended amount.


See Fundamentals of Sleep Technology, 2nd edition, Chapter 11, pages 113–114.

18. Not appropriate. The test is unnecessary.


See Fundamentals of Sleep Technology, 2nd edition, Chapter 11, pages 113–114.

19. Not appropriate. Scheduled naps are recommended as treatment for


excessive sleepiness due to narcolepsy.
See Fundamentals of Sleep Technology, 2nd edition, Chapter 11, page 115.

20. Appropriate.
See Fundamentals of Sleep Technology, 2nd edition, Chapter 11, page 115.

21. “It is now firmly established that narcolepsy type 1 is caused by deficiencies
in hypocretin signaling, most likely due to a selective loss of hypothalamic
hypocretin-producing neurons. Several animal models lacking hypocretin
neurotransmission demonstrate narcolepsy, indicating a causal relationship.
The vast majority of patients (90% to 95%) with narcolepsy and cataplexy
have undetectable or low (<110 pg/mL) levels of hypocretin-1 in the CSF.”
(American Academy of Sleep Medicine. International classification of sleep
disorders, 3rd ed. Darien, IL: American Academy of Sleep Medicine, 2014.)

22. “Determination of CSF hypocretin-1 concentration to diagnose narcolepsy


might be most useful in ambulatory patients with cataplexy but with a normal
multiple sleep latency test (MSLT) result, or if MSLT is not interpretable,
conclusive, or feasible. Because 98% of patients with hypocretin-1 deficiency
are positive for HLA DQB1*0602, we suggest that HLA typing is a useful
screen before lumbar puncture.” (Bourgin P, Zeitzer JM, Mignot E. CSF
hypocretin-1 assessment in sleep and neurological disorders. Lancet Neurol
2008;7(7):649–662.)

23. “We have a limited repertoire of therapeutic interventions for young people
with this devastating, lifelong neuroimmunologic disorder. Modafinil is an
important agent to promote daytime wakefulness, and we have shown that,
according to our collective clinical experience, it can be a successful and safe
treatment in children and young people with narcolepsy.” (Lecendreux M,
Bruni O, Franco P, et al. Clinical experience suggests that modafinil is an
effective and safe treatment for paediatric narcolepsy. J Sleep Res
2012;21(4):481–483.)
CHAPTER
9
Insomnia
CONNSTANCE SHIVERS-SMITH

NOTE: This chapter corresponds to Chapter 12 in Fundamentals of Sleep


Technology, 2nd edition.

Special Note: Publication of the revised version of the ICSD (American
Academy of Sleep Medicine. International Classification of Sleep Disorders, 3rd
ed. Darien, IL: American Academy of Sleep Medicine, 2014) occurred after the
publication of the Fundamentals of Sleep Technology, second edition, and prior
to the publication of this Workbook. Substantial changes in the diagnosis and
conceptual framework for understanding insomnia were incorporated into ICSD-
3. Insomnia diagnoses are now limited to chronic insomnia disorder, short-term
insomnia disorder, and other insomnia disorder. The concept of comorbid disease
was incorporated into the diagnostic criteria and essential features of insomnia.
A discussion of the previous classification occurs in the section “Clinical and
Pathophysiological Subtypes.” The reader is strongly encouraged to reference
the ICSD-3 for up-to-date information regarding diagnosis of insomnia.

1. Insomnia subtypes include sleep-onset insomnia, sleep maintenance


insomnia, and:
A. Fragmented sleep insomnia
B. Early morning awakening
C. Sleep attack insomnia
D. Dream enactment insomnia

2. Transient or short-term insomnia is expected to resolve:


A. With the administration of a suitable hypnotic
B. Within 1 year
C. After the first or second visit to the sleep specialist
D. When the stressor resolves or the patient adjusts
3. Patients with psychophysiologic insomnia tend to:
A. Have heightened arousal
B. Feel that they never sleep even when presented with objective evidence
C. Progress from short-term insomnia to total sleeplessness over weeks or
months
D. Sleep normally when allowed to self-select bedtime and sleep until noon

4. During the interview, questions about bedtime and wake time allow the sleep
specialist to evaluate the patient for the possibility of:
A. Obstructive sleep apnea
B. Narcolepsy
C. Substance abuse and mood disorders
D. Circadian rhythm sleep disorders

5. One argument in favor of including mood and personality disorder


assessment in the sleep center evaluation process for patients complaining of
insomnia is that:
A. All sleep center patients have mood and/or personality disorders
B. Some patients with insomnia may benefit from mental health services
C. Treating mood and personality disorders almost always cures insomnia
D. Mental health professionals always miss mood and personality disorders
during routine evaluations

6. Diagnostic criteria for insomnia have evolved from a “primary” versus


“secondary” categorization to recognition that most patients have comorbid
disorders that mediate or moderate the complaint of insomnia. Discuss how
this has led to the classification system in ICSD-3.

7. Most theories of the pathophysiology of the psychophysiologic subtype of


insomnia include some component of hyperarousal. Review the evidence
pro and con for the “hyperarousal” theory.

ANSWERS
1. B, Early morning awakening.
See Fundamentals of Sleep Technology, 2nd edition, Chapter 12, page 118.

2. D, When the stressor resolves or the patient adjusts.


See Fundamentals of Sleep Technology, 2nd edition, Chapter 12, page 119.

3. A, Have heightened arousal.


See Fundamentals of Sleep Technology, 2nd edition, Chapter 12, page 120.

4. D, Circadian rhythm sleep disorders.


See Fundamentals of Sleep Technology, 2nd edition, Chapter 12, page 122.

5. B, Some patients with insomnia may benefit from mental health services.
See Fundamentals of Sleep Technology, 2nd edition, Chapter 12, page 125.

6. “The insomnia nosology presented in this text represents a marked departure


from the previous International Classification of Sleep Disorders, second
edition, insomnia classification system in terms of its conceptual framework
and relative simplicity. The previous insomnia nosology of the International
Classification of Sleep Disorders promoted the concept that insomnia can
exist as a primary sleep disorder or arise as a secondary form of sleep
disturbance related to an underlying primary psychiatric, medical, or
substance abuse disorder. However, many symptoms and associated features
of so-called primary and secondary insomnias are overlapping, thus making
differentiation among such subtypes difficult, if not impossible. There is
increasing recognition that even when insomnia arises ‘secondary’ to another
condition, it often develops an independent course over time and may remain
as a clinically significant condition, even if the so-called primary condition is
adequately treated. Evidence suggests that insomnia, if left untreated, may
adversely affect the outcome of these comorbid conditions. Moreover, it
appears that treatment of the insomnia may improve outcome of both the
sleep disturbance and the comorbid conditions. Given these observations,
insomnia seems best viewed as a comorbid disorder that warrants separate
treatment attention.”
See Fundamentals of Sleep Technology, 2nd edition, Chapter 12, page 121–122.

7. “Although insomnia is considered a sleep disorder, its pathophysiology


suggests hyperarousal during sleep and wakefulness. Evidence of
hyperarousal in insomnia includes elevated whole-body metabolic rate
during sleep and wakefulness, elevated cortisol and adrenocorticotropic
hormone during the early sleep period, reduced parasympathetic tone in heart
rate variability, and increased high-frequency electroencephalographic
activity during non–rapid eye movement sleep. Functional imaging studies
demonstrate smaller wake–sleep differences in regional brain metabolism in
individuals with insomnia compared to good sleepers.”
See Fundamentals of Sleep Technology, 2nd edition, Chapter 12, page 119.

ADDITIONAL READING FOR DISCUSSION QUESTIONS:


American Academy of Sleep Medicine. International Classification of Sleep Disorders, 3rd ed. Darien,
IL: American Academy of Sleep Medicine, 2014.
Buysse DJ. Insomnia. JAMA. 2013;309(7):706–716.
CHAPTER
10
Parasomnias
MELINDA TRIMBLE and CONSTANCE SHIVERS-SMITH

NOTE: This chapter corresponds to Chapter 13 in Fundamentals of Sleep


Technology, 2nd edition.

1. Confusional arousals most commonly arise during:


A. The last third of the night
B. Sleep onset
C. Stage N3 sleep
D. The last or next to last REM episode

2. An NREM parasomnia associated with atypical sexual manifestations is:


A. Enuresis
B. Freudian dream expression
C. REM behavior disorder
D. Sexsomnia

3. A variety of medications have been implicated in causing or worsening


parasomnias, especially when the medications are mixed with:
A. Grapefruit juice
B. Alcohol
C. Caffeinated beverages
D. Cognitive–behavioral therapy

4. Episodes of sleepwalking in 6-year-old children are most often treated with:


A. Temazepam
B. Pad and bell devices
C. Low-dose clonidine
D. Observation and protection from harm

5. REM sleep behavior disorder is characterized by:


A. Violent outbursts during sleep
B. Recurrent nightmares
C. Loud screaming followed by a rapid return to sleep
D. Dream enactment

6. Patients experiencing a sudden, loud, imagined noise or explosion occurring


during the transition from waking to sleep are said to have:
A. Exploding head syndrome
B. Hypnic jerks
C. Jactatio capitus nocturnus
D. Scanners syndrome

7. If performed in the evaluation of a parasomnia, polysomnography must


include:
A. Temporal EEG leads
B. Arm and trunk EMG leads
C. Video recording
D. Full seizure montage

CLINICAL VIGNETTES

QUESTIONS 8 TO 10

The parents of a 4-year-old boy bring him to the sleep clinic after witnessing
awakenings with screaming occurring once or twice a week. During these
episodes, which usually happen within the first 2 to 3 hours of sleep, the boy
shows signs of fear, sits up in the bed, and breathes rapidly. During these events,
he is inconsolable. In the morning after these episodes, he has no recall of the
awakening. His behavior is normal during the day, and he does not seem to be
sleepy. Physical examination is unremarkable.

8. What is the most likely diagnosis for these events?


9. What is the most common treatment for this diagnosis?


10. What additional investigations are recommended?


QUESTIONS 11 TO 13

A 64-year-old man is being evaluated for yelling and other abnormal behaviors
that occur during sleep. He is accompanied by his wife who reports that she has
been hit and kicked several times by her husband while he was asleep. The
patient adds that he frequently dreams about someone breaking into his house
and having to fight off the intruder. He has no history of enuresis, sleepwalking,
sleep talking, or sleep terrors as a child. He has a regular bedtime and wake time
and does not drink alcoholic beverages. The patient has recently developed
difficulty getting out of bed. His wife has noticed a tremor in his left hand when
at rest. Polysomnography with video monitoring is performed, which
demonstrates the presence of body movements, yelling, twitching in the leg
electromyogram (EMG), and sustained augmentation of the chin EMG activity
during REM sleep.
11. What is the most likely diagnosis?

12. What is the significance of difficulty getting out of bed and hand tremor in
this patient?

13. What does the polysomnogram demonstrate?


QUESTIONS 14 TO 16

A 33-year-old military veteran has frequent awakenings that began shortly after
his second tour to Afghanistan. His unit was hit by mortar fire and he lost his
best friend during this encounter. His awakenings are associated with dreams,
and the dreams are always related to this mortar attack. After awakening from
these dreams, he finds it difficult to return to sleep. He feels unrefreshed in the
morning but is not sleepy. In fact, he is often agitated and unable to sit still. He
finds that the dreams remind him of his traumatic wartime experience, and these
have hindered his attempts to move on. His friends and family have noted that he
has been irritable and withdrawn lately.
14. What is the most likely diagnosis?

15. Is there a likely comorbid diagnosis?

16. What treatments are thought to be effective?


DISCUSSION QUESTION

17. An important theoretical framework for parasomnias is the overlap of “states


of being” promoted by Mark Mahowald and others. How does this
perspective add to our understanding of parasomnias?

ANSWERS

1. C, Stage N3 sleep.
See Fundamentals of Sleep Technology, 2nd edition, Chapter 13, page 128.

2. D, Sexsomnia.
See Fundamentals of Sleep Technology, 2nd edition, Chapter 13, page 129.

3. B, Alcohol.
See Fundamentals of Sleep Technology, 2nd edition, Chapter 13, page 129.

4. D, Observation and protection from harm.


See Fundamentals of Sleep Technology, 2nd edition, Chapter 13, page 130.

5. D, Dream enactment.
See Fundamentals of Sleep Technology, 2nd edition, Chapter 13, page 130.

6. A, Exploding head syndrome.


See Fundamentals of Sleep Technology, 2nd edition, Chapter 13, page 132.

7. C, Video recording.
See Fundamentals of Sleep Technology, 2nd edition, Chapter 13, page 133.
8. Sleep terrors.
See Fundamentals of Sleep Technology, 2nd edition, Chapter 13, page 130.

9. Treatment may not be required as the disorder will often resolve over
time.
See Fundamentals of Sleep Technology, 2nd edition, Chapter 13, page 130.

10. Not indicated unless there is injury or atypical behavior, or suspicion of


another sleep disorder.
See Fundamentals of Sleep Technology, 2nd edition, Chapter 13, page 130.

11. REM sleep behavior disorder (RBD).


See Fundamentals of Sleep Technology, 2nd edition, Chapter 13, page 130.

12. May be symptoms of parkinsonism, which is associated with RBD.


See Fundamentals of Sleep Technology, 2nd edition, Chapter 13, page 130.

13. These are polysomnographic features of RDB.


See Fundamentals of Sleep Technology, 2nd edition, Chapter 13, page 130.

14. Nightmare disorder.


See Fundamentals of Sleep Technology, 2nd edition, Chapter 13, page 130.

15. Posttraumatic stress disorder.


See Fundamentals of Sleep Technology, 2nd edition, Chapter 13, page 130.

16. Prazosin, image rehearsal therapy, and other forms of cognitive–


behavioral therapy.
See Fundamentals of Sleep Technology, 2nd edition, Chapter 13, page 130.

17. “With the multiplicity of state markers, and the relatively rapid normal cycling
of states requiring recruitment of these numerous physiologic markers, there
are innumerable theoretically possible state combinations. It is likely that
major psychic or neural insults can result in an acquired functional
restructuring of the CNS, which then may interfere with conventional state
determination. There is strong evidence that environmentally mediated events
can and do affect the structure and function of the CNS and that the CNS
displays learning of new neural behaviors (i.e., the development of secondary
epileptogenesis [mirror foci] or acquired sensory synesthesia). Such
dissociated states may play a role in the appearance of the posttraumatic
stress disorder, nocturnal panic attacks, and even in psychogenic dissociative
states. Given the genetic variability of CNS development and its plasticity, the
relentless cycling, and the ever-present multiplicity of endogenous and
environmental influences upon both CNS plasticity and cycling, it is
surprising that state component timing errors have not been identified more
frequently. The drive for complete state determination must be very strong,
indeed. Striking sleep abnormalities have been reported in a wide variety of
degenerative and acquired neurologic conditions. This patient population
should serve as a rich source of ‘high risk for state-dissociation’ subjects.”
(Mahowald MW, Schenck CH. Status dissociatus—a perspective on states of
being. Sleep 1991;14(1):69–79.)
CHAPTER
11
Movement Disorders
MELINDA TRIMBLE

NOTE: This chapter corresponds to Chapter 14 in Fundamentals of Sleep


Technology, 2nd edition.

1. What seizure type emerges from an epileptic foci located in the mesial and
orbital cortex and results in seizures occurring almost exclusively during
sleep?
A. Nocturnal paroxysmal dystonia
B. Nocturnal frontal lobe epilepsy
C. Nocturnal nosology
D. Nocturnal epileptic nyquist

2. Nonepileptic seizures are more commonly seen in young ______________.


A. Women
B. Men

3. Nonepileptic seizures are not common during sleep. True or false?


4. ______________ is an autosomal dominant prion disease, characterized by


difficulties in falling asleep and maintaining sleep.
A. Nocturnal dyskinesias
B. Fragmentary myoclonus
C. Fatal familial insomnia
D. Pseudoseizures

5. Hypnagogic foot tremor is a rhythmic movement of the feet or toes that


occurs during the transition between wake and sleep. True or false?

6. Sleep-related events characterized by discrete periods of intense fear or


discomfort upon awakening occur in relation to:
A. Posttraumatic stress disorder
B. Panic disorder
C. Diurnal movement disorders
D. Hypnic disorder

7. An anxiety disorder with which the patient experiences recurrent


stereotypical anxiety dreams occurring in the aftermath of a traumatic event
is called:
A. Posttraumatic stress disorder
B. Panic disorder
C. Diurnal movement disorders
D. Hypnic disorder

8. ______________ is a motor disorder characterized by resting tremor


persisting in stages N1 and N2 sleep.
A. Alternating leg movement activation (ALMA)
B. Fatal familial insomnia (FFI)
C. Excessive fragmentary myoclonus (EFM)
D. Parkinson disease (PD)

9. Choreic movements seen in selected circumstances such as drug side effect


or toxicity are most commonly seen in patients with:
A. Nocturnal dyskinesias
B. Fragmentary myoclonus
C. Fatal familial insomnia
D. Pseudoseizures

10. List the missing information in this table:


11. Another name for sleep starts is ______________.

12. OSA-induced arousals from NREM (or occasionally REM) sleep may trigger
repeated episodes of sleep-related eating disorder. True or false?

13. When hooking up a patient, where would you place the leg leads and how
many leads would you place on each leg in order to see alternating leg
muscle activation (ALMA)?
A.

B.
C.

14. Which diagram shows the best placement for electrodes on the forearm
extensors for evaluation of parasomnia during a sleep study?
A.
B.

C.
15. Attended video PSG (VPSG) is the accepted and preferred standard
technique in the evaluation of patents with motor events in sleep. True or
false?

16. Actigraphy can be used during:


A. Sleep only
B. Wake only
C. Sleep and wake
D. Should not be used

17. Explain why a detailed history and physical is so important when treating a
patient with a possible movement disorder.

18. Match the disorder with the stage of sleep in which it is predominantly seen.
(Answers may be used more than once.)
Panic attacks A. Wakefulness
Confusional arousals B. Sleep onset
Sleepwalking C. N1
Sleep starts D. N2
Nightmare disorder E. N3
Sleep terrors F. REM
Dissociative disorder G. Transitional
Pseudoseizures H. All stages
Propriospinal myoclonus (PSM)
Posttraumatic stress disorder (PTSD)
Alternating leg muscle activation (ALMA)
Sleep-related eating disorder
Bruxism


19. List the following disorders by name:
EFM = ___________________________
HFT = ___________________________
ALMA = ___________________________
FFI = ___________________________
VPSG = ___________________________
RBD = ___________________________
SRED = ___________________________
CSWS = ___________________________
PSM = ___________________________
ESES = ___________________________
PLMS = ___________________________
BECT = ___________________________
OSAS = ___________________________
NFLE = ___________________________

DISCUSSION QUESTIONS

20. What is the role of the sleep technologist when a patient presents with a
movement disorder during the sleep study?

21. When a sleep study is ordered on a patient with a history of movement


disorder, what are the general guidelines for the montage that should be
used?

22. It is important to stay calm and protect the patient from harm while you are
recording any movement disorder. Explain how you would insure the
patient's safety.

ANSWERS

1. B, Nocturnal frontal lobe epilepsy (NFLE) is an epileptic foci in the frontal


lobe that is seen during sleep.
See Fundamentals of Sleep Technology, 2nd edition, Chapter 14, page 136.

2. A, Women. Nonepileptic seizures are three times more common in women


than in men.
See Fundamentals of Sleep Technology, 2nd edition, Chapter 14, page 148.

3. True. Nonepileptic seizures are not common in sleep but may rarely occur on
awakening from sleep at night.
See Fundamentals of Sleep Technology, 2nd edition, Chapter 14, page 148.

4. C, Lapses from quiet wakefulness into a sleep state with enacted dreams and
loss of slow-wave sleep (SWS) are also characteristic of fatal familial
insomnia (FFI).
See Fundamentals of Sleep Technology, 2nd edition, Chapter 14, page 149.

5. True. Hypnagogic foot tremor (HFT) can also be seen during light NREM
sleep stages N1 and N2.
See Fundamentals of Sleep Technology, 2nd edition, Chapter 14, page 150.

6. B, Panic disorder. Panic attacks may come without warning or without


obvious causes. Symptoms may include heart palpitations, pressure in the
chest, dizziness, nausea, and sweating.
See Fundamentals of Sleep Technology, 2nd edition, Chapter 14, page 151.

7. A, Posttraumatic stress disorder. Posttraumatic stress disorder (PTSD) patients


may strike out violently, and although disturbed sleep is a common
complaint, PSG findings have failed to reveal consistent evidence of sleep
disturbance in all patients.
See Fundamentals of Sleep Technology, 2nd edition, Chapter 14, page 151.
8. D, Sleep dysfunction in Parkinson disease (PD) is reported in 70% to 90% of
PD patients.
See Fundamentals of Sleep Technology, 2nd edition, Chapter 14, page 151.

9. A, Nocturnal dyskinesias. Movements are less intense and more frequent at


night; generally occur during transitions from sleep to wakefulness and in
light NREM sleep.
See Fundamentals of Sleep Technology, 2nd edition, Chapter 14, page 149.

10.

See Fundamentals of Sleep Technology, 2nd edition, Chapter 14, page 150.

11. Hypnic jerks.


See Fundamentals of Sleep Technology, 2nd edition, Chapter 14, page 150.

12. True.
See Fundamentals of Sleep Technology, 2nd edition, Chapter 14, page 149.
Schenck CH, Mahowald MW. Review of nocturnal sleep-related eating disorders. Int J Eat Disord.
1994;15(4):343–356.

13. B, Two electrodes should be placed on the tibialis anterior muscle on both legs
to see the alternating activation.
See Fundamentals of Sleep Technology, 2nd edition, Chapter 14, page 150.

14. A, Electrodes should be placed on the forearm extensors 2 to 4 cm apart.


See Fundamentals of Sleep Technology, 2nd edition, Chapter 14, page 139.

15. True.
See Fundamentals of Sleep Technology, 2nd edition, Chapter 14, page 152.

16. C, Sleep and wake. Actigraphy is a technique of motion detection used during
sleep and waking to record activity.
See Fundamentals of Sleep Technology, 2nd edition, Chapter 14, page 152.
17. Because many of the disorders associated with motor movements during sleep
are sporadic in nature, accurate identification and diagnosis can be clinically
challenging. A detailed sleep and medical history including general medical,
neurologic, psychiatric, and social and family history all form the foundation
for assessment of patients with complaints of movement disorders in sleep.
See Fundamentals of Sleep Technology, 2nd edition, Chapter 14, page 136.
18.
Panic attacks = G, Transitional
Confusional arousals = E, N3
Sleepwalking = E, N3
Sleep starts = B, Sleep onset
Nightmare disorder = F, REM
Sleep terrors = E, N3
Dissociative disorder = A, Wakefulness
Pseudoseizures = A, Wakefulness
Propriospinal myoclonus (PSM) = B, Sleep onset Posttraumatic stress disorder (PTSD) = H, All stages
Alternating leg muscle activation (ALMA) = H, All stages Sleep-related eating disorder = E, N3
Bruxism = H, All stages
See Fundamentals of Sleep Technology, 2nd edition, Chapter 14, page 139.
19.
EFM = Excessive fragmentary myoclonus
HFT = Hypnagogic foot tremor
ALMA = Alternating leg muscle activation
FFI = Fatal familial insomnia
VPSG = Video polysomnography
RBD = REM behavior disorder
SRED = Sleep-related eating disorder
CSWS = Continuous spike and slow-wave sleep PSM = Propriospinal myoclonus
ESES = Electrical status epilepticus of sleep PLMS = Periodic leg movements
BECT = Benign epilepsy of childhood
OSAS = Obstructive sleep apnea syndrome
NFLE = Nocturnal frontal lobe epilepsy
See Fundamentals of Sleep Technology, 2nd edition, Chapter 14.

20. Particular attention should be given to the patient's history. A detailed sleep
and medical history should be available in the patient's chart for review or
may be obtained at the time of the study. Direct observation and detailed
patient notes during the study are important. Notes describing the movements
observed should document what stage of sleep the event occurred in and
whether it was elicited or preceded by an arousal or any other stimulus. The
technologist should refrain from drawing any conclusions but must
objectively describe the behavior.

21. An extended EEG montage and additional EMG recording sites are added to
the PSG montage for evaluation of movement disorders. Sample montages
are shown in Table 11-1.

Table 11-1 Sample Electroencephalographic–Polysomnographic Montages

ECG, electrocardiogram; EMG, electromyogram; EOG, electrooculogram; L,


left; LAT, left anterior tibialis; LED, left extensor digitorum; LFD, left flexor
digitorum; LPT, left posterior tibialis; R, right; RAT, right anterior tibialis; RED,
right extensor digitorum; RFD, right flexor digitorum; RPT, right posterior
tibialis; PTAF, pressure transducer airflow.
Modified from: Polysomnography. In: Chokroverty S, Hening WA, Walters AS, eds. Sleep and Movement
Disorders. Philadelphia, PA: Butterworth-Heinemann, 2003:128.

See Fundamentals of Sleep Technology, 2nd edition, Chapter 14, page 138.

22. There is some risk of injury in patients with unexplained motor activity.
Appropriate precautions should be utilized to ensure the safety of the patient
during the sleep study. A clear policy and procedure should be in place and
followed by the sleep center staff.
Something to think about: The technologist must remain alert and attentive throughout the recording.
To reduce response time getting to the patient in case of an emergency, the patient should be placed in
the bedroom closest to the technologist monitoring area. Bed rails may be helpful. Breakable room
decorations should be removed or taken away from the area around the bed.
CHAPTER
12
Periodic Limb Movement Disorder and
Restless Legs Syndrome
CHARLOTTE FROMER

Note: This chapter corresponds to Chapter 15 in Fundamentals of Sleep


Technology, 2nd edition.

1. Restless legs syndrome (RLS) is also called:


A. Willis-Ekbom disease
B. Periodic limb movement disorder
C. Jumping Frenchmen of Maine disease
D. Clonic activity

2. The greatest impact of RLS occurs in the:


A. Upper extremities
B. Last REM episode of the sleep period
C. First half of the night
D. Early morning hours

3. The clinical criteria for the diagnosis of RLS are often represented by the
acronym URGE. The “R” in URGE stands for:
A. REM sleep, when RLS is most prominent
B. Restraint, which relieves the symptoms of RLS
C. Reflection, because the movements in one limb are always reflected in
the other
D. Rest, which worsens the urge to move

4. Periodic limb movements in sleep (PLMS) are defined by the American


Academy of Sleep Medicine (AASM) 2012 Scoring Manual as bursts of
activity from the:
A. Anterior tibialis muscle
B. Brachioradialis muscle
C. Gluteus maximus muscle
D. Movement detectors at the ankle

5. Diagnosis of RLS:
A. Is based on 5 or more periodic limb movements per hour of sleep
B. Is based on 15 or more periodic limb movements per hour of sleep
C. Is based on 15 or more periodic limb movements per hour of sleep with
arousals
D. Does not require sleep testing

6. Effective treatments for RLS include gabapentin, opiates, and:


A. Tricyclic antidepressants
B. Dopaminergic agents
C. Melatonin
D. Nonamphetamine stimulants

DISCUSSION QUESTIONS

7. Dopaminergic treatment of RLS is associated with some significant side


effects, including increased risk taking and impulsive behaviors, such as
gambling. Do the benefits outweigh the risks of treatment?

8. Dysfunction of the iron system is associated with RLS, and a common test
for patients with RLS is serum ferritin. Is this sufficient to test the iron status
of patients?

ANSWERS

1. A, Willis-Ekbom disease.
See Fundamentals of Sleep Technology, 2nd edition, Chapter 15, page 156.
2. C, First half of the night.
See Fundamentals of Sleep Technology, 2nd edition, Chapter 15, page 156.

3. D, Rest, which worsens the urge to move.


See Fundamentals of Sleep Technology, 2nd edition, Chapter 15, page 157.

4. A, Anterior tibialis muscle.


See Fundamentals of Sleep Technology, 2nd edition, Chapter 15, page 157.

5. D, Does not require sleep testing.


See Fundamentals of Sleep Technology, 2nd edition, Chapter 15, page 167.

6. B, Dopaminergic agents.
See Fundamentals of Sleep Technology, 2nd edition, Chapter 15, page 167.

7. “Overall, dopaminergic agents are the most extensively investigated and used
therapies for the treatment of RLS. Since the prior practice parameter update,
the literature has advanced considerably with regard to both the number and
quality of studies for dopaminergic treatment of RLS. While these agents
confer many benefits, there are some adverse effects that should be
recognized. Similar to patients with Parkinson disease, RLS patients treated
with dopamine agonists may develop dopamine dysregulation syndrome.
These patients may exhibit an addictive pattern of dopamine replacement
therapy use and/or behavioral disturbances including spending and impulse
control disorders such as pathologic gambling, compulsive shopping,
compulsive eating, and hypersexuality. … Values and Trade-Offs:
Pramipexole is upgraded to standard from the previous practice parameter
based on multiple studies showing efficacy in RLS. Pramipexole is typically
well tolerated and side effects are self-limited with cessation of pramipexole
therapy.” (Aurora RN, Kristo DA, Bista SR, et al. The treatment of restless
legs syndrome and periodic limb movement disorder in adults—an update
for 2012: Practice parameters with an evidence-based systematic review and
meta-analyses. Sleep. 2012;35(8):1039–1062.) See Fundamentals of Sleep
Technology, 2nd edition, Chapter 15, page 167.

8. “Iron status should be evaluated in all patients with RLS. In addition to


ferritin, it is helpful to obtain transferrin saturation, which is the ratio of
serum iron to TIBC. Supplemental iron is recommended if ferritin is less
than 50 ng/mL or transferrin saturation is less than 20%.” (Mackie S,
Winkelman JW. Normal ferritin in a patient with iron deficiency and RLS. J
Clin Sleep Med. 2013;9(5):511–513.) See Fundamentals of Sleep
Technology, 2nd edition, Chapter 15, page 167.
CHAPTER
13
Obstructive Sleep Apnea
JOYCE BLACK

Note: This chapter corresponds to Chapter 16 in Fundamentals of Sleep


Technology, 2nd edition.

1. Tidal volume is reduced at sleep onset resulting in mild:


A. Laryngeal spasm
B. Snoring
C. Hypoventilation
D. Hyperoxia

2. Mild obstruction of the upper airway causes turbulent airflow that leads to
tissue vibrations and creates:
A. Oxygen desaturations
B. Cardiac arrhythmias
C. Nasal polyps
D. Snoring

3. Apnea severity is typically defined by:


A. Degree of daytime sleepiness
B. Number of breathing disruptions per hour of sleep
C. Duration of the respiratory events
D. Consequences for the cardiovascular system

4. An apnea–hypopnea index of less than five in an adult is considered:


A. Normal
B. Preobstructive sleep apnea syndrome
C. Mild obstructive sleep apnea syndrome
D. Indicative of sensor failure

5. For patients with positional obstructive sleep apnea (OSA), upper airway
occlusion occurs predominantly while lying:
A. On the left side
B. Supine
C. Prone
D. On the right side

6. In women, there is a marked increase in risk of OSA:


A. During the childbearing years
B. In early middle age
C. After menopause
D. After age 80

7. It has been shown that ______________ immediately before bedtime


increases the likelihood of OSA by three times.
A. Eating
B. Drinking caffeinated beverages
C. Exercise
D. Smoking

8. Sympathetic nervous system activation, hypoxia, hypercapnia, and increased


intrathoracic pressure increase the risk of ______________ in OSA patients.
A. Neurologic disorders
B. PAP therapy intolerance
C. Cardiac arrhythmia
D. Nasal obstruction

9. A pattern of cardiac variability often seen in patients with OSA is:


A. Persistent tachycardia
B. Persistent bradycardia
C. Tachycardia alternating with bradycardia
D. Prolonged QT interval

10. The relationship between OSA and diabetes is:


A. All diabetics have OSA.
B. All OSA patients have diabetes.
C. There is no relationship between diabetes and OSA.
D. Diabetes and OSA are reciprocal independent influences.

11. The relationship between OSA severity and daytime sleepiness is:
A. All sleepy people have severe OSA.
B. All patients with severe OSA have severe excessive daytime sleepiness.
C. Patients with mild OSA do not have daytime sleepiness.
D. Unclear.

12. There is a strong association between OSA and mood disorders, particularly:
A. Depression
B. Bipolar disorder Type II
C. Mania
D. Cyclothymia

13. The Mallampati scale is used to rate:


A. Severity of oxygen desaturation
B. Size of the airway at the back of the throat
C. Risk of suicide in patients with major depressive disorder
D. Risk of stroke in patients with severe OSA

14. When there is a high likelihood of OSA and comorbidities are absent, OSA
may be evaluated using:
A. Audio recordings
B. Oximetry
C. Tensilon test
D. Home sleep testing

15. Devices that prevent or reduce time spent sleeping on the back can be helpful
for OSA patients with:
A. Positional OSA
B. Retrognathia
C. REM-related OSA
D. Arrhythmias

16. The “cornerstone” of treatment for OSA is:


A. Oral appliance therapy
B. Positive airway pressure
C. Weight loss therapy
D. Improved sleep hygiene

17. PAP therapy is useful for:


A. Severe OSA only
B. Mild-to-moderate OSA only
C. Central apnea only
D. All categories of OSA

18. Oral appliances may be used to treat:


A. Cheyne-Stokes breathing pattern
B. Severe OSA
C. Mild-to-moderate OSA
D. Idiopathic central sleep apnea

19. Upper airway surgical management of OSA is typically reserved for patients
with an anatomical obstruction or:
A. Failure of other treatments
B. Morbid obesity
C. Severe apnea-associated arrhythmias
D. REM-predominant OSA

20. Maxillomandibular advancement surgery may be helpful for patients with


OSA, especially those with retrognathia or micrognathia, and works by:
A. Enlarging the posterior airway space
B. Realignment of the teeth
C. Repositioning the hyoid bone
D. Reducing the amount of tissue in the soft palate

DISCUSSION QUESTIONS

21. AASM Guidelines recommend laboratory (attended) polysomnography in


cases in which portable monitoring or home sleep testing is negative. What
is the rationale for this, and is it valid?

22. Risk factors and consequences of untreated OSA are sometimes difficult to
distinguish. Discuss the reciprocal relationship between OSA and cardiac,
endocrine, and neurologic disorders.

ANSWERS

1. C, Hypoventilation.
See Fundamentals of Sleep Technology, 2nd edition, Chapter 16, page 169.

2. D, Snoring.
See Fundamentals of Sleep Technology, 2nd edition, Chapter 16, page 169.

3. B, Number of breathing disruptions per hour of sleep.


See Fundamentals of Sleep Technology, 2nd edition, Chapter 16, page 169.

4. A, Normal.
See Fundamentals of Sleep Technology, 2nd edition, Chapter 16, page 169.

5. B, Supine.
See Fundamentals of Sleep Technology, 2nd edition, Chapter 16, page 170.

6. C, After menopause.
See Fundamentals of Sleep Technology, 2nd edition, Chapter 16, page 171.

7. D, Smoking.
See Fundamentals of Sleep Technology, 2nd edition, Chapter 16, page 171.

8. C, Cardiac arrhythmia.
See Fundamentals of Sleep Technology, 2nd edition, Chapter 16, page 171.

9. C, Tachycardia alternating with bradycardia.


See Fundamentals of Sleep Technology, 2nd edition, Chapter 16, page 171–172.

10. D, Diabetes and OSA are reciprocal independent influences.


See Fundamentals of Sleep Technology, 2nd edition, Chapter 16, page 173.

11. D, Unclear.
See Fundamentals of Sleep Technology, 2nd edition, Chapter 16, page 173.

12. A, Depression.
See Fundamentals of Sleep Technology, 2nd edition, Chapter 16, page 173.

13. B, Size of the airway at the back of the throat.


See Fundamentals of Sleep Technology, 2nd edition, Chapter 16, page 174.

14. D, Home sleep testing.


See Fundamentals of Sleep Technology, 2nd edition, Chapter 16, page 174–5.
15. A, Positional OSA.
See Fundamentals of Sleep Technology, 2nd edition, Chapter 16, page 175.

16. B, Positive airway pressure.


See Fundamentals of Sleep Technology, 2nd edition, Chapter 16, page 176.

17. D, All categories of OSA.


See Fundamentals of Sleep Technology, 2nd edition, Chapter 16, page 176.

18. C, Mild-to-moderate OSA.


See Fundamentals of Sleep Technology, 2nd edition, Chapter 16, page 176.

19. A, Failure of other treatments.


See Fundamentals of Sleep Technology, 2nd edition, Chapter 16, page 177.

20. A, Enlarging the posterior airway space.


See Fundamentals of Sleep Technology, 2nd edition, Chapter 16, page 177.

21. “PMs may be used in the unattended setting as an alternative to PSG for the
diagnosis of OSA in patients with a high pretest probability of moderate-to-
severe OSA and no comorbid sleep disorder or major comorbid medical
disorders when all of the previous parameters are met (Consensus). The
diagnosis of OSA is confirmed and severity determined using the same
criteria as used for PSG. Scoring criteria should be consistent with the current
published AASM standards for scoring of apneas and hypopneas
(Consensus). The term RDI has been defined differently when used with PMs
than when used with PSG. RDI PM is the number of apneas +
hypopneas/total recording time rather than total sleep time. As a result, PMs
are likely to underestimate the severity of events compared to the AHI by
PSG. Due to the known rate of false-negative PM tests, in-laboratory PSG
should be performed in cases where PM is technically inadequate or fails to
establish the diagnosis of OSA in patients with a high pretest probability
(Consensus).” (Epstein LJ, Kristo D, Strollo PJ, et al. Clinical guideline for
the evaluation, management and long-term care of obstructive sleep apnea in
adults. J Clin Sleep Med. 2009;5(3):263–276.) See Fundamentals of Sleep
Technology, 2nd edition, Chapter 16, page 174–175.

22. “A comprehensive sleep history in a patient suspected of OSA should include


an evaluation for snoring, witnessed apneas, gasping/choking episodes,
excessive sleepiness not explained by other factors, including assessment of
sleepiness severity by the Epworth Sleepiness Scale, total sleep amount,
nocturia, morning headaches, sleep fragmentation/sleep maintenance
insomnia, and decreased concentration and memory (Consensus). An
evaluation of secondary conditions that may occur as a result of OSA,
including hypertension, stroke, myocardial infarction, cor pulmonale,
decreased daytime alertness, and motor vehicle accidents, should also be
obtained.” (Epstein LJ, Kristo D, Strollo PJ, et al. Clinical guideline for the
evaluation, management and long-term care of obstructive sleep apnea in
adults. J Clin Sleep Med. 2009;5(3):263–276.) See Fundamentals of Sleep
Technology, 2nd edition, Chapter 16, page 174.
CHAPTER
14
Central Sleep Apnea
JOANNE HEBDING

Note: This chapter corresponds to Chapter 17 in Fundamentals of Sleep


Technology, 2nd edition.

1. A 93-year-old Caucasian man has increasing shortness of breath when


walking to and from his house. He reports a history of loud snoring and
excessive sleepiness. He habitually goes to bed at 11:30 PM and wakes up at
8:00 AM, for a nightly sleep duration of 9.5 hours. Although he sleeps well
and wakes up feeling refreshed, he falls asleep when sitting down or
watching television. He described needing to go to the bathroom several
times during the night.
He has a history of hypertension, coronary artery disease (myocardial
infarction), atrial fibrillation, and sick sinus syndrome. He was undergone
pacemaker placement and coronary artery bypass graft surgery. A lifelong
nonsmoker, he has no known lung disease.
Physical examination reveals a BMI of 28.8 kg/m2 and bibasilar
crackles. Overnight oximetry shows repetitive sawtooth-pattern oxygen
desaturation with a lowest oxygen saturation (SaO2 min) of 76%.
A split-night polysomnogram demonstrates the following respiratory
events (Fig. 14.1).
Which of the following will be most effective in eliminating these
respiratory events?
Figure 14.1 Five-minute window.

A. Continuous positive airway pressure (CPAP)


B. Bilevel positive airway pressure (BPAP)
C. BPAP with a backup rate or adaptive servo ventilation (ASV)
2. All of the following respiratory patterns are classified as central sleep apnea,
except:
A. Cheyne-Stokes respiration
B. High-altitude periodic breathing
C. Obesity hypoventilation syndrome
D. Complex sleep apnea

3. A patient undergoing a split night sleep study has obstructive apneas and
hypopneas, and an apnea hypopnea index (AHI) greater than 40 events per
hour. Central events are rare at baseline but increased during continuous
positive airway pressure (CPAP) titration in stages N2 and N3 sleep. What
type of central apnea is most likely present in this patient?
A. Hypercapnic central sleep apnea
B. Cheyne-Stokes respiration
C. Complex sleep apnea
D. Sleep-onset periodic breathing

4. Hypercapnia is an essential feature of all hypoventilation syndromes. True or


false?

5. Which of the following is a cause of hypercapnic central sleep apnea?


A. Heart failure
B. High altitude
C. Neuromuscular disorder
D. Onset of sleep

6. Cheyne-Stokes respiration (CSR) is characterized by which type of


respiratory pattern?
A. Irregularly regular
B. Relaxed–narrowing
C. Waxing–waning
D. Repetitive–staccato

MATCH THE TERM WITH ITS DEFINITION

7. Central sleep A. Abnormally high CO2 levels


apnea
8. Complex sleep B. Can give rise to obstructive and central sleep apnea as
apnea well as Cheyne-Stokes respiration
9. Hypercapnia C. Automatic airflow-targeted ventilation
10. Hypoventilation D. Abnormal respiratory patterns, including
hypoventilation, central apnea, and Biot breathing
11. Cheyne-Stokes E. Application of continuous positive airway pressure
respiration (CPAP) for obstructive sleep apnea elicits central
apneas
12. Congestive heart E. Slow and/or shallow breathing that results in
failure increased CO2 levels
13. Central nervous G. Cessation of breathing with absence of airflow and
system effort
dysfunction
14. Positive airway H. Rhythmic waxing and waning of breathing with
pressure therapy regularly recurring central apneic periods
15. Adaptive servo I. Pressure delivered in increments of cmH2O
ventilation

TRUE OR FALSE?

16. Hypoventilation syndromes can result in daytime sleepiness and pulmonary


hypertension.

17. Ventilatory response to hypoxia and hypercapnia during sleep is similar to


that during wakefulness.

18. Central apnea can develop after a deep breath, such as a sigh or yawn.

19. Complex sleep apnea is considered a form of nonhypercapnic central sleep


apnea.

20. Upper airways remain patent in central sleep apnea.


21. Complex sleep apnea, also referred to as treatment-emergent apnea, is


defined as the development of central sleep apnea during continuous
positive airway pressure (CPAP) titration or treatment in a patient with
obstructive sleep apnea at baseline.

22. Positive airway pressure therapy has been shown to improve cardiac function
in patients with congestive heart failure.

23. Central sleep apnea can be diagnosed using airflow signals alone without
measuring thoracic wall and abdominal movements.

24. Arousals associated with central sleep apnea generally occur at the
termination of the apneic period.

ANSWERS

1. C, BPAP with a backup rate or adaptive servo ventilation (ASV). The figure
shows central sleep apneic events characterized by a complete loss of
diaphragmatic and intercostal activity associated with decrease in oxygen
saturation (SaO2). Some of the events shown have a waxing–waning pattern.
Depending on the nature of the central events, either bilevel positive airway
pressure (BPAP) with or without a backup respiratory rate, or servo
ventilation may be tried to abolish central sleep apnea.
See Fundamentals of Sleep Technology, 2nd edition, Chapter 17, page 187.

2. C, Obesity hypoventilation syndrome.


See Fundamentals of Sleep Technology, 2nd edition, Chapter 17, page 183.

3. C, Complex sleep apnea.


See Fundamentals of Sleep Technology, 2nd edition, Chapter 17, page 183.

4. True.
See Fundamentals of Sleep Technology, 2nd edition, Chapter 17, page 182.

5. C, Neuromuscular disorder.
See Fundamentals of Sleep Technology, 2nd edition, Chapter 17, page 183.

6. C, Waxing–waning.
See Fundamentals of Sleep Technology, 2nd edition, Chapter 17, page 185.

7. G, Cessation of breathing with absence of airflow and effort.


See Fundamentals of Sleep Technology, 2nd edition, Chapter 17, page 181.

8. E, Application of continuous positive airway pressure (CPAP) for obstructive


sleep apnea elicits central apneas.
See Fundamentals of Sleep Technology, 2nd edition, Chapter 17, page 183.

9. A, Abnormally high CO2 levels.


See Fundamentals of Sleep Technology, 2nd edition, Chapter 17, page 183.

10. F, Slow and/or shallow breathing that results in increased CO2 levels.
See Fundamentals of Sleep Technology, 2nd edition, Chapter 17, page 184.

11. H, Rhythmic waxing and waning of breathing with regularly recurring central
apneic periods.
See Fundamentals of Sleep Technology, 2nd edition, Chapter 17, page 185.

12. B, Can give rise to obstructive and central sleep apnea as well as Cheyne-
Stokes respiration.
See Fundamentals of Sleep Technology, 2nd edition, Chapter 17, page 182.

13. D, Abnormal respiratory patterns, including hypoventilation, central apnea,


and Biot breathing.
See Fundamentals of Sleep Technology, 2nd edition, Chapter 17, page 182.

14. I, Pressure delivered in increments of cmH2O.


See Fundamentals of Sleep Technology, 2nd edition, Glossary, page 636.

15. C, Automatic airflow-targeted ventilation.


See Fundamentals of Sleep Technology, 2nd edition, Chapter 17, page 187.

16. True
See Fundamentals of Sleep Technology, 2nd edition, Chapter 17, page 183.

17. False
See Fundamentals of Sleep Technology, 2nd edition, Chapter 17, page 182.

18. True
See Fundamentals of Sleep Technology, 2nd edition, Chapter 17, page 181.

19. True
See Fundamentals of Sleep Technology, 2nd edition, Chapter 17, page 183.

20. True
See Fundamentals of Sleep Technology, 2nd edition, Chapter 17, page 183.

21. True
See Fundamentals of Sleep Technology, 2nd edition, Chapter 17, page 183.
22. True
See Fundamentals of Sleep Technology, 2nd edition, Chapter 17, page 186.

23. False
See Fundamentals of Sleep Technology, 2nd edition, Chapter 17, page 186.

24. True
See Fundamentals of Sleep Technology, 2nd edition, Chapter 17, page 182.
CHAPTER
15
Seizures and Sleep
MELINDA TRIMBLE

NOTE: This chapter corresponds to Chapter 18 in Fundamentals of Sleep


Technology, 2nd edition.

1. A sensation experienced by some people with epilepsy before a seizure is:


A. Ictal activity
B. An aura
C. Generalized epilepsy
D. Clonic activity

2. A sudden abnormal discharge of electrical activity of the brain usually


affecting how a person acts or feels for a short time is:
A. Epilepsy
B. A seizure
C. Phantom spike and wave
D. Electrical paroxysm

3. Generalized discharges and clinical seizures mostly occur in:


A. Overlap syndromes
B. REM sleep
C. NREM sleep
D. Narcoleptics

4. Patients with primary generalized myoclonic seizures are most likely to have
them:
A. At sleep onset
B. In the transition from N2 to REM
C. On awakening
D. During REM sleep

5. Three per second spike and wave discharges are typically associated with:
A. Brief episodes of unresponsiveness
B. Complex nocturnal wanderings
C. Loss of muscle tone and dropping to the floor
D. Tonic–clonic activity lasting more than 3 minutes

6. Benign focal epilepsy of childhood is associated with abnormal EEG


discharges originating:
A. In the frontal lobe
B. Centrotemporally
C. From multiple areas in the occipital lobe
D. In the orbitofrontal area

7. Nocturnal seizures with paroxysmal arousal, dystonia, or episodic nocturnal


wandering are classified as subgroups of:
A. Benign focal epilepsy of childhood
B. Nocturnal frontal lobe epilepsy
C. Lennox-Gastaut syndrome
D. Electrical status epilepticus of sleep

8. A 10-year-old boy has generalized spike and wave complexes in his EEG
tracing. Spike waves are present in 90% of stage N3 sleep, but are much less
during wake and REM sleep. What is the most likely diagnosis?
A. Benign focal epilepsy of childhood
B. Nocturnal frontal lobe epilepsy
C. Lennox-Gastaut syndrome
D. Electrical status epilepticus of sleep

9. The most commonly used treatment for seizures is:


A. Medication
B. Behavioral therapy
C. Vagal nerve stimulation
D. Surgical intervention

10. In patients suspected of a seizure disorder, additional electrodes may be


applied, especially those over:
A. Frontal and temporal lobes
B. Occipital and parietal lobes
C. Central and occipital lobes
D. Infraorbital cortex
11. Recommendations for technologist response during a seizure include
remaining calm and:
A. Minding the gap
B. Disconnecting the head box
C. Activating EMS only if the patient has a cardiac arrest
D. Continuing the recording

12. The patient having a seizure should be protected from harm. This should
include:
A. Restraining the patient during clonic movements
B. Turning the patient on his or her side
C. Placing a tongue depressor in the patient's mouth
D. Shouting at the patient to try and arouse him or her

13. Ms. Sparrow, a 22-year-old woman with nocturnal episodes of uncertain


etiology, arrives at the sleep disorders center for evaluation. Her episodes
consist of abruptly sitting up in bed, calling out, and on occasion getting out
of bed and wandering through the house in a semiconscious state. A
polysomnogram is ordered. Which of the following are appropriate
preparations for the test?

DISCUSSION QUESTIONS

14. One of the more difficult clinical distinctions is between nocturnal epilepsy
and parasomnias. Are there clinical features of nocturnal seizures that can be
used to make this differentiation?

15. Sleep deprivation is known to increase the risk of seizures and has been used
for decades in clinical electroencephalography to increase the probability of
recording interictal abnormalities. What physiologic changes occur with
sleep deprivation that might explain this phenomenon?

ANSWERS

1. B, An aura.
See Fundamentals of Sleep Technology, 2nd edition, Chapter 18, page 189.

2. B, A seizure.
See Fundamentals of Sleep Technology, 2nd edition, Chapter 18, page 189.

3. C, NREM sleep.
See Fundamentals of Sleep Technology, 2nd edition, Chapter 18, page 190.

4. C, On awakening.
See Fundamentals of Sleep Technology, 2nd edition, Chapter 18, page 191.

5. A, Brief episodes of unresponsiveness.


See Fundamentals of Sleep Technology, 2nd edition, Chapter 18, page 192.

6. B, Centrotemporally.
See Fundamentals of Sleep Technology, 2nd edition, Chapter 18, page 194.

7. B, Nocturnal frontal lobe epilepsy.


See Fundamentals of Sleep Technology, 2nd edition, Chapter 18, pages 195–196.

8. D, Electrical status epilepticus of sleep.


See Fundamentals of Sleep Technology, 2nd edition, Chapter 18, page 196.

9. A, Medication.
See Fundamentals of Sleep Technology, 2nd edition, Chapter 18, page 196.

10. A, Frontal and temporal lobes.


See Fundamentals of Sleep Technology, 2nd edition, Chapter 18, page 197.
11. D, Continue the recording.
See Fundamentals of Sleep Technology, 2nd edition, Chapter 18, page 199.

12. B, Turning the patient on his or her side.


See Fundamentals of Sleep Technology, 2nd edition, Chapter 18, page 199.
13.

See Fundamentals of Sleep Technology, 2nd edition, Chapter 18, pages 197–199.

14. “The differential diagnosis of nocturnal seizures and parasomnias is often


successfully achieved based on the clinical context, the timing and frequency
of events, and results of EEG and polysomnography. In challenging cases,
however, the diagnosis depends upon an analysis of ictal semiology of events
recorded on video–EEG monitoring. Even this may not yield a clear
diagnosis, largely because the features of parasomnias (as compared to that of
nocturnal frontal lobe seizures), although well known in outline, have not
been adequately described in detail. In this study we have addressed this issue
by analyzing the semiology of the NREM arousal parasomnias in detail, and
identifying important features that help distinguish them from the seizures of
NFLE.” (Derry CP, Harvey AS, Walker MC, et al. NREM arousal
parasomnias and their distinction from nocturnal frontal lobe epilepsy: a
video EEG analysis. Sleep. 2009;32(12):1637–1644.) See Fundamentals of
Sleep Technology, 2nd edition, Chapter 18, pages 195–196.

15. “Although the restricted number of subjects and age limitation to adulthood
prevent any generalization, we suggest that the temporary loss of inhibition,
induced by total sleep deprivation, in healthy subjects might be important for
explaining the facilitatory influence of SD upon seizures. The SD, in fact,
may modify the cortical excitability, seen as the balance between inhibitory
and excitatory cortical phenomena, which could reduce the epileptic
threshold.” (Scalise A, Desiato MT, Gigli GL, et al. Increasing cortical
excitability: a possible explanation for the proconvulsant role of sleep
deprivation. Sleep. 2006;29(12):1595–1598.) See Fundamentals of Sleep
Technology, 2nd edition, Chapter 18, pages 190–191.
CHAPTER
16
Sleep and Medical Disorders
CYNTHIA MATTICE

NOTE: This chapter corresponds to Chapter 19 in Fundamentals of Sleep


Technology, 2nd edition.

1. Polysomnography in patients with chronic obstructive pulmonary disease


(COPD) frequently demonstrates:
A. Increased sleep efficiency
B. Reduced awakenings during the night
C. Increased total sleep time
D. Increased arousal frequency

2. Oxygen desaturation in patients with chronic obstructive pulmonary disease


(COPD) is generally worse during:
A. Stage N1
B. Stage N2
C. Stage N3
D. REM

3. Which of the following disorders should be considered in a 425-pound man


with obstructive sleep apnea, moderate hypoxia, and significant hypercarbia
during a sleep study?
A. Obesity hypoventilation syndrome
B. Nocturnal asthma
C. Diaphragm paralysis
D. Neuromuscular disorder

4. Patients with neuromuscular disorders can develop respiratory difficulties


during sleep due to:
A. Nonhypercapnic central sleep apnea
B. Hypoventilation
C. Ineffective clearing of airway secretions
D. Both B and C

5. Patients with symptomatic gastroesophageal reflux may have:


A. Reduced stage N1 sleep
B. Nocturnal asthmatic attacks
C. Frequent arousals in sleep
D. Both B and C

6. Patients with acromegaly are at increased risk for:


A. Insomnia
B. Narcolepsy
C. Restless legs syndrome
D. Obstructive and central sleep apnea

7. Fatal familial insomnia is a prion disease that shares pathophysiologic


features with:
A. Multiple sclerosis
B. Myasthenia gravis
C. Alzheimer disease
D. Creutzfeldt-Jakob disease

8. Which of the following is a nonspecific EEG pattern seen in some patients


with fibromyalgia?
A. Alpha–delta sleep
B. Fourteen and six spike and wave
C. Occipital slow waves
D. Hyperspindling

MATCH THE TERM WITH THE DEFINITION

9. Overlap A. A group of genetic diseases characterized by progressive


syndrome weakness and degeneration of skeletal muscles
10. Cushing B. A hormone released by the cortex of the adrenal gland when
syndrome an individual experiences stress
11. Muscular C. Inflammatory disease of the airways characterized by
dystrophy episodic dyspnea and wheezing
12. Cortisol D. Coexisting COPD and OSA
13. Myasthenia E. Patients have an excess of adrenocorticosteroid hormones,
gravis usually the hormone cortisol
14. Asthma F. Chronic autoimmune disease characterized by varying
degrees of skeletal muscle weakness
15. Discuss the importance of medical comorbidities in the different sleep
disorders. How do these influence treatment strategies?

ANSWERS

1. D, Increased arousal frequency.


See Fundamentals of Sleep Technology, 2nd edition, Chapter 19, page 203.

2. D, REM.
See Fundamentals of Sleep Technology, 2nd edition, Chapter 19, page 203.

3. A, Obesity hypoventilation syndrome.


See Fundamentals of Sleep Technology, 2nd edition, Chapter 19, page 203.

4. D, Both B and C.
See Fundamentals of Sleep Technology, 2nd edition, Chapter 19, page 204.

5. D, Both B and C.
See Fundamentals of Sleep Technology, 2nd edition, Chapter 19, page 205.

6. D, Obstructive and central sleep apnea.


See Fundamentals of Sleep Technology, 2nd edition, Chapter 19, page 206.

7. D, Creutzfeldt-Jakob disease.
See Fundamentals of Sleep Technology, 2nd edition, Chapter 19, page 209.

8. A, Alpha–delta sleep.
See Fundamentals of Sleep Technology, 2nd edition, Chapter 19, page 207.

9. D, Coexisting COPD and OSA.


See Fundamentals of Sleep Technology, 2nd edition, Chapter 19, page 203.

10. E, Patients have an excess of adrenocorticosteroid hormones, usually the


hormone cortisol.
See Fundamentals of Sleep Technology, 2nd edition, Chapter 19, page 207.

11. A, A group of genetic diseases characterized by progressive weakness and


degeneration of skeletal muscles.
See Fundamentals of Sleep Technology, 2nd edition, Chapter 19, page 204.

12. B, A hormone released by the cortex of the adrenal gland when an individual
experiences stress.
See Fundamentals of Sleep Technology, 2nd edition, Chapter 19, page 207.

13. F, Chronic autoimmune disease characterized by varying degrees of skeletal


muscle weakness.
See Fundamentals of Sleep Technology, 2nd edition, Chapter 19, page 204.

14. C, Inflammatory disease of the airways characterized by episodic dyspnea and


wheezing.
See Fundamentals of Sleep Technology, 2nd edition, Chapter 19, page 202.

15. “The clinical presentation varies with the underlying disorder responsible for
the sleep-related hypoventilation. Chronic obstructive pulmonary disease is
characterized by generally fixed and not fully reversible lower airways
obstruction, and includes chronic bronchitis, emphysema, cystic fibrosis, and
bronchiectasis. Chronic bronchitis is a clinical entity defined by the presence
of chronic productive cough for at least 3 months of the year, for at least 2
consecutive years, in the absence of other identifiable etiologies. Emphysema
is characterized by destruction of lung tissue and the dilation of peripheral
airspaces without evident fibrosis. Emphysema and chronic bronchitis often
coexist. Alpha-1 antitrypsin deficiency is a genetic cause of chronic
obstructive pulmonary disease. Both bronchiectasis and cystic fibrosis are
characterized by lower airway inflammation and destruction of airways and
lung parenchyma. Patients with chronic lower airways obstruction are
increasingly predisposed to developing hypoventilation as the severity of the
underlying lower airways obstruction increases.” (American Academy of
Sleep Medicine. International Classification of Sleep Disorders, 3rd ed.
Darien, IL: American Academy of Sleep Medicine, 2014.)
CHAPTER
17
Psychiatric Disorders That Affect Sleep
KAREN I. SMITH

NOTE: This chapter corresponds to Chapter 22 in Fundamentals of Sleep


Technology, 2nd edition.

1. Mood disorders include:


A. Major depressive disorder (MDD)
B. Dissociative disorder
C. Schizophrenia
D. Attention deficit hyperactivity disorder (ADHD)

2. Depression is most likely to begin:


A. At retirement
B. During adolescence or early adulthood
C. At age 40
D. In extreme old age

3. Total or partial deprivation of which sleep stage can result in significant


improvement in depressive symptoms?
A. N1 sleep
B. N2 sleep
C. N3 sleep
D. REM sleep

4. Major depressive disorder (MDD) is typically associated with:


A. Increased percentage of N3 sleep
B. Elevated sleep efficiency
C. Reduction or absence of N2 sleep
D. Early morning awakening

5. Changes in REM sleep associated with a major depressive episode consist


of:
A. Decreased REM density
B. Prolonged REM sleep latency
C. Increased REM density
D. Decreased REM sleep

6. Most antidepressant medications can give rise to:


A. Shortened REM latency
B. Decreased REM sleep
C. Increased REM density
D. Decreased stage N3 sleep

7. Bipolar disorder is characterized by the episodes of depression, mania, and:


A. Hypomania
B. Hyperphagia
C. Hallucinations
D. Halitosis

8. During the manic phase of a bipolar disorder, patients frequently have:


A. Excessive sleepiness
B. Periods of sleeplessness
C. Decreased REM sleep
D. Severe phase advance

9. During the winter, patients with seasonal affective disorder (SAD) frequently
complain of:
A. Anxiety
B. Decreased sleep requirements
C. Hypersomnia
D. Hypomanic episodes

10. Seasonal affective disorder (SAD) may improve with:


A. Sedative hypnotics
B. Antihistamine drugs
C. Melatonin
D. Bright light therapy

11. Patients with generalized anxiety disorder (GAD) commonly have difficulty
with:
A. Sleep onset and sleep maintenance insomnia
B. Waking up in the morning
C. Staying awake during the day
D. Sleep paralysis

12. Patients with panic disorder may experience:


A. Hypnagogic hallucinations
B. Sleep attacks
C. Cataplexy
D. Insomnia

13. Obsessive–compulsive disorder (OCD) patients often exhibit:


A. Decreased wake time after sleep onset (WASO)
B. Increased total sleep time (TST)
C. Delayed bedtimes
D. Increased sleep efficiency

14. Posttraumatic stress disorder (PTSD) is often associated with:


A. Nocturnal epilepsy
B. Increased stage N3 sleep
C. Advanced sleep phase
D. Anxiety dreams

15. Among other things, schizophrenia is characterized by:


A. Sleep attacks
B. Hypomania
C. Depressed mood
D. Delusional beliefs and inappropriate behavior

16. Following REM sleep deprivation, schizophrenic patients exhibit:


A. N3 rebound
B. Paradoxical decreases in REM sleep time
C. A lack of REM rebound
D. Total but temporary relief from symptoms

17. It takes a typical healthy adult about _______________ to metabolize a can


of beer, glass of wine, or shot of whiskey (10 mL of pure alcohol).
A. 30 minutes
B. 45 minutes
C. 60 minutes
D. 90 minutes

18. Changes in sleep architecture following acute ingestion of alcohol include:


A. Decreased REM sleep in the first half of the sleep period
B. Decreased REM sleep in the second half of the sleep period
C. Decreased REM sleep latency
D. Increased sleep onset latency (SOL)

19. Sleep-disordered breathing may be missed if a habitual alcohol user:


A. Refrains from drinking on the night of the test
B. Has a large meal before coming to the sleep center
C. Sleeps only on his or her back during the study
D. Has wine instead of beer on the night of the test

20. Alcohol use is associated with:


A. Cataplexy
B. Increased sleep efficiency
C. Paradoxical insomnia
D. Enuresis and nightmares

Matching
Match the proper phrase or term with the appropriate definition.

21. Generalized anxiety A. Intense, irrational fear of an object or situation


disorder (GAD)
22. Panic disorder (PD) B. Persistent, upsetting, and unwanted thoughts that
may motivate repetitive behaviors
23. Phobia C. Worry and apprehension is excessive and long
standing
24. Obsessive–compulsive D. Stress reaction to a severe disturbing or
disorder (OCD) upsetting experience
25. Posttraumatic stress E. Recurrent extremely intense attacks of fright and
disorder (PTSD) dread

Discussion Question
26. Discuss the special safety issues that need to be addressed when conducting
polysomnography in patients with psychiatric disorders.

ANSWERS

1. A, Major depressive disorder (MDD).


See Fundamentals of Sleep Technology, 2nd edition, Chapter 22, page 228.

2. B, During adolescence or early adulthood.


See Fundamentals of Sleep Technology, 2nd edition, Chapter 22, page 228.

3. D, REM sleep.
See Fundamentals of Sleep Technology, 2nd edition, Chapter 22, page 229.

4. D, Early morning awakening.


See Fundamentals of Sleep Technology, 2nd edition, Chapter 22, page 229.

5. C, Increased REM density.


See Fundamentals of Sleep Technology, 2nd edition, Chapter 22, page 229.

6. B, Decreased REM sleep.


See Fundamentals of Sleep Technology, 2nd edition, Chapter 22, page 229.

7. A, Hypomania.
See Fundamentals of Sleep Technology, 2nd edition, Chapter 22, page 229.

8. B, Periods of sleeplessness.
See Fundamentals of Sleep Technology, 2nd edition, Chapter 22, page 230.

9. C, Hypersomnia.
See Fundamentals of Sleep Technology, 2nd edition, Chapter 22, page 230.

10. D, Bright light therapy.


See Fundamentals of Sleep Technology, 2nd edition, Chapter 22, page 230.

11. A, Sleep onset and sleep maintenance insomnia.


See Fundamentals of Sleep Technology, 2nd edition, Chapter 22, page 232.

12. D, Insomnia.
See Fundamentals of Sleep Technology, 2nd edition, Chapter 22, page 232.

13. C, Delayed bedtimes.


See Fundamentals of Sleep Technology, 2nd edition, Chapter 22, page 232.

14. D, Anxiety dreams.


See Fundamentals of Sleep Technology, 2nd edition, Chapter 22, page 233.

15. D, Delusional beliefs and inappropriate behavior.


See Fundamentals of Sleep Technology, 2nd edition, Chapter 22, page 233.

16. C, A lack of REM rebound.


See Fundamentals of Sleep Technology, 2nd edition, Chapter 22, page 234.

17. C, 60 minutes.
See Fundamentals of Sleep Technology, 2nd edition, Chapter 22, page 234.

18. A, Decreased REM sleep in the first half of the sleep period.
See Fundamentals of Sleep Technology, 2nd edition, Chapter 22, page 234.

19. A, Refrains from drinking on the night of the test.


See Fundamentals of Sleep Technology, 2nd edition, Chapter 22, page 235.

20. D, Enuresis and nightmares.


See Fundamentals of Sleep Technology, 2nd edition, Chapter 22, page 235.

21. C, Worry and apprehension is excessive and long standing.


See Fundamentals of Sleep Technology, 2nd edition, Chapter 22, pages 231–232.

22. E, Recurrent extremely intense attacks of fright and dread.


See Fundamentals of Sleep Technology, 2nd edition, Chapter 22, pages 231–232.

23. A, Intense, irrational fear of an object or situation.


See Fundamentals of Sleep Technology, 2nd edition, Chapter 22, pages 231–232.

24. B, Persistent, upsetting, and unwanted thoughts that may motivate repetitive
behaviors.
See Fundamentals of Sleep Technology, 2nd edition, Chapter 22, pages 231–232.

25. D, Stress reaction to a severe disturbing or upsetting experience.


See Fundamentals of Sleep Technology, 2nd edition, Chapter 22, pages 231–232.

26. To maintain patient and staff safety, assure a second technologist is in the
sleep center. Whenever possible, a caregiver should accompany the patient
for testing. The staff in the sleep center should have an understanding of the
particular psychiatric disorder and the ability to work calmly with patients.
Psychiatric patients may become aggressive; to avoid aggressive behavior, the
technologist should maintain a reasonable distance from the patient, avoid
direct eye contact and remain nonconfrontational. There should be ready
access to security if needed.
See Fundamentals of Sleep Technology, 2nd edition, Chapter 23, page 251.
SECTION III
Patient Care and Assessment
CHAPTER
18
Patient and Employee Safety
DEBBIE AKERS

NOTE: This chapter corresponds to Chapter 23 in Fundamentals of Sleep


Technology, 2nd edition.

1. The purpose of the material safety data sheets is to:


A. Provide information to patients about proper use of products
B. Absolve the manufacturer of responsibility for products
C. Provide workers with procedures for handling and storage of products
D. Provide methods for testing and rating the safety of potentially
dangerous products

2. Universal precautions refer to infection control policies that assume which of


the following?
A. Any direct contact with a patient, particularly with body fluids, has the
potential for transmitting disease.
B. All hospitalized patients have highly communicable infections.
C. Contact with the same air being breathed by a patient has the potential
for disease transmission.
D. All patients are HIV (human immunodeficiency virus) positive.

3. Hand washing must occur:


A. Only after contact with body fluids and secretions
B. After touching fluids, secretions, and contaminated items
C. Only when gloves are not worn
D. After entering the patient room, even if no contact with the patient
occurred

4. Semicritical equipment requires high-level disinfection in a separate cleaning


area with wet pasteurization or a chemical disinfectant. Which of the
following items is considered a semicritical item?
A. CPAP masks
B. Electroencephalography (EEG) electrodes
C. Body position sensors
D. Respiratory inductance plethysmography (RIP) belts

5. Any nondisposable item that comes into contact with a patient should be:
A. Discarded
B. Pressure-sprayed
C. Pasteurized
D. Decontaminated

6. Proper lifting techniques include:


A. Bending at the back
B. Stretching overhead but not to the side
C. Twisting at the waist
D. Bending at the knees and hips

7. Evacuation policy and procedures for employees should be part of a sleep


center's:
A. Material safety data sheets
B. Daily operating procedures
C. Emergency action plan
D. HIPAA compliance plan

8. The first step in responding to a fire in a hospital or other medical setting is:
A. Rescue
B. Retreat
C. Release
D. Run

9. Sleep center personnel may be first responders to medical emergencies and


must, therefore, be trained and certified in:
A. Basic first aid
B. Cardiopulmonary resuscitation
C. Performing field tracheostomy
D. Use of the “jaws of life”

10. Preparation for patients suspected of having seizure disorders includes:


A. Use of bedrail and headboard padding
B. Keeping a tongue depressor nearby to place in the patient's mouth when
a seizure occurs
C. Preadministration of a valium suppository prior to the study
D. Using restraints to prevent the patient from rolling out of the bed

VIOLENT PATIENT: THE CASE OF MR. B


Mr. B arrives at the sleep center approximately 1 hour late smelling of alcohol.
His gait is unsteady, and his speech is slurred. The sleep technologist assigned to
care for him is already preparing another patient, and Mr. B is asked to wait.
When the technologist arrives at his room half an hour later to put on the
electrodes, Mr. B is clearly agitated and begins yelling at the technologist, telling
him that the delay is inexcusable.
Unfortunately, the technologist responds to Mr. B inappropriately. These
incorrect behaviors by the technologist are listed below. Describe the behaviors
that the technologist should have used instead.

DISCUSSION QUESTIONS

15. Medical emergencies are infrequent in sleep centers, and many laboratories
restrict testing to medically stable patients with few comorbidities. Is it still
important for these centers to have policies and procedures for medical
emergencies? Please explain your answer.

16. Electrical standards help minimize potential hazards by specifying safety


aspects in the design and use of electrical equipment and systems. Discuss
how the sleep center protects the patient from an electrical accident.

ANSWERS

1. C, Provide workers with procedures for handling and storage of products.


See Fundamentals of Sleep Technology, 2nd edition, Chapter 23, page 239.

2. A, Any direct contact with a patient, particularly with body fluids, has the
potential for transmitting disease.
See Fundamentals of Sleep Technology, 2nd edition, Chapter 23, page 240.

3. B, After touching fluids, secretions, and contaminated items.


See Fundamentals of Sleep Technology, 2nd edition, Chapter 23, page 240.

4. A, CPAP masks.
See Fundamentals of Sleep Technology, 2nd edition, Chapter 23, pages 241–242.

5. D, Decontaminated.
See Fundamentals of Sleep Technology, 2nd edition, Chapter 23, page 242.

6. D, Bending at the knees and hips.


See Fundamentals of Sleep Technology, 2nd edition, Chapter 23, pages 242–243.

7. C, Emergency action plan.


See Fundamentals of Sleep Technology, 2nd edition, Chapter 23, pages 243–244.

8. A, Rescue.
See Fundamentals of Sleep Technology, 2nd edition, Chapter 23, page 245.

9. B, Cardiopulmonary resuscitation.
See Fundamentals of Sleep Technology, 2nd edition, Chapter 23, page 248.

10. A, Use of bedrail and headboard padding.


See Fundamentals of Sleep Technology, 2nd edition, Chapter 23, page 250.
12. Sleep center personnel are first responders for medical emergencies, and
policies and procedures are essential guidelines for the technologist. Since
medical emergencies are not common, having immediate access to
information assists the technologist when a medical emergency occurs.

13. First it is essential to assure safety of all equipment used in the sleep center.
Make sure that all equipment has a properly grounded power cord and all
outlets are grounded. All equipment in the patient room should have properly
insulated cords and effective electrical grounding. Equipment grounding for
safety is not the same as the patient–ground connection. The patient–ground
connection is used to help prevent line-frequency interference (50 or 60 Hz)
by providing a conductive pathway from the patient to ground through the
recording system. The technologist should assure that electrode leads and
head boxes do not contact any electrical equipment or any conductive or wet
surfaces. When dry, the patient's skin has a fairly high resistance to electric
current; but when it is moist, there is a significant drop in resistance. Pure
water is a poor conductor, but small amounts of impurities, such as salt and
acid (both of which are contained in perspiration), make water a ready
conductor. These precautions help to assure electrical safety in the sleep
center.
CHAPTER
19
Medications and Their Effects on Sleep
and Sleep Disorders
DEBBIE AKERS

NOTE: This chapter corresponds to Chapter 24 in Fundamentals of Sleep


Technology, 2nd edition.

1. The most common prescribed sleeping pills are nonbenzodiazepine


hypnotics (“z-drugs” such as zolpidem, zopiclone, and zaleplon). The effects
of these medications on sleep pattern include:
A. Decreased sleep onset latency and reduced N1 sleep
B. Increased REM sleep and decreased REM latency
C. Increased sleep latency and increased sleep-related breathing events
D. Decreased total sleep time and increased slow-wave sleep time

2. In comparison to most benzodiazepines, the nonbenzodiazepine hypnotics


(“z-drugs” such as zolpidem, zopiclone, and zaleplon) have:
A. Longer latency to onset of action
B. More suppression of stage N3 sleep
C. More risk of increased sleep-related breathing events
D. A shorter half-life

3. Tricyclic antidepressants cause profound decreases in:


A. N2 sleep and sleep spindles
B. N3 sleep
C. REM sleep
D. Daytime sleepiness

4. First-generation antihistamines often give rise to as an unwanted side effect.


A. Seizures
B. Increased sleep latency
C. Sedation
D. Sleep-onset REM episodes

5. Modafinil belongs to which class of drugs?


A. Stimulants
B. Sedative hypnotics
C. Anticonvulsants
D. Opioids

CONSIDER THE FOLLOWING HISTORY OF A


SLEEP CENTER PATIENT
A 34-year-old man comes to the sleep center with a complaint of daytime
sleepiness. He takes clonazepam (Klonopin) as needed during the day to control
panic attacks. He has difficulty falling asleep, and his primary care physician has
prescribed quazepam (Doral) at bedtime. Other medical conditions include high
blood pressure, for which he takes lisinopril (Zestril); allergies to pollen and tree
mold treated with diphenhydramine (Benadryl); and high cholesterol treated
with simvastatin (Zocor). His primary care doctor has sent him to the sleep
center for possible treatment with modafinil (Provigil). After reviewing the
patient's medical history and medication list, you decide to contact the primary
care doctor to discuss replacement of some the medications because they may be
causing daytime sleepiness.
Which of the medications listed above are associated with daytime sedation?
What alternatives might be considered for the medications causing sedation?

DISCUSSION QUESTIONS
12. Current medications for sleep disorders typically affect central nervous
system sleep–wake neurotransmitters and hormones, such as dopamine,
norepinephrine, serotonin, gamma-aminobutyric acid, or melatonin. The
neurotransmitter hypocretin (orexin) has an important role in promoting
wakefulness. Discuss how agonists and antagonists of hypocretin might be
used to treat sleep disorders.

13. A 52-year-old woman currently in pain management on long-acting opioid


therapy was diagnosed with moderate obstructive sleep apnea. Discuss how
chronic use of opiates impacts sleep.

14. A 21-year-old man who frequently flies from the United States to Japan for
business asks whether melatonin is a safe and effective treatment for his jet
lag symptoms. Discuss the efficacy of melatonin treatment for insomnia as
well as recommendations for dose and timing.

ANSWERS

1. A, Decreased sleep onset latency and reduced N1 sleep.


See Fundamentals of Sleep Technology, 2nd edition, Chapter 24, page 256.

2. D, A shorter half-life.
See Fundamentals of Sleep Technology, 2nd edition, Chapter 24, page 254.

3. C, REM sleep.
See Fundamentals of Sleep Technology, 2nd edition, Chapter 24, pages 254–256.

4. C, Sedation.
See Fundamentals of Sleep Technology, 2nd edition, Chapter 24, page 260.

5. A, Stimulants.
See Fundamentals of Sleep Technology, 2nd edition, Chapter 24, page 260.
7. “In just the last 10 years, much has been learned about the ways in which
orexins promote arousal. In general, it may be best to think of this as a
system for sustaining wakefulness as people and mice with narcolepsy have
approximately normal amounts of wakefulness, but have great difficulty
maintaining long periods of wakefulness. Orexins may also stabilize sleep as
people with narcolepsy often have fragmented sleep, and orexins certainly
regulate REM sleep as discussed below. In addition, orexins promote arousal
responses to homeostatic challenges and drive motivated behaviors such as
seeking food. Orexins directly excite neurons of the mesolimbic reward
pathways, and orexin antagonists can reduce the motivation to seek drugs of
abuse. The orexin neurons are also activated by humoral indicators of hunger
such as low glucose or high levels of ghrelin, and while normal mice have a
clear increase in arousal when deprived of food, mice lacking the orexin
neurons show little response. Thus, one can view the orexin system as
helping sustain wakefulness across much of the day, and increasing arousal
in motivating conditions.” (España RA, Scammell TE. Sleep neurobiology
from a clinical perspective. Sleep. 2011;34(7):845–858.)

8. “In summary, we have found that patients being treated with chronic opioids
who are seen for evaluation of possible obstructive sleep apnea syndrome,
the respiratory abnormalities are complex, difficult to define using standard
criteria, and ataxic breathing or Biot's respiration is characteristic. These
cases are invariably complicated by their comorbidities, numerous
medications, and pain requiring chronic opioid therapy. In our experience,
these patients are becoming increasingly common as the prescription rate for
chronic opioids increases.” (Farney RJ, Walker JM, Boyle KM, et al.
Adaptive servoventilation (ASV) in patients with sleep disordered breathing
associated with chronic opioid medications for non-malignant pain. J Clin
Sleep Med. 2008;4(4):311–319.)

9. “In summary, our results support previous claims that exogenous melatonin
has hypnotic properties only during sleep episodes that are out of phase with
the traditional, nocturnal sleep episode and when endogenous melatonin is
not present. This supports the claim that melatonin itself exerts sleep-
facilitating effects by suppressing the circadian drive for wakefulness that
normally occurs during the habitual waking day and is maximal just prior to
the onset of melatonin secretion.” (Wyatt JK, Dijk DJ, De Cecco AR, et al.
Sleep-facilitating effect of exogenous melatonin in healthy young men and
women is circadian phase dependent. Sleep. 2006;29(5):609–618.)
SECTION IV
Polysomnography
CHAPTER
20
Digital Polysomnography
CHRISTINA L. TROXELL

NOTE: This chapter corresponds to Chapter 26 in Fundamentals of Sleep


Technology, 2nd edition.

1. Which one of the following is a bioelectrical potential signal?


A. ECG
B. Snore
C. Position
D. EtCO2

2. Several metals have high conductivity, but gold is considered best for long
sleep study recordings because it is:
A. More malleable
B. Impervious to cold and heat
C. The traditional choice
D. Resistant to corrosive effects

3. Ohm law tells us that when voltage is constant, increasing electrode


resistance:
A. Reduces current
B. Increases radiant heat
C. Blocks the flow of ions in neurons
D. Is futile

4. Differential amplifiers reduce noise in EEG signals using:


A. Floating reference electrodes
B. Linked mastoid electrodes
C. Smoothing functions
D. Common mode rejection
5. By convention, EEG voltages are represented as:
A. Large triangles with multiple inputs
B. A series of sine waves of varying frequency
C. Negative voltage up
D. Nanovolts (10−9 V)

6. When the two inputs to the differential amplifier detect equal voltages, the
output is:
A. Polarized
B. Zero
C. Rectified
D. Two times the voltage of the positive input

7. When the impedance of two inputs is not the same (impedance mismatch),
the effect is to reduce the:
A. Voltage of the positive input
B. Voltage of the negative input
C. Effectiveness of the common mode rejection
D. Resistance to the flow of low-frequency signals

8. The filter typically used to remove power line electrical noise is a:


A. Line filter
B. Brita filter
C. Notch (60 Hz) filter
D. High-frequency filter

9. Accurate reproduction of a waveform requires analog-to-digital sampling at


a rate at least twice that of the highest frequency of interest. This is known
as the:
A. Nyquist theorem
B. Lord Adrian rule
C. Rule of doubles
D. Frequency conversion theorem

10. The American Academy of Sleep Medicine (AASM) recommends a


minimum 12 bits for analog-to-digital conversion. This affects the:
A. Number of samples per second
B. Voltage or amplitude resolution
C. Frequency aliasing
D. Screen resolution

Matching

Match the proper phase or term with the appropriate definition.

11. Conductivity A. Ratio of input voltage and output amplitude


12. Polarity B. Opposition of alternating current flow by resistance and
capacitance
13. Impedence C. Used to attenuate a limited frequency band, such as
power line frequencies
14. Frequency D. Used to attenuate signals below the cutoff frequency
15. Amplitude E. Ability to transmit voltage
16. Sensitivity F. Duration of signal responses set by high- and low-
frequency filters
17. Notch filter G. Used to attenuate signals above the cutoff frequency
18. High-frequency H. Represents the electrical voltage of wave
filter
19. Low-frequency I. Up or down referencing to the electric zero
filter
20. Time constant J. Width of wave expressed as cycles per second

DISCUSSION QUESTIONS

21. The American Academy of Sleep Medicine (AASM) Digital Task Force for
the Manual for the Scoring of Sleep and Associated Events stated that
available digital filtering technology should not be used. They
recommended that digital approximations of analog filtering should be used
instead. Do you think this was appropriate?

22. What purpose does the ground electrode serve, and what effect does a bad
ground electrode have on the recording?

23. Explain the importance of sampling rate to assure adequate analog-to-digital
conversion for the signals recorded on the polysomnogram.

ANSWERS

1. A, ECG.
See Fundamentals of Sleep Technology, 2nd edition, Chapter 26, page 276.

2. D, Resistant to corrosive effects.


See Fundamentals of Sleep Technology, 2nd edition, Chapter 26, page 276.

3. A, Reduces current.
See Fundamentals of Sleep Technology, 2nd edition, Chapter 26, page 277.

4. D, Common mode rejection.


See Fundamentals of Sleep Technology, 2nd edition, Chapter 26, page 277.

5. C, Negative voltage up.


See Fundamentals of Sleep Technology, 2nd edition, Chapter 26, page 277.

6. B, Zero.
See Fundamentals of Sleep Technology, 2nd edition, Chapter 26, page 278.

7. C, Effectiveness of the common mode rejection.


See Fundamentals of Sleep Technology, 2nd edition, Chapter 26, page 279.

8. C, Notch (60 Hz) filter.


See Fundamentals of Sleep Technology, 2nd edition, Chapter 26, page 281.

9. A, Nyquist theorem.
See Fundamentals of Sleep Technology, 2nd edition, Chapter 26, page 282.

10. B, Voltage or amplitude resolution.


See Fundamentals of Sleep Technology, 2nd edition, Chapter 26, page 282.

11. E, Ability to transmit voltage.


See Fundamentals of Sleep Technology, 2nd edition, Chapter 26, page 276.

12. I, Up or down referencing to the electric zero.


See Fundamentals of Sleep Technology, 2nd edition, Chapter 26, pages 277–278.
13. B, Opposition of alternating current flow by resistance and capacitance.
See Fundamentals of Sleep Technology, 2nd edition, Chapter 26, page 275.

14. J, Width of wave expressed as cycles per second.


See Fundamentals of Sleep Technology, 2nd edition, Chapter 26, page 279.

15. H, Represents the electrical voltage of wave.


See Fundamentals of Sleep Technology, 2nd edition, Chapter 26, page 279.

16. A, Ratio of input voltage and output amplitude.


See Fundamentals of Sleep Technology, 2nd edition, Chapter 26, page 279.

17. C, Used to attenuate a limited frequency band, such as power line frequencies.
See Fundamentals of Sleep Technology, 2nd edition, Chapter 26, page 281.

18. G, Used to attenuate signals above the cutoff frequency.


See Fundamentals of Sleep Technology, 2nd edition, Chapter 26, page 280.

19. D, Used to attenuate signals below the cutoff frequency.


See Fundamentals of Sleep Technology, 2nd edition, Chapter 26, page 280.

20. F, Duration of signal responses set by high- and low-frequency filters.


See Fundamentals of Sleep Technology, 2nd edition, Chapter 26, page 280.

21. “Present-day digital filters tend to overprocess the input signal. As a result, the
recorded data may appear ‘clean,’ but lack essential detail. The sharp cut-off
design of some digital filters essentially removes all fast and slow frequency
artifacts, regardless of input signal quality. Consequently, the attending
technologist may not be aware of input signal degradation (i.e., electrodes
may be detached from the patient and continue to generate a narrow
bandwidth of nonphysiological frequencies resembling the EEG). Digital
filters can produce practically any kind of frequency response. Digital filters
can simulate conventional analogue circuit filter frequency response curves.
Recommendations for the filter design can help ensure the reproducibility of
data generated by different brands of equipment and preserve continuity with
the previous gold standard, as applied to paper-based recordings.” (Penzel T,
Hirshkowitz M, Harsh J, et al. Digital analysis and technical specifications. J
Clin Sleep Med. 2007:3(2);109–120.) See Fundamentals of Sleep
Technology, 2nd edition, Chapter 26, pages 280–281.

22. The ground electrode serves to equalize electrical potentials generated by the
patient and the input circuits of the amplifier to provide an electrical zero
point. A bad ground electrode eliminates electrical zero and may cause a
wandering baseline or, more likely, amplifier saturation.
See Fundamentals of Sleep Technology, 2nd edition, Chapter 26, pages 277, 279.

23. Sampling rate determines how many times an analog signal is captured or
“sampled” per second in order to reproduce the signal digitally. The sampling
rate necessary to accurately reproduce a signal is based on the frequency of
the analog signal; a faster analog signal requires a higher sampling rate to
adequately reproduce the signal. The Nyquist theorem states that adequate
analog-to-digital conversion requires a sampling rate of at least two times the
highest frequency being measured in order to reproduce a signal accurately.
Most polysomnographs today sample EEG at 500 Hz, although 200 Hz is
sufficient to reproduce the EEG signal digitally. Slower signals such as
respiratory effort and airflow can adequately be reproduced at a sampling rate
of about 100 Hz, while a slow DC signal such as SpO2 can be adequately
reproduced at a sampling rate of 25 Hz.
See Fundamentals of Sleep Technology, 2nd edition, Chapter 26, pages 282, 284.
CHAPTER
21
Recording the Biopotentials of Sleep
CHRISTINA L. TROXELL

NOTE: This chapter corresponds to Chapter 27 in Fundamentals of Sleep


Technology, 2nd edition.

1. Sweat artifact in the EEG is:


A. Slow, rolling activity
B. Often mistaken for interictal spike activity
C. Similar to line noise (60 Hz)
D. Intermittent amplifier shorting due to salt in the sweat

2. Recording the EOG is possible because the ______________ has a negative


charge and the ______________ has a positive charge.
A. Inion, nasion
B. Retina, cornea
C. Iris, vitreous humour
D. Cathode, anode

3. The physiologic calibrations are a series of instructions that are designed to


determine that the:
A. Patient understands English
B. Patient will cooperate with the technologist
C. Sensors are picking up signals correctly
D. Optical isolation circuits are working properly

4. The presence of alpha waves during the physiologic calibrations typically


indicates:
A. That the patient has fallen asleep
B. That the patient has closed his or her eyes
C. A seizure disorder
D. Poor electrode contact with the skin
5. Asking the patient to count during the physiologic calibrations:
A. Usually puts the patient to sleep
B. Tests the chin EMG recording
C. Tests the snoring channel
D. Allows the patient to time the breath-holding maneuver

6. Asking the patient to breathe through the mouth will cause a signal to be
recorded in the:
A. Effort channel
B. Nasal pressure channel
C. Snoring channel
D. Thermal flow channel

7. Asking the patient to hold the breath for 10 seconds during the physiologic
calibrations:
A. Causes severe oxygen desaturation
B. Simulates an apnea
C. Has no effect on the flow signals
D. Causes flattening of the chin EMG waveform

8. The complete QRS complex is a waveform that is:


A. Down and then up
B. Up and then down
C. Up, down, up
D. Down, up, down

9. Prolonged muscle artifact in the EEG:


A. Should be explained in the technologist notes
B. Indicates a ground electrode problem
C. Is always associated with a seizure
D. Means the technologist should replace the electrodes

10. Accessory equipment such as a CPAP machine or radio placed near the
headbox may cause:
A. 60-Hz artifact
B. Electrode pops
C. Spurious electrode impedance measures
D. Gap junctions with sparking
Matching

Calibration: Match the elements of the physiologic calibration with the proper
figure.
A. Eyes closed, eyes open
B. Move left leg, move right leg
C. Breath hold 10 seconds
D. Make snoring sounds
E. Left, right, left, right, blink 5 times
11.

Figure 21-1

12.
Figure 21-2

13.

Figure 21-3

14.
Figure 21-4

15.

Figure 21-5
Artifacts: Match the event with the appropriate figure:
A. Ballistocardiographic artifact
B. Blinking
C. Sweat artifact
D. Electrode disconnection
E. Muscle artifact
16.

Figure 21-6

17.
Figure 21-7

18.

Figure 21-8

19.
Figure 21-9

20.

Figure 21-10
DISCUSSION QUESTION

21. The physiologic calibrations test many but not all of the channels recorded
during a polysomnogram. Discuss the value of pre- and posttest calibrations
for the technologist in the role of obtaining an accurate recording of the
patient during sleep. How are the channels that are not tested during the
physiologic calibrations evaluated?

ANSWERS

1. A, Slow, rolling activity.


See Fundamentals of Sleep Technology, 2nd edition, Chapter 27, page 287.

2. B, Retina, cornea.
See Fundamentals of Sleep Technology, 2nd edition, Chapter 27, page 287.

3. C, Sensors are picking up signals correctly.


See Fundamentals of Sleep Technology, 2nd edition, Chapter 27, page 289.

4. B, The patient has closed his or her eyes.


See Fundamentals of Sleep Technology, 2nd edition, Chapter 27, page 289.

5. C, Tests the snoring channel.


See Fundamentals of Sleep Technology, 2nd edition, Chapter 27, page 289.

6. D, Thermal flow channel.


See Fundamentals of Sleep Technology, 2nd edition, Chapter 27, page 289.

7. B, Simulates an apnea.
See Fundamentals of Sleep Technology, 2nd edition, Chapter 27, page 293.

8. D, Down, up, down.


See Fundamentals of Sleep Technology, 2nd edition, Chapter 27, page 293

9. A, Should be explained in the technologist notes.


See Fundamentals of Sleep Technology, 2nd edition, Chapter 27, page 293.
10. A, 60-Hz artifact.
See Fundamentals of Sleep Technology, 2nd edition, Chapter 27, page 294.

11. E, Left, right, left, right, blink 5 times.

12. A, Eyes closed, eyes open.

13. B, Move left leg, move right leg.

14. C, Breath hold 10 seconds.

15. D, Make snoring sounds.


See Fundamentals of Sleep Technology, 2nd edition, Chapter 27, pages 289–293.

16. C, Sweat artifact.

17. A, Ballistocardiographic artifact.

18. D, Electrode disconnection.

19. E, Muscle artifact.

20. B, Blinking.
See Fundamentals of Sleep Technology, 2nd edition, Chapter 27, pages 287–288.

21. “The physiologic calibrations (sometimes called biocalibrations or patient


calibrations) are performed before and after a sleep study. The physiologic
calibrations are a series of instructions that are designed to determine that the
sensors are correctly picking up signals from a patient, that the polarity of the
signals is correct, that the strength of the signal is sufficient for recording, and
that there is no unwanted interference.” The ECG is evaluated based on wave
shape and frequency, and the oxygen saturation is reviewed to ensure that it is
within established guidelines.
See Fundamentals of Sleep Technology, 2nd edition, Chapter 27, pages 290–293.
CHAPTER
22
Patient Interviewing and Assessment
JOANNE HEBDING

NOTE: This chapter corresponds to Chapter 28 in Fundamentals of Sleep


Technology, 2nd edition.

1. The history and physical portion of the patient's chart includes:


A. A list of current medications
B. Insurance information
C. Method of payment
D. Preapproval by the insurer

2. The facility orientation typically includes:


A. Showing the patient the control room
B. Introducing the patient to other patients being tested that night
C. Showing the patient the bathroom
D. A discussion of treatment alternatives

3. An appropriate strategy for patients with mental disorders and special


emotional needs is to:
A. Be firm and brusque and do not spend extra time with the patient
B. Challenge their viewpoints with logical arguments
C. Be calm and do not argue
D. Mirror their behavior—if they get angry, return their anger

4. Patients with special physical needs:


A. Cannot be studied in the sleep center
B. Usually require more than one technologist for the sleep study
C. May require modified placement of electrodes
D. Typically have difficulties with language and unwillingness to learn

5. An elderly patient who functions normally during the day but becomes
confused and agitated at night may be suffering from:
A. Twilight zone
B. Paradoxical insomnia
C. Wernicke encephalopathy
D. Sundowning

6. Prior to the start of the sleep study, the physician calls the technologist and
asks that his patient have temporal EEG leads and arm and leg EMG
monitors added to the physician orders. The technologist should:
A. Document the order with date and time and ask the physician to sign it
the next day
B. Refuse to make the change because the orders make no sense
C. Cancel the study and ask the patient to return on another day when the
orders have been changed
D. Follow the order but lodge a complaint with the center supervisor

7. For patients approved for a split night study with possible PAP titration, the
technologist should:
A. Not discuss PAP therapy as the patient may not qualify for it
B. Provide mask fitting if and when the patient qualifies for a split night
study
C. Educate the patient and fit a mask prior to the start of the study
D. Warn the patient that he or she may wake with a mask on

8. At the end of the study, the technologist should provide the patient with:
A. A reminder that the physician will make the final diagnosis
B. A diagnosis to reduce the patient's anxiety or to confirm the patient's best
guess
C. A treatment plan but not a diagnosis
D. Both a diagnosis and a treatment plan

CLINICAL VIGNETTES

9. A 37-year-old man suspected of sleep apnea is referred to your center for a


study. He is approved for a split night. His current medication is fluoxetine
(Prozac). The technologist shows him the bedroom and leaves to greet
another patient. When she returns, the patient is naked from the waist down.
The patient invites the technologist to “have a good time.” Which of the
following responses are appropriate, and which are inappropriate?

10. A 77-year-old woman comes to the sleep center for evaluation of daytime
sleepiness. She typically takes clonazepam (Klonopin) at night to help her
fall asleep. She has brought some with her for the night of the test. Although
there is no information regarding her physical status in the chart, she tells
you that she is often unsteady during the night and may require assistance to
get to the bathroom. Which of the following responses are appropriate and
which are inappropriate?


DISCUSSION QUESTION

11. Mrs. H arrives unaccompanied to the sleep center in a calm mood. During
the hookup at 9:00 PM, she takes flurazepam. She wakes up in the middle of
the study around 1:45 AM and tosses and turns for about 20 minutes. She
calls out to the technologist and says she wants to go home, and that she
knows she will not be able to go back to sleep. Discuss appropriate actions
for the sleep technologist to take.

ANSWERS

1. A, A list of current medications.


See Fundamentals of Sleep Technology, 2nd edition, Chapter 28, page 301.

2. C, Showing the patient the bathroom.


See Fundamentals of Sleep Technology, 2nd edition, Chapter 28, page 302.

3. C, Be calm and do not argue.


See Fundamentals of Sleep Technology, 2nd edition, Chapter 28, page 303.

4. C, May require modified placement of electrodes.


See Fundamentals of Sleep Technology, 2nd edition, Chapter 28, page 303.

5. D, Sundowning.
See Fundamentals of Sleep Technology, 2nd edition, Chapter 28, page 304.

6. A, Document the order with date and time and ask the physician to sign it the
next day.
See Fundamentals of Sleep Technology, 2nd edition, Chapter 28, page 304.

7. C, Educate the patient and fit a mask prior to the start of the study.
See Fundamentals of Sleep Technology, 2nd edition, Chapter 28, page 305.

8. A, A reminder that the physician will make the final diagnosis.


See Fundamentals of Sleep Technology, 2nd edition, Chapter 28, pages 308–309.

9.
See Fundamentals of Sleep Technology, 2nd edition, Chapter 28, page 304.

10.

See Fundamentals of Sleep Technology, 2nd edition, Chapter 28, page 303.

11. The technologist would first assess how alert the patient is and determine how
far she lives from the sleep center. One option may be to take a cab home, and
another is to call her spouse or other family member and see if they can come
pick her up. The technologist can talk with the patient and try to convince her
to try to go back to sleep until 5:00 in the morning, telling her that most times
people do fall back to sleep. When the patient is discharged, the technologist
should ensure that the patient is safe. The technologist should document the
situation completely with pertinent details in the patient's chart.
See Fundamentals of Sleep Technology, 2nd edition, Chapter 28, page 306.
CHAPTER
23
Patient Preparation
SHALANDA L. MITCHELL

NOTE: This chapter corresponds to Chapter 29 in Fundamentals of Sleep


Technology, 2nd edition.

1. Describe what a technologist should do before a patient arrives at the sleep


center.

2. Name the four main landmarks used in the International 10/20 system for
electroencephalograph electrode placement.

3. The CZ reference can also be used as the ground electrode. True or false?

4. Which of the following units does a sleep technologist use when measuring
the scalp for electrode placement?
A. Centimeters
B. Inches
C. Millijoules
D. Millimeters

5. Name the six American Academy of Sleep Medicine (AASM)–


recommended electroencephalographic (EEG) electrodes used for
polysomnography (PSG).

6. Which of the following is the preferred sensor to monitor respiratory effort?


A. Respiratory inductance plethysmography
B. Electromyography
C. Piezoelectric crystal sensors
D. Differential respiratory sensors

7. What types of sensors are used to monitor airflow?


8. Scalp and face electrodes should not exceed an impedance of:


A. 5 cm
B. 10 mV
C. 10 kΩ
D. 5 kΩ

9. What is the minimum recording time for a polysomnogram before


proceeding with a multiple sleep latency test (MSLT)?
A. 8 hours
B. 6 hours
C. 5.5 hours
D. 3 hours

10. Why is it important to prepare the skin before applying electrodes?


11. When applying electrodes using the 10/20 International system, what are
three key elements that are vital to collecting quality data?

12. Sensitivity is synonymous with:


A. Gain
B. Amplitude
C. Hertz
D. Frequency

13. A nasal air pressure transducer is used to identify apneas. True or false?

14. Discuss the significance of biocalibrations and describe when they should be
performed.

15. On which muscle should the leg electromyogram (EMG) electrodes be
placed?
A. Tibia
B. Anterior tibialis
C. Mastoid
D. Preauricular

16. If the distance between A1 and A2 is 38 cm, what should the distance
between A1 and C3 be?
A. 7.2 cm
B. 3.8 cm
C. 8.0 cm
D. 6.0 cm

17. Describe the precautions needed when using collodion for electrode
attachment.

18. Name the locations identified by the letters A through K in Figure 23-1.

Figure 23-1 10-20 System of Electrode Placement

A. ____________________________
B. ____________________________
C. ____________________________
D. ____________________________
E. ____________________________
F. ____________________________
G. ____________________________
H. ____________________________
I. ____________________________
J. ____________________________
K. ____________________________

ANSWERS

1. The technologist should review the physician's orders, set up and calibrate the
recording equipment, prepare the necessary supplies, and assess and respond
to any special needs the patient may have.
See Fundamentals of Sleep Technology, 2nd edition, Chapter 29, page 311.

2. Nasion, inion, and left and right preauricular areas See Fundamentals of
Sleep Technology, 2nd edition, Chapter 29, pages 314–315.

3. False. CZ is used only as a reference point. A ground electrode prevents line-


frequency interference by serving as a conducting pathway from the patient
to ground via the recording system.
See Fundamentals of Sleep Technology, 2nd edition, Chapter 29, page 315.

4. A, Centimeters.
See Fundamentals of Sleep Technology, 2nd edition, Chapter 29, page 315.

5. Two frontal leads (F3 and F4), two central leads (C3 and C4), two occipital
leads (O1 and O2).
See Fundamentals of Sleep Technology, 2nd edition, Chapter 29, page 314.

6. A, Respiratory inductance plethysmography.


See Fundamentals of Sleep Technology, 2nd edition, Chapter 29, page 319.

7. Thermal sensors and nasal cannula pressure transducers.


See Fundamentals of Sleep Technology, 2nd edition, Chapter 29, pages 319–320.

8. D, 5 kΩ.
See Fundamentals of Sleep Technology, 2nd edition, Chapter 29, page 321.

9. B, 6 hours.
See Fundamentals of Sleep Technology, 2nd edition, Chapter 29, page 322.

10. Preparing the skin before applying electrodes ensures good signal quality and
reduces impedance.
See Fundamentals of Sleep Technology, 2nd edition, Chapter 29, page 316.

11. Electrodes must be placed in the correct locations, electrode sites must be
prepared properly, and electrodes must be securely attached.
See Fundamentals of Sleep Technology, 2nd edition, Chapter 29, page 315.

12. A, Gain.
See Fundamentals of Sleep Technology, 2nd edition, Chapter 29, page 321.

13. False. Thermal sensors are used to identify apneas. Nasal pressure transducers
are used to identify hypopneas.
See Fundamentals of Sleep Technology, 2nd edition, Chapter 29, pages 319–320.

14. Biocalibrations are a series of steps performed to verify correct input


derivations and signal quality, and accurate signal responses. Biocalibrations
should be performed at the beginning and end of each study.
See Fundamentals of Sleep Technology, 2nd edition, Chapter 29, pages 321–322.

15. B, Anterior tibialis.


See Fundamentals of Sleep Technology, 2nd edition, Chapter 29, page 318.

16. A, 7.2 cm.


See Fundamentals of Sleep Technology, 2nd edition, Chapter 29, pages 314–316.

17. Work in a well-ventilated area when using collodion. Use an air filtration or
exhaust system in small spaces to reduce fumes. Store collodion in a fireproof
safe.
See Fundamentals of Sleep Technology, 2nd edition, Chapter 29, page 324.
18.

A. FP1
B. FP2
C. F7
D. F8
E. T3
F. C3
G. C4
H. T4
I. CZ
J. O1
K. O2
See Fundamentals of Sleep Technology, 2nd edition, Chapter 29, page 316.
CHAPTER
24
Polysomnographic Recording
Procedures
KAREN I. SMITH

NOTE: This chapter corresponds to Chapter 30 in Fundamentals of Sleep


Technology, 2nd edition.

1. The basic requirements for obtaining high-quality polysomnogram


recordings include accurate signal processing, conscientious maintenance of
the recording, and:
A. Proper electrode and sensor application
B. Use of collodion to secure the electrode to the scalp
C. Minimum analog-to-digital conversion rate of 1 GHz
D. Use of a 60 Hz notch filter with sharp drop-off

2. Electrode application is the first step toward obtaining an artifact-free


polysomnogram. This requires precise head measurement, site preparation,
high-quality electrodes, secure adhesion, and:
A. Sufficient skin abrasion to produce a small amount of blood
B. Use of alcohol wipes on the earlobes
C. Electrode impedances less than 5 kΩ
D. Velcro straps to bundle wires together

3. The arrangement of the recording channel selection and their settings is


called a:
A. Derivation
B. Diagnostic set
C. Montage
D. Reference electrode position

4. To ascertain the correct montage for a recording:


A. Allow the PSG system to use its default
B. Review the physician's order
C. Select each channel you prefer
D. Delete the unnecessary channels

5. Bio-calibration assesses:
A. The ability for the patient to hear through the audio system
B. Impedance levels
C. The proper physiologic response in each channel
D. The patient's awake status prior to “lights out”

6. The most common reason patients undergo overnight polysomnography is to


diagnose:
A. Insomnia
B. Sleep-related breathing disorders (SRBD)
C. Narcolepsy
D. Parasomnias

7. If a patient requests early termination of the study, the first response of the
technologist should be:
A. To ignore the patient's request the first and second time it is made
B. To terminate the study immediately and discharge the patient
C. To make a reasonable attempt to get the patient to stay
D. To offer to remove only the respiratory sensors

8. The sleep technologist performing a polysomnogram is responsible for


monitoring the data as they are recorded, making appropriate adjustments
and corrections to the signals, and:
A. Providing a preliminary evaluation of the study to the patient in the
morning
B. Degaussing the amplifiers after the study
C. Replacing electrodes and sensors that have been dislodged
D. Making sure each patient has at least two electrical ground wires
attached

9. Most modern recording systems record and store:


A. Each electrode referenced to linked mastoid electrodes
B. Recordings only as they appear to the technologist
C. Recordings only as specified by the interpreting physician
D. Each electrode recorded against a system reference

10. The filter that attenuates low frequencies but allows high frequencies to pass
is:
A. A notch filter
B. A high-frequency filter (HFF)
C. A low-frequency filter (LFF)
D. A band-pass filter

11. Artifacts detected during the biologic calibration:


A. Should be corrected prior to starting the study
B. Usually do not affect the integrity of the study
C. Should be corrected when the patient awakens
D. Typically resolve on their own and no action is needed

12. Poor electrical contact of the electrodes to the skin usually causes:
A. A small shock to the patient
B. Respiratory artifact
C. Sawtooth waves in the EEG
D. 60 Hz noise

13. System referencing is referencing all electrodes to:


A. A single electrode such as CZ
B. The ground electrode
C. M1
D. M2

14. Excessive filtering:


A. Cleans up an otherwise unreadable recording without affecting the
integrity of the signal
B. Makes it possible to produce an acceptable recording without having to
worry about electrode impedance
C. Affects the integrity of the recording by masking problems that should
be corrected at the source
D. Is often necessary because channels cannot be reformatted or
remontaged once the recording is finalized and stored

15. For best results, common mode rejection requires:


A. Dual ground electrodes
B. Special amplifiers with crossed resistance
C. Extremely low impedance for the ground electrode
D. Low and relatively equal electrode impedances

16. In contrast to muscle artifact, 50/60 Hz interference is:


A. A regular, sinusoidal wave
B. Irregular and varying in amplitude
C. Synchronous with the heart rate
D. Most common in the mastoid electrodes

17. M1 and M2 electrodes should be attached:


A. On the earlobe
B. Over the firm bony area behind the ear
C. As high as possible on the neck
D. Where the mandible attaches to the skull

18. A common slow-frequency artifact may appear synchronous with the


patient's:
A. Eyeblinks
B. ECG
C. Breathing patterns
D. Periodic leg movements

19. The effect of electrode popping in the C4-M1 EEG channel can be reduced
by:
A. Using a linked reference
B. Temporarily increasing the high-frequency filter
C. Temporarily increasing the low-frequency filter
D. Using a 60-Hz notch filter

20. Ending the sleep study should include:


A. Repeating the calibration procedures
B. Attempting to get the patient to leave as quickly as possible
C. Informing the patient of the preliminary results
D. Waiting until stage R sleep begins before waking the patient

DISCUSSION QUESTION
21. The AASM Scoring Manual no longer requires “biologic calibrations.” Does
your center policy and procedures manual require them? What is the value
of biologic calibrations?

22. Describe the steps that should be taken to improve the quality of a poor
recording before the electrode is discarded and replaced.

ANSWERS

1. A, Proper electrode and sensor application.


See Fundamentals of Sleep Technology, 2nd edition, Chapter 30, page 325.

2. C, Electrode impedances less than 5 kΩ.


See Fundamentals of Sleep Technology, 2nd edition, Chapter 30, page 325.

3. C, Montage
See Fundamentals of Sleep Technology, 2nd edition, Chapter 30, page 326.

4. B, Review the physician's order See Fundamentals of Sleep Technology, 2nd


edition, Chapter 30, page 327.

5. C, The proper physiologic response in each channel See Fundamentals of


Sleep Technology, 2nd edition, Chapter 30, page 327.

6. B, Sleep-related breathing disorders (SRBD) See Fundamentals of Sleep


Technology, 2nd edition, Chapter 30, page 327.

7. C, To make a reasonable attempt to get the patient to stay See Fundamentals


of Sleep Technology, 2nd edition, Chapter 30, page 328.

8. C, Replacing electrodes and sensors that have been dislodged See


Fundamentals of Sleep Technology, 2nd edition, Chapter 30, page 328.

9. D, Each electrode recorded against a system reference See Fundamentals of


Sleep Technology, 2nd edition, Chapter 30, page 329.
10. C, A low-frequency filter (LFF) See Fundamentals of Sleep Technology, 2nd
edition, Chapter 30, page 330.

11. A, Should be corrected prior to starting the study See Fundamentals of Sleep
Technology, 2nd edition, Chapter 30, page 331.

12. D, 60 Hz noise
See Fundamentals of Sleep Technology, 2nd edition, Chapter 30, page 333.

13. A, A single electrode such as CZ


See Fundamentals of Sleep Technology, 2nd edition, Chapter 30, page 329.

14. C, Affects the integrity of the recording by masking problems that should be
corrected at the source See Fundamentals of Sleep Technology, 2nd edition,
Chapter 30, page 330.

15. D, Low and relatively equal electrode impedances See Fundamentals of


Sleep Technology, 2nd edition, Chapter 30, page 325.

16. A, A regular, sinusoidal wave See Fundamentals of Sleep Technology, 2nd


edition, Chapter 30, page 333.

17. B, Over the firm bony area behind the ear See Fundamentals of Sleep
Technology, 2nd edition, Chapter 30, page 335.

18. C, Breathing patterns


See Fundamentals of Sleep Technology, 2nd edition, Chapter 30, page 336.

19. C, Temporarily increasing the low-frequency filter See Fundamentals of


Sleep Technology, 2nd edition, Chapter 30, page 336.

20. A, Repeating the calibration procedures See Fundamentals of Sleep


Technology, 2nd edition, Chapter 30, page 339.

21. Biologic calibrations provide an indication that the various sensors are
working properly based on polygraphic response to specific patient
maneuvers. These calibrations ensure that eye movements can be seen in
REM and slow-wave sleep, apnea and other breathing disturbances will be
identified, and EMG recordings are adequate.
See Fundamentals of Sleep Technology, 2nd edition, Chapter 30, page 331.
Berry RB, Brooks R, Gamaldo CE, et al.; for the American Academy of Sleep Medicine. The AASM
Manual for the Scoring of Sleep and Associated Events: Rules, Terminology and Technical
Specifications, Version 2.0.2. www.aasmnet.org, Darien, IL: American Academy of Sleep Medicine,
2013.

22. In order to improve the quality of a poor recording, check electrode


impedances, re-prep, and reapply a problem electrode before the electrode is
discarded and replaced.
See Fundamentals of Sleep Technology, 2nd edition, Chapter 30, page 331.
Berry RB, Brooks R, Gamaldo CE, et al.; for the American Academy of Sleep Medicine. The AASM
Manual for the Scoring of Sleep and Associated Events: Rules, Terminology and Technical
Specifications, Version 2.0.2. www.aasmnet.org, Darien, IL: American Academy of Sleep Medicine,
2013.
CHAPTER
25
Cardiac Arrhythmias
JON W. ATKINSON

NOTE: This chapter corresponds to Chapter 31 in Fundamentals of Sleep


Technology, 2nd edition.

1. According to the American Academy of Sleep Medicine (AASM) Scoring


Manual, a sustained (30 seconds or longer) heart rate of 90 beats per minute
or higher during sleep is called:
A. Atrial fibrillation
B. Left bundle branch block
C. Bradycardia
D. Tachycardia

2. According to the AASM Scoring Manual, a pause in the heart rate of 3


seconds or longer is called:
A. Asystole
B. Bradycardia
C. Prolonged QT interval
D. Premature ventricular contraction

3. The cardiac cycle begins with depolarization of the atria. This is caused by
discharges originating from the:
A. Bundle of His
B. Sinoatrial node
C. Purkinje system
D. Cathode

4. The signal from the atrioventricular node passes through the atrioventricular
bundle, the bundle of His, and the left and right bundles to cause contraction
of the:
A. Mitral valve
B. Ventricles
C. Aorta
D. Atrium

5. In the ECG, the pause between the atrial depolarization and the ventricular
depolarization in a normal sinus rhythm is the:
A. T wave
B. QRS complex
C. PR interval
D. Junction block

6. Each P wave in a normal sinus heart rhythm is associated with:


A. Two or more QRS complexes
B. A single QRS complex
C. A premature ventricular contraction
D. Sinus arrhythmia

7. Heartbeats that do originate in the atrium, atrioventricular node, or the


ventricle, and not in the sinoatrial node, are not associated with a:
A. P wave
B. Ventricular contraction
C. Depolarization
D. Repolarization

8. Measuring P–P or R–R intervals allows the technologist to evaluate whether


the heart rhythm is:
A. Associated with a widened QRS complex
B. Originating outside of the sinoatrial node
C. Regular or irregular
D. Resulting in adequate blood flow

9. A simple way for a sleep technologist to estimate heart rate is to count the
number of QRS complexes in a 30-second epoch and:
A. Write down the number as the heart rate
B. Divide by 30
C. Multiply by 2
D. Determine the interval in millimeters

10. The QRS complex in a patient with sinus bradycardia is:


A. Widened
B. Present and each appears the same
C. Transformed into a QR–T complex due to the absence of the S wave
D. Not linked to a P wave

11. Sinus arrhythmia in a 5-year-old child is:


A. Normal
B. Potentially life threatening
C. Due to abnormal Purkinje bundle conduction
D. The most common reason for pacemaker implantation in children

12. During atrial flutter, P waves are:


A. Higher than normal amplitude
B. Locked to QRS waves on a 1:1 ratio
C. Replaced by sawtooth deflections
D. Replaced by K complexes

13. In some forms of junctional rhythm, the P wave may be:


A. Normal
B. Associated with multiple QRS complexes
C. Caused by mitral valve prolapse
D. Hidden within the QRS complex

14. First-degree atrioventricular (AV) block is characterized by:


A. Absence of P waves
B. Absence of QRS complexes
C. Markedly increased heart rate
D. Prolonged PR interval

15. In third-degree atrioventricular (AV) block, both the P–P interval and the R–
R interval are regular but:
A. There is no QRS complex
B. The PR interval is shortened and regular
C. The rate is much higher than normal
D. The rates are dissociated and not the same

16. In bundle branch block, the QRS interval is:


A. Widened
B. Normal
C. Shortened
D. Unrelated to the P wave

17. Multifocal premature ventricular complexes (PVCs) can be identified


because beats arising from different locations within the heart have:
A. Different morphologies
B. Different beat-to-beat intervals
C. Prominent notches at the top of the waveform
D. Couplets

18. The QRS complex in ventricular tachycardia is:


A. Normal
B. Absent
C. Wide and bizarre
D. Narrow and uniform

19. Ventricular fibrillation is:


A. Seen from time to time in normal patients
B. Always a medical emergency
C. Associated with normal blood flow from the heart
D. A regular sinus rhythm

20. AASM Sleep Manual guidelines recommend ECG recordings:


A. From linked ear electrodes
B. Using the left leg and right mastoid electrodes
C. Only on patients suspected of obstructive sleep apnea
D. Using a modified lead II placement

IDENTIFY THE HEART RHYTHM IN THE


FOLLOWING TRACINGS

21. Ten-second sample: Figure 25-1.


Figure 25.1

22. Ten-second sample: Figure 25-2.

Figure 25.2

23. Ten-second sample (two channels): Figure 25-3.

Figure 25.3

24. Ten-second sample: Figure 25-4.

Figure 25.4

25. Ten-second sample: Figure 25-5. There are three arrhythmias present.
Identify the arrhythmias at A, B, and C.
Figure 25.5

26. Ten-second sample: Figure 25-6.

Figure 25.6

27. Ten-second sample: Figure 25-7.

Figure 25.7

28. Three contiguous 12-second samples: Figure 25-8.


Figure 25.8

29. Thirty-second sample: Figure 25-9.

Figure 25.9

30. Thirty-second sample: Figure 25-10.

Figure 25.10

31. Thirty-second epoch: Figure 25-11. Identify the heart rhythms in box A and
in box B.
Figure 25.11

32. Thirty-second sample: Figure 25-12. Identify the arrhythmia in box A and in
box B.

Figure 25.12

33. Thirty-second sample: Figure 25-13.

Figure 25.13

34. Thirty-second sample: Figure 25-14. Identify the arrhythmia in the box.

Figure 25.14

35. Thirty-second epoch: Figure 25-15. Identify the heart rhythm in box A and in
box B.

Figure 25.15

DISCUSSION QUESTION

36. Review the recommendations for scoring arrhythmias in the AASM Manual.
Are these distinctions adequate for the identification of cardiac
emergencies? Are these criteria adequate for identification of arrhythmia
types?

ANSWERS

1. D, Tachycardia.
See Fundamentals of Sleep Technology, 2nd edition, Chapter 31, page 340.

2. A, Asystole.
See Fundamentals of Sleep Technology, 2nd edition, Chapter 31, page 340.

3. B, Sinoatrial node.
See Fundamentals of Sleep Technology, 2nd edition, Chapter 31, page 341.

4. B, Ventricles.
See Fundamentals of Sleep Technology, 2nd edition, Chapter 31, page 341.

5. C, PR interval.
See Fundamentals of Sleep Technology, 2nd edition, Chapter 31, page 342.
6. B, A single QRS complex.
See Fundamentals of Sleep Technology, 2nd edition, Chapter 31, page 342.

7. A, P wave.
See Fundamentals of Sleep Technology, 2nd edition, Chapter 31, page 342.

8. C, Regular or irregular.
See Fundamentals of Sleep Technology, 2nd edition, Chapter 31, page 343.

9. C, Multiply by 2.
See Fundamentals of Sleep Technology, 2nd edition, Chapter 31, page 344.

10. B, Present and each appears the same.


See Fundamentals of Sleep Technology, 2nd edition, Chapter 31, page 344.

11. A, Normal.
See Fundamentals of Sleep Technology, 2nd edition, Chapter 31, page 346.

12. C, Replaced by sawtooth deflections.


See Fundamentals of Sleep Technology, 2nd edition, Chapter 31, page 346.

13. D, Hidden within the QRS complex.


See Fundamentals of Sleep Technology, 2nd edition, Chapter 31, page 349.

14. D, Prolonged PR interval.


See Fundamentals of Sleep Technology, 2nd edition, Chapter 31, page 351.

15. D, The rates are dissociated and not the same.


See Fundamentals of Sleep Technology, 2nd edition, Chapter 31, page 352.

16. A, Widened.
See Fundamentals of Sleep Technology, 2nd edition, Chapter 31, page 353.

17. A, Different morphologies.


See Fundamentals of Sleep Technology, 2nd edition, Chapter 31, page 354.

18. C, Wide and bizarre.


See Fundamentals of Sleep Technology, 2nd edition, Chapter 31, page 357.

19. B, Always a medical emergency.


See Fundamentals of Sleep Technology, 2nd edition, Chapter 31, page 357.

20. D, Using a modified lead II placement.


See Fundamentals of Sleep Technology, 2nd edition, Chapter 31, page 358.
21. There are no P waves, and there are five consecutive beats with wide bizarre
QRS complex at a rate of about 150 beats per minute. This is ventricular
tachycardia, classified as wide complex tachycardia according to AASM
guidelines.

22. There is an increasingly longer PR interval at beats 3, 4, and 5 and again at 6,


7, and 8. Each sequence is followed by a nonconducted P wave. This is
second-degree AV block, Mobitz I, Wenckebach phenomenon.

23. Starting at beat 4, there is a 10-beat run of supraventricular tachycardia at a


rate of about 150 beats per minute. This could be classified as paroxysmal
supraventricular tachycardia since it occurs abruptly and disappears just as
abruptly. List it under narrow complex tachycardia for AASM purposes.

24. Note the P wave standing by itself between the fifth and the sixth R waves.
There is no corresponding QRS complex. The previous beats have constant
PR intervals. This is second-degree AV block, Mobitz II.

25. A, Examining the P wave reveals an inverted P wave with a slightly short PR
interval. The QRS complex is normal, and the beat occurs earlier than
expected. Hence, this is a premature junctional contraction (PJC). B, The P
wave in this section has a somewhat different appearance from the other P
waves in the sample. The QRS complex is normal and occurs earlier than
expected. This is a premature atrial contraction (PAC). C, In box C, there is a
beat occurring earlier than expected with a wide, bizarre QRS complex and
no P wave preceding it. This is a premature ventricular contraction (PVC).

26. The irregularly irregular ventricular rhythm with the absence of distinct P
wave is the key to identifying this as atrial fibrillation.

27. There is a 3.5- to 4-second period between the second and third ECG
complexes. No P wave or QRS complexes are present. This is asystole.

28. There is a sudden increase in heart rate to 100 to 105 beats per minute, and the
P wave is inverted. This is junctional tachycardia, and could be classified as
narrow complex tachycardia. Note that the QRS complex is slightly
prolonged and that the QRS complex is notched suggesting the presence of a
bundle branch block. The scoring manual does not address what to do with
QRS complexes arising from supraventricular origins that are widened by the
presence of bundle branch block.
29. There is a widened bizarre QRS complex without a corresponding P wave
occurring every third beat. This is ventricular trigeminy.

30. This sample shows a sinus rhythm with P waves and QRS complexes for
every beat and a regular rhythm of 105 beats per minute. This is sustained
sinus tachycardia.

31. A, In this section, there is a nonconducted P wave. This is preceded by a


number of ECG complexes with progressively lengthening PR intervals. This
demonstrates second-degree AV block, Mobitz I. B, Box B shows a fairly
regular rhythm with a 1:1 P:QRS ratio. The PR interval is prolonged at about
0.26 seconds. This is first-degree AV block.

32. A, The 10th beat has a P wave of different appearance than the other P wave in
the example. Therefore, it has started in a different location in the atrium. It
occurs earlier than expected, hence a PAC, premature atrial contraction. B,
The next beat does not have an associated P wave and is wide and bizarre
compared to the other QRS complexes. It is therefore a PVC, premature
ventricular contraction.

33. There is a wide QRS complex without associated P wave occurring every
other beat and earlier than what one would expect. This is ventricular
bigeminy.

34. Note the absence of P wave and the wide, bizarre QRS complexes although of
a different shape. These are paired (back to back), hence a multifocal
ventricular couplet.

35. A, There is an irregularly irregular ventricular rhythm with the absence of


normal P waves that have been replaced by rapid oscillations of varying size,
shape, and duration. This is atrial fibrillation. B, There is a spontaneous
change from atrial fibrillation to normal sinus rhythm. Note the sudden
change to normal P waves with normal, regular R waves at a rate of 55 to 60
beats per minute.

36. “Changes in body position, frequent and sometimes mandated during


diagnostic PSG, are known to result in ST segment fluctuations, thus
rendering the tracing unreliable for inclusion or exclusion of ischemic
changes. Because the amplitudes of clinically significant ST changes are as
small as 1 mm, a degraded signal related to muscle or movement artifact
would hinder reliability in assessing waveform changes. It would be
anticipated, furthermore, that an excessive amount of technician time and
effort would be spent tending to loose chest or limb electrodes. Finally, a
consensus following a nonsystematic review (no evidence grade) by the
American Heart Association (AHA) in 2004 recommended more than one
precordial lead to accurately detect ECG changes of acute myocardial
ischemia. Nevertheless, in the setting of chest pain occurring during
polysomnographic monitoring, the ST segments or T waves may be used by a
physician to supplement the clinical evaluation. A further potential
shortcoming of the single ECG channel relates to the widened QRS complex.
There are limited data to suggest that limb leads without an accompanying
precordial lead, such as V1, pose limitations for differentiating ventricular
from supraventricular origin in evaluating wide complex tachycardias. … The
Cardiac Task Force agreed that conventional definitions for abnormal
tachycardias could be modified for the purposes of interpretation during PSG.
A wide complex tachycardia is a sustained rhythm lasting more than three
cardiac cycles with a QRS duration ≥120 ms and a rate >100 bpm. A narrow
complex tachycardia is a sustained rhythm lasting more than three cardiac
cycles with a QRS duration <120 ms and a rate >100 bpm. Atrial fibrillation
is an irregularly irregular ventricular rhythm associated with the replacement
of P waves with rapid oscillations or waves that vary in size, shape, and
timing. This definition of atrial fibrillation is consistent with AHA
guidelines.” (Caples SM, Somers VK, Rosen CL, et al. The scoring of cardiac
events during sleep. J Clin Sleep Med. 2007;3(2):147–154).
CHAPTER
26
Adult Sleep Scoring
HARRY R. WHITMORE

NOTE: This chapter corresponds to Chapter 32 in Fundamentals of Sleep


Technology, 2nd edition.

Scoring Updates: There have been updates to the AASM scoring manual since
the publication of Chapter 32 “Adult Sleep Scoring” in The Fundamentals of
Sleep Technology, 2nd Edition. Chapter 32 used the 2007 The AASM Manual for
the Scoring of Sleep and Associated Events (1) as its primary source. There have
been four revisions as of August of 2014, all being made available exclusively in
electronic form. The current version of the scoring manual is 2.1 and is available
on the AASM Web site (2)
(http://www.aasmnet.org/scoringmanual/myscoringmanual.aspx). There is also a
link there to a summary of the differences between the 2007 manual and version
2.1 of the manual.
The most significant changes in version 2.1 are to the respiratory rules.
Namely, the addition of polyvinylidene fluoride (PVDF) thermal sensors in the
definition of RECOMMENDED sensors for thermal airflow and polyvinylidene
fluoride (PVDF) belts and SUM channel as ACCEPTABLE for the monitoring
of respiratory effort. More importantly, clarifications were made to the rule for
measuring the duration of respiratory events and the rules for scoring apneas and
hypopneas. Many other minor changes were also made, including the coding of
rules in the manual. In Version 2.1 additional changes were made to the scoring
of sleep stages sections of the manual and numerous figures were added to
clarify these rules. Thus, the reader is strongly encouraged to review version 2.1
of the scoring manual as the following questions are based on it, rather than the
2007 manual.

1. What is the predominant EEG rhythm seen in wakefulness?


A. Low-amplitude mixed frequency
B. Theta
C. Alpha
D. Slow waves

2. What is the predominant EEG rhythm seen in N2 sleep?


A. Low-amplitude mixed frequency
B. Sawtooth waves
C. Alpha
D. Slow-wave activity

3. What waveform defines N3 sleep?


A. Low-amplitude mixed frequency
B. Sawtooth waves
C. Alpha
D. Slow waves

4. What waveforms would one expect to see in N2 sleep?


A. K complexes and sleep spindles
B. Sawtooth waves
C. Vertex sharp waves
D. Slow waves

5. At what frequency do slow waves oscillate?


A. 0.5 to 4 Hz
B. 3 to 7 Hz
C. 8 to 13 Hz
D. 0.5 to 2 Hz

6. At what frequency does the alpha rhythm oscillate?


A. 0.5 to 4 Hz
B. 3 to 7 Hz
C. 8 to 13 Hz
D. 0.5 to 2 Hz

7. What is the minimal recommended sampling rate for EEG?


A. 50 Hz
B. 10 Hz
C. 1 Hz
D. 200 Hz

8. What is the recommended low-frequency filter setting for EEG?


A. 50 Hz
B. 100 Hz
C. 0.3 Hz
D. 35 Hz

9. What is the recommended high-frequency filter setting for EEG?


A. 50 Hz
B. 100 Hz
C. 0.3 Hz
D. 35 Hz

10. What is the recommended sensor for the scoring of hypopneas during a
diagnostic study?
A. Oronasal thermal airflow sensor
B. Microphone
C. Oronasal pressure transducer
D. Respiratory inductance plethysmography belts

11. What is the recommended sensor for the scoring of apneas during a
diagnostic study?
A. Oronasal thermal airflow sensor
B. Microphone
C. Oronasal pressure transducer
D. Respiratory inductance plethysmography belts

12. From which derivation might one expect to see the highest amplitude for
slow-wave activity?
A. Central derivations
B. Occipital derivations
C. Frontal derivations
D. EOG derivations

13. From which derivation might one expect to see the highest amplitude for
alpha activity?
A. Central derivations
B. Occipital derivations
C. Frontal derivations
D. EOG derivations
14. From which derivation might one expect to see the highest amplitude for
vertex sharp waves?
A. Central derivations
B. Occipital derivations
C. Frontal derivations
D. EOG derivations

15. From which derivation might one expect to see the highest amplitude for K
complexes?
A. Central derivations
B. Occipital derivations
C. Frontal derivations
D. EOG derivations

16. A slow eye movement is defined as having an initial deflection that is:
A. >1 second
B. >500 msec
C. <500 msec
D. >1,000 msec

17. Assign a sleep stage to the 30-second epoch in Figure 26-1.


Figure 26.1

A. W
B. N1
C. R
D. N2
E. N3

18. In order for an arousal to be scored, it must be:


A. At least 0.5 seconds in duration
B. At least 3 seconds in duration
C. At least 2 seconds in duration
D. At least 1,000 msec in duration

19. Assign a sleep stage to the 30-second epoch in Figure 26-2.


Figure 26.2

A. W
B. N1
C. R
D. N2
E. N3

20. In order for an arousal to be scored in stage R sleep, what is required other
than an abrupt change in EEG frequency?
A. A burst of rapid eye movements
B. A K complex or sleep spindle
C. A simultaneous increase in submental EMG for at least 1 second
D. Six seconds of slow-wave activity

21. Assign a sleep stage to the 30-second epoch in Figure 26-3.


Figure 26.3

A. W
B. N1
C. R
D. N2
E. N3

22. It is possible to score an arousal in an epoch staged as W. True or false?


23. Assign a sleep stage to the 30-second epoch in Figure 26-4.


Figure 26.4

A. W
B. N1
C. R
D. N2
E. N3

24. What is the minimum amplitude necessary to score a slow wave?


A. 150 μV
B. 75 μV
C. 35 μV
D. 45 μV
E. There are no amplitude criteria for slow waves.

25. Assign a sleep stage to the 30-second epoch in Figure 26-5.


Figure 26.5

A. W
B. N1
C. R
D. N2
E. N3

26. What is the minimum amplitude necessary to score a K complex?


A. 150 μV
B. 75 μV
C. 35 μV
D. 45 μV
E. There are no amplitude criteria for K complexes.

27. Identify the activity in the boxes in the 2-minute window in Figure 26-6.
Figure 26.6

A. One PLM
B. A series of five PLMs
C. Five hypopneas
D. Movement artifact
E. Five body movements

28. In adults, an apnea must be of what duration in order to be scored?


A. Five seconds
B. Eight seconds
C. Ten seconds
D. >500 msec
E. <500 msec

29. Identify the event in the box in the 1-minute window in Figure 26-7.
Figure 26.7

A. An arousal
B. An RERA
C. A mixed apnea
D. A central apnea
E. An obstructive apnea

30. Identify the event in the box in the 2-minute window in Figure 26-8.
Figure 26.8

A. An arousal
B. An RERA
C. A mixed apnea
D. A central apnea
E. An obstructive apnea

31. Identify the event in the box in the 1-minute window in Figure 26-9.
Figure 26.9

A. A hypopnea
B. An RERA
C. A mixed apnea
D. A central apnea
E. An obstructive apnea

32. Identify the event in the box in the 2-minute window in Figure 26-10.
Figure 26.10

A. A hypopnea (1B)
B. An RERA
C. A mixed apnea
D. A central apnea
E. An obstructive apnea

33. Identify the event in the box in the 2-minute window in Figure 26-11.
Figure 26.11

A. A hypopnea (1A)
B. An RERA
C. A mixed apnea
D. A central apnea
E. An obstructive apnea

34. Identify the event in the box in the 30-second window in Figure 26-12.
Figure 26.12

A. One PLM
B. An RERA
C. Bruxism
D. An arousal
E. A body movement

ADULT SLEEP SCORING SUPPLEMENT: EPOCH


SCORING
The images in this supplement are courtesy of the American Academy of Sleep
Medicine, reproduced by permission.

35. Assign a sleep stage to the 30-second epoch in Figure 26-E1.


Figure 26.13

A. W
B. N1
C. R
D. N2
E. N3

36. Assign a sleep stage to the 30-second epoch in Figure 26-E2.


Figure 26.14

A. W
B. N1
C. R
D. N2
E. N3

37. Assign a sleep stage to the 30-second epoch in Figure 26-E3.


Figure 26.15

A. W
B. N1
C. R
D. N2
E. N3

38. Assign a sleep stage to the 30-second epoch in Figure 26-E4.


Figure 26.16

A. W
B. N1
C. R
D. N2
E. N3

39. Assign a sleep stage to the 30-second epoch in Figure 26-E5.


Figure 26.17

A. W
B. N1
C. R
D. N2
E. N3

40. Assign a sleep stage to the 30-second epoch in Figure 26-E6.


Figure 26.18

A. W
B. N1
C. R
D. N2
E. N3

41. Assign a sleep stage to the 30-second epoch in Figure 26-E7.


Figure 26.19

A. W
B. N1
C. R
D. N2
E. N3

42. Assign a sleep stage to the 30-second epoch in Figure 26-E8.


Figure 26.20

A. W
B. N1
C. R
D. N2
E. N3

43. Assign a sleep stage to the 30-second epoch in Figure 26-E9.


Figure 26.21

A. W
B. N1
C. R
D. N2
E. N3

44. Assign a sleep stage to the 30-second epoch in Figure 26-E10.


Figure 26.22

A. W
B. N1
C. R
D. N2
E. N3

45. Assign a sleep stage to the 30-second epoch in Figure 26-E11.


Figure 26.23

A. W
B. N1
C. R
D. N2
E. N3

46. Assign a sleep stage to the 30-second epoch in Figure 26-E12.


Figure 26.24

A. W
B. N1
C. R
D. N2
E. N3

47. Assign a sleep stage to the 30-second epoch in Figure 26-E13A.


Figure 26.25

A. W
B. N1
C. R
D. N2
E. N3

48. Assign a sleep stage to the 30-second epoch in Figure 26-E13B.


Figure 26.26

A. W
B. N1
C. R
D. N2
E. N3

49. Assign a sleep stage to the 30-second epoch in Figure 26-E14.


Figure 26.27

A. W
B. N1
C. R
D. N2
E. N3

50. Assign a sleep stage to the 30-second epoch in Figure 26-E15A (review
Figure 26-E15B prior to scoring this epoch).
Figure 26.28

A. W
B. N1
C. R
D. N2
E. N3

51. Assign a sleep stage to the 30-second epoch in Figure 26-E15B.


Figure 26.29

A. W
B. N1
C. R
D. N2
E. N3

ANSWERS

1. C, Alpha. The alpha rhythm is the predominant EEG rhythm seen during
wakefulness. Most individuals will demonstrate alpha with eyes closed, with
only a small minority (about 10%) displaying activity similar to eye opening
during eyes closed (2.1 Rule IV E1 and E2, as well as Note 2).
See Fundamentals of Sleep Technology, 2nd edition, Chapter 32, page 368.

2. A, Low-amplitude mixed frequency. The predominant rhythm during N2 sleep


is low-amplitude, mixed-frequency EEG activity. Although the presence of K
complexes and sleep spindles define the beginning and continuation of N2
sleep, the predominant background rhythm is low-amplitude, mixed-
frequency EEG activity. Since neither K complex nor sleep spindle is offered
as an answer choice, Option A (low-amplitude mixed frequency) is the best
answer (2.1 Rule IV G2, G3 and G4).
See Fundamentals of Sleep Technology, 2nd edition, Chapter 32, page 369.

3. D, Slow waves. The waveform that defines N3 sleep is slow-wave activity


(2.1 Rule IV H1 and H2).
See Fundamentals of Sleep Technology, 2nd edition, Chapter 32, page 371.

4. A, K complexes and sleep spindles. These are the waveforms that define N2,
and thus, we would expect to see are K complexes and sleep spindles (2.1
Rule IV G1 and G2).
See Fundamentals of Sleep Technology, 2nd edition, Chapter 32, pages 365–366.

5. D, 0.5 to 2 Hz. Slow waves oscillate at between 0.5 and 2 Hz (2.1 Rule IV
H1). Although many investigators have used quantitative EEG analysis
results in the range of 0.5 to 4 Hz or so-called delta slow-wave activity to
measure sleep depth, for the purposes of visually sleep stage scoring, a slow
wave is considered to be between 0.5 and 2 Hz.
See Fundamentals of Sleep Technology, 2nd edition, Chapter 32, page 366.

6. C, 8 to 13 Hz. Alpha rhythm is defined as “trains of sinusoidal 8 to 13 Hz


activity recorded over the occipital region with eye closure, attenuation with
eye opening” (2.1 Rule IV E1).
See Fundamentals of Sleep Technology, 2nd edition, Chapter 32, page 363.

7. D, 200 Hz. The minimal acceptable sampling rate for EEG is 200 Hz (2.1
Rule III A3).
See Fundamentals of Sleep Technology, 2nd edition, Chapter 32, page 363.

8. C, 0.3 Hz. The recommended low-frequency filter setting for EEG is 0.3 Hz
(2.1 Rule III A4).
See Fundamentals of Sleep Technology, 2nd edition, Chapter 32, page 363.

9. D, 35 Hz. The recommended high-frequency filter setting for EEG is 35 Hz


(2.1 Rule III A4).
See Fundamentals of Sleep Technology, 2nd edition, Chapter 32, page 363.

10. C, Oronasal pressure transducer. The recommended sensor for the scoring of
hypopneas during a diagnostic study is an oronasal pressure transducer (2.1
Rule VIII part 1 A3).
See Fundamentals of Sleep Technology, 2nd edition, Chapter 32, page 373.

11. A, Oronasal thermal airflow sensor. The recommended sensor for the scoring
of apneas during a diagnostic study is an oronasal thermal airflow sensor (2.1
Rule VIII part 1 A1).
See Fundamentals of Sleep Technology, 2nd edition, Chapter 32, page 373.

12. C, Frontal derivations. One would expect to see the highest amplitude for
slow-wave activity from the frontal derivations (2.1 Rule IV H2).
See Fundamentals of Sleep Technology, 2nd edition, Chapter 32, page 371.

13. B, Occipital derivations. One would expect to see the highest amplitude for
alpha activity from the occipital derivations (2.1 Rule IV E1).
See Fundamentals of Sleep Technology, 2nd edition, Chapter 32, page 363.

14. A, Central derivations. One would expect to see the highest amplitude for
vertex sharp waves from the central derivations (2.1 Rule IV F1).
See Fundamentals of Sleep Technology, 2nd edition, Chapter 32, page 365.

15. C, Frontal derivations. One would expect to see the highest amplitude for K
complexes from the frontal derivations (2.1 Rule IV G1).
See Fundamentals of Sleep Technology, 2nd edition, Chapter 32, pages 365–366.

16. B, >500 msec. A slow eye movement is defined as having an initial deflection
of >500 msec (2.1 Rule IV F1).
See Fundamentals of Sleep Technology, 2nd edition, Chapter 32, page 368.

17. A, W. This 30-second epoch displays eyeblinks during the first 14 seconds of
the epoch and about 16 seconds of alpha activity during the last 16 seconds
and thus meets the definition of stage W (2.1 Rule IV E1 and E2).
See Fundamentals of Sleep Technology, 2nd edition, Chapter 32, page 368.

18. B, At least 3 seconds in duration. In order for an arousal to be scored, it must


be at least 3 seconds in duration (2.1 Rule V A1).
See Fundamentals of Sleep Technology, 2nd edition, Chapter 32, page 368.

19. B, N1. This 30-second epoch displays about 26 seconds of low-amplitude,


mixed-frequency EEG activity as well as slow eye movements and thus meets
the definition for N1 sleep (2.1 Rule IV F1, F2 and F3).
See Fundamentals of Sleep Technology, 2nd edition, Chapter 32, pages 369–370.

20. C, A simultaneous increase in submental EMG for at least 1 second. In order


for an arousal to be scored in REM sleep, a concurrent increase in submental
EMG is necessary for at least 1 second of the 3-second increase in EEG
activity (2.1 Rule V A1).
See Fundamentals of Sleep Technology, 2nd edition, Chapter 32, page 368.

21. E, N3. This 30-second epoch displays about 14 seconds of slow-wave activity,
which meets the definition of N3 sleep (2.1 Rule IV H1 and H2).
See Fundamentals of Sleep Technology, 2nd edition, Chapter 32, page 371.

22. True; an arousal can be scored in an epoch scored as wake as long as there is
10 seconds of sleep before the arousal (2.1 Rule V, Note 2).
See Fundamentals of Sleep Technology, 2nd edition, Chapter 32, page 368.

23. C, R. This 30-second epoch demonstrates low-amplitude, mixed-frequency


EEG; low chin EMG tone; and rapid eye movements in the EOG channels—
all markers of stage R sleep (2.1 Rule IV I1 and I2).
See Fundamentals of Sleep Technology, 2nd edition, Chapter 32, pages 371–372.

24. B, 75 μV. A slow wave requires an amplitude of at least 75 μV (2.1 Rule IV


H1).
See Fundamentals of Sleep Technology, 2nd edition, Chapter 32, page 371.

25. D, N2. This 30-second epoch demonstrates low-amplitude, mixed-frequency


EEG with prominent sleep spindles and K complexes that meet the definition
of N2 sleep (2.1 Rule IV G1 and G2).
See Fundamentals of Sleep Technology, 2nd edition, Chapter 32, page 371.

26. E, There are no amplitude criteria for K complexes. There are no amplitude
criteria for K complexes (2.1 Rule IV G1).
See Fundamentals of Sleep Technology, 2nd edition, Chapter 32, page 365.

27. B, A series of five PLMs.


A series of five PLMs (2.1 Rule VII A1 and A2).
See Fundamentals of Sleep Technology, 2nd edition, Chapter 32, page 378.

28. C, Ten seconds. An apnea must be at least 10 seconds in duration in order to


be scored (2.1 Rule VIII C1b).
See Fundamentals of Sleep Technology, 2nd edition, Chapter 32, page 373.

29. E, An obstructive apnea.


The box in this 60-second epoch demonstrates an obstructive apnea. Note the absence of thermal
airflow for at least 10 seconds and the continued respiratory effort for the entire duration of the absence
of airflow (2.1 Rule VIII C1 and C2).
See Fundamentals of Sleep Technology, 2nd edition, Chapter 32, page 374.

30. C, A mixed apnea.


The box in this 120-second epoch demonstrates a mixed apnea. Note the absence of thermal airflow for
at least 10 seconds and the lack of respiratory effort during the initial portion of the period with an
absence of thermal airflow that resumes before thermal airflow resumes (2.1 Rule VIII C1 and C4).
See Fundamentals of Sleep Technology, 2nd edition, Chapter 32, page 374.

31. D, A central apnea.


The box in this 120-second epoch demonstrates a central apnea. Note the absence of thermal airflow for
at least 10 seconds as well as the concurrent absence of respiratory effort (2.1 Rule VIII C1 and C3).
See Fundamentals of Sleep Technology, 2nd edition, Chapter 32, page 374.

32. A, A hypopnea (1B). The box in this 120-second epoch demonstrates a


hypopnea according to rule 1B. Note the ≥30% decrease in signal of the nasal
pressure signal and the associated 5% oxygen desaturation (2.1 Rule VIII D
1B).
See Fundamentals of Sleep Technology, 2nd edition, Chapter 32, page 374.

33. A, A hypopnea (1A). The box in this 120-second epoch demonstrates a


hypopnea according to rule 1A. Note the ≥30% decrease in signal of the nasal
pressure signal and the associated arousal. There was no desaturation
associated with the event, and thus the event does not meet the criteria for
hypopnea rule 1B (2.1 Rule VIII D 1A and 1B).
See Fundamentals of Sleep Technology, 2nd edition, Chapter 32, page 374.

34. C, Bruxism. The box in this 30-second epoch demonstrates bruxism. Note the
seven bursts of muscle activity in the chin EMG that are also visible as
muscle artifact in the EEG and that occur in a regular sequence (2.1 Rule VII
E 1b).
See Fundamentals of Sleep Technology, 2nd edition, Chapter 32, page 380.

REFERENCES
Berry RB, Brooks R, Gamaldo CE, et al.; for the American Academy of Sleep
Medicine. The AASM Manual for the Scoring of Sleep and Associated Events:
Rules, Terminology and Technical Specifications, Version 2.0.3.
www.aasmnet.org, Darien, IL: American Academy of Sleep Medicine, 2014.
Berry RB, Budhiraja R, Gottlieb DJ, et al. Rules for scoring respiratory events
in sleep: update of the 2007 AASM Manual for the Scoring of Sleep and
Associated Events. J Clin Sleep Med. 2012;8(5):597–619.
Iber C, Ancoli-Israel S, Chesson AL Jr, et al. The AASM Manual for the
Scoring of Sleep and Associated Events: Rules, Terminology and Technical
Specifications. Westchester, IL: American Academy of Sleep Medicine, 2007.

ADULT SLEEP SCORING SUPPLEMENT: EPOCH


SCORING ANSWERS

35. A, W.

36. C, R.

37. D, N2

38. D, N2

39. C, R.

40. E, N3

41. A, W

42. B, N1

43. B, N1

44. E, N3

45. D, N2

46. D, N2

47. C, R.

48. B, N1

49. A, W

50. C, R.

51. C, R.
ADDITIONAL REFERENCES
Berry RB, Brooks R, Gamaldo CE, et al.; for the American Academy of Sleep
Medicine. The AASM Manual for the Scoring of Sleep and Associated Events:
Rules, Terminology and Technical Specifications, Version 2.1.
www.aasmnet.org, Darien, IL: American Academy of Sleep Medicine, 2014.
Berry RB, Budhiraja R, Gottlieb DJ, et al. Rules for scoring respiratory events
in sleep: update of the 2007 AASM Manual for the Scoring of Sleep and
Associated Events. J Clin Sleep Med. 2012;8(5):597–619.
Iber C, Ancoli-Israel S, Chesson AL, Jr, et al. The AASM Manual for the
Scoring of Sleep and Associated Events: Rules, Terminology and Technical
Specifications. Westchester, IL: American Academy of Sleep Medicine, 2007.
CHAPTER
27
Report Generation
JON W. ATKINSON

NOTE: This chapter corresponds to Chapter 33 in Fundamentals of Sleep


Technology, 2nd edition.

Consider the following information for questions 1 to 3.
Lights off epoch = 15
Lights on epoch = 931
Sleep onset epoch = 30
Stage R onset epoch = 42
1. What is the total recording time (TRT)?

2. What is sleep onset latency (SOL)?

3. What is stage R latency (RL)?

Answer questions 4 to 8 using the information below.


Total recording time (TRT) = 400 minutes
Stage N1 = 80 epochs
Stage N2 = 400 epochs
Stage N3 = 20 minutes
Stage R = 100 minutes
4. What is the percentage of total sleep time (%TST) of N1?

5. What is the %TST of N2?

6. What is the %TST of N3?

7. What is the %TST of stage R?

8. What is the sleep efficiency (SE)?

For questions 9 to 10, use the following information.


Sleep efficiency (SE) = 80%
Total recording time (TRT) = 400 minutes
Obstructive apneas (OA) = 40
Mixed apneas (MA) = 10
Hypopneas (H) = 100
9. Calculate the total sleep time (TST).

10. Calculate the apnea–hypopnea index (AHI).

Questions 11 to 20 are based on the following information.


Stage N1 sleep = 30 minutes
Stage N2 sleep = 200 minutes
Stage N3 sleep = 86 minutes
Stage R = 80 minutes
Sleep efficiency = 90%
Periodic limb movements (PLM) = 200
PLM with arousals = 80
Obstructive apneas (OA) = 60
Mixed apneas (MA) = 20
Central apnea (CA) = 10
Hypopneas (H) = 90
11. What is the total recording time (TRT)?

12. What is the periodic limb movement index (PLMI)?

13. What is the periodic limb movement with arousal index (PLMArI)?

14. What is the apnea index (AI)?

15. What is the hypopnea index (HI)?

16. What is the apnea–hypopnea index (AHI)?

17. What is the percentage of total sleep time (%TST) of N1 sleep?

18. What is the %TST of N2 sleep?

19. What is the %TST of N3 sleep?

20. What is the %TST of stage R sleep?

Given the following information, answer questions 21 to 22.


Apneas = 100
Hypopneas = 200
AHI = 50 events per hour
Sleep efficiency (SE) = 75%
21. What is the total sleep time (TST)?

22. What is the total recording time (TRT)?

For questions 23 to 28, fill the blank cells in the grid below.
29. What is the arousal index (ArI) if there are 60 arousals recorded during 6
hours of total sleep time (TST)?

Questions 30 to 31 refer to the following information.


Lights off: epoch 24 (stage W)
Epoch 25: stage W
Epoch 26: stage W
Epoch 27: stage R
30. What is the sleep onset latency (SOL)?

31. What is stage R latency (RL)?

32. Oxygen desaturation index (ODI) is defined as the number of qualifying
oxygen desaturation episodes per ______________ of sleep.

33. During a recording, oxygen saturation nadir is the:
A. Longest desaturation episode
B. Highest saturation level
C. Average saturation level
D. Lowest saturation level
34. Total recording time (TRT) minus total wake time (TWT) =
______________.

35. Total time in N1 + total time in N2 + total time in N3 = ______________.

36. Total time in N1 + total time in N2 + total time in N3 + total time in stage R
= ______________.

For questions 37 to 40, consider the information presented in Table 27-1.

Table 27-1
37. What is the supine apnea–hypopnea index (AHI)?

38. What is the left-side AHI?

39. What is the right-side AHI?

40. What is the overall AHI?

For questions 41 to 43, consider the information in Table 27-2.

Table 27-2

41. What is the stage R apnea–hypopnea index (AHI)?



42. What is the NREM AHI?

43. What is the total AHI?

ANSWERS

1. TRT = 458 minutes. In this question, you are given the “lights off” epoch and
“lights on” epoch. To determine TRT, subtract the “lights off” epoch from the
“lights on” epoch. Divide the result by 2 to convert the answer from epochs
to minutes. In this question, lights on is at epoch 931 and lights off at epoch
15. Thus, 931 − 15 = 916 epochs, and 916 epochs/2 epochs per minute = 458
minutes RT.
See Fundamentals of Sleep Technology, 2nd edition, Chapter 33, page 402.

2. SOL = 7.5 minutes. Sleep onset latency is the time between the “lights off”
epoch and the first epoch of sleep. To determine SOL, subtract the “lights
off” epoch from the sleep onset epoch, and divide the result by 2 to convert
from epochs to minutes. Sleep onset is at epoch 30 and lights off at epoch 15.
Subtracting sleep onset from lights off, 30 − 15 = 15 epochs, and 15 epochs/2
epochs per minute = 7.5 minutes SOL.
See Fundamentals of Sleep Technology, 2nd edition, Chapter 33, page 402.

3. RL = 6 minutes. Stage R latency is defined as the time between sleep onset


and the onset of stage R sleep. To determine RL, subtract the stage R onset
epoch from the sleep onset epoch, and divide the result by 2 to convert from
epochs to minutes. Stage R onset is at epoch 42 and sleep onset at epoch 30.
Therefore, 42 − 30 = 12 epochs, and 12 epochs/2 epochs/minute = 6 minutes
RL.
See Fundamentals of Sleep Technology, 2nd edition, Chapter 33, page 402.

QUESTIONS 4 TO 8.
To answer these questions, first determine total sleep time (TST). For N1, 80
epochs/2 epochs per minute = 40 minutes. For N2, 400 epochs/2 epochs per
minute = 200 minutes. N3 is 20 minutes, and stage R is 100 minutes. Thus, TST
= 40 + 200 + 20 + 100 = 360.
See Fundamentals of Sleep Technology, 2nd edition, Chapter 33, page 402.

4. 11.1%. %N1 = 100 × (Time N1/TST) = 100 × (40 min N1/360 min TST) =
11.1%

5. 55.6%. %N2 = 100 × (Time N2/TST) = 100 × 200 min N1/360 min TST) =
55.6%

6. 5.6%. %N3 = 100 × (Time N1/TST) = 100 × (20 min N3/360 min TST) =
5.6%

7. 27.8%. %Stage R = 100 × (Time Stage R/TST) = 100 × (100 min N3/360 min
TST) = 27.8%

8. SE = 90%. Sleep efficiency is the total sleep time (TST) compared to total
recording time (TRT). In this question, 100 × TST/TRT = 100 (360 TST/400
TRT) = 90%.

9. TST = 320 minutes. Sleep efficiency (SE) = total sleep time (TST)/total
recording time (TRT). Therefore, substituting the known values gives 80% =
TST/400 minutes. Multiply both sides of the equation by 400 minutes (400
min × 80% = TST × 400 minutes/400 minutes). Hence, TST = 400 min ×
80% = 320 minutes.
See Fundamentals of Sleep Technology, 2nd edition, Chapter 33, page 402.

10. AHI = 28.1 events/hour. The apnea–hypopnea index is the sum of apneas and
hypopneas per hour of sleep time. Since 60 minutes = 1 hour, substituting
known values gives an AHI = 60 × (40 + 10 + 100)/320 = 28.1 events per
hour.
See Fundamentals of Sleep Technology, 2nd edition, Chapter 33, page 402.

11. TRT = 440 minutes. Total recording time (TRT) can be calculated from sleep
efficiency (SE) and total sleep time (TST). In this question, SE is 90%, and
TST = N1 + N2 + N3 + Stage R = 30 + 200 + 86 + 80 = 396 minutes.
Substituting values into the formula gives 90% = 396/TRT. Multiply both
sides by TRT to get 90% × TRT = 396 minutes. Divide both sides by 90% to
get TRT = 396 min/90% = 440 minutes.
See Fundamentals of Sleep Technology, 2nd edition, Chapter 33, page 402.

12. PLMI = 30.3 events/hour. The calculation is made by dividing the number of
events by the TST in hours (or in minutes and multiplying the result by 60).
Hence, PLMI = 60 min/hour × 200 events/396 min = 30.3 events per hour.
See Fundamentals of Sleep Technology, 2nd edition, Chapter 33, page 407.

13. PLMArI = 12.1 events/hour. Dividing the number of events by the TST in
hours gives a PLMArI = 60 min/hour × 80 events/396 min = 12.1 events per
hour.
See Fundamentals of Sleep Technology, 2nd edition, Chapter 33, page 407.

14. AI = 13.6 events/hour. Add all the apneas and divide by total sleep time
(TST) in minutes. Then, multiply the result by 60. For this question, AI = 60
min/hour × (60 + 20 + 10)/396 min = (60 × 90)/396 = 13.6 events/hour.
See Fundamentals of Sleep Technology, 2nd edition, Chapter 33, page 402.

15. HI = 13.6 events/hour. Total the hypopneas and divide by total sleep time
(TST). Multiply the result by 60 minutes per hour. Thus, HI = 60 min/hour ×
(90)/396 min = (60 × 90)/396 = 13.6 events/hour.
See Fundamentals of Sleep Technology, 2nd edition, Chapter 33, page 402.

16. AHI = 27.3 events/hour. Total apneas and hypopneas divide by TST and
multiply the result by 60 minutes per hour.
60 min/hour × (60 + 20 + 10 + 90)/396 minutes = (60 × 180)/396 = 27.3
events/hour See Fundamentals of Sleep Technology, 2nd edition, Chapter
33, page 402.

17. %N1 = 7.6%. Minutes of N1 sleep divided by total sleep time (TST) in
minutes × 100 to give percentage. Therefore, %N1 = 100 × 30/396 = 7.6%.
See Fundamentals of Sleep Technology, 2nd edition, Chapter 33, page 402.

18. %N2 = 50.5%. Minutes of N2 sleep divided by total sleep time (TST) in
minutes × 100 to give percentage. In this question, %N2 = 100 × 200/396 =
50.5%.
See Fundamentals of Sleep Technology, 2nd edition, Chapter 33, page 402.

19. %N3 = 21.7%. Minutes of N3 sleep divided by total sleep time (TST) in
minutes × 100 to give percentage. Thus, %N3 = 100 × 86/396 = 21.7%.
See Fundamentals of Sleep Technology, 2nd edition, Chapter 33, page 402.

20. %Stage R = 20.2%. Minutes of stage R sleep divided by total sleep time
(TST) in minutes × 100 to give percentage. Substituting, %Stage R = 100 ×
200/396 = 20.2%.
See Fundamentals of Sleep Technology, 2nd edition, Chapter 33, page 402.

21. TST = 6 hours. If there are 300 total apneas and hypopneas and the apnea–
hypopnea index (AHI) is 50 events per hour, total sleep time (TST) can be
calculated by dividing 300 events by 50 events per hour to yield 6 hours of
TST. Thus, AHI = events/hours of sleep. Substituting known values, 50
events/hour of sleep = (100 + 200)/TST. Thus, 50 = 300/TST. Multiplying
both sides of the equation by TST gives TST × 50 = TST × 300/TST. Then,
divide both sides by 50, leaving TST × 50/50 = TST × 300/TST × 50. Finally,
TST = 300/50 = 6 hours.
See Fundamentals of Sleep Technology, 2nd edition, Chapter 33, page 402.

22. TRT = 8 hours. To calculate:


1. Sleep efficiency (SE) is 75%, and total sleep time is 6 hrs.
2. Substituting in the formula for SE (SE = TST/TRT), 75% = 6 hour/total
recording time (TRT) 3. 75% × TRT = TRT × 6 hours/TRT (multiply both
sides by TRT) 4. 75% × TRT = 6 hours
5. 75% × TRT 75% = 6 hours /75% (divide both sides by 75%) 6. Thus, TRT
= 6 hour/75% = 8 hours.
See Fundamentals of Sleep Technology, 2nd edition, Chapter 33, page 402.

23. 80 events/hour. Apnea–hypopnea index (AHI) = 60 × events/sleep in minutes


= 60 × (20/15) = 80 events per hour.
See Fundamentals of Sleep Technology, 2nd edition, Chapter 33, page 402.

24. 15 events. To calculate:


1. Apnea–hypopnea index (AHI) = (60 × events)/minute of sleep; 45 = (60 ×
events)/20 minutes 2. 20 × 45 = 60 × 20 × events/20 (multiply both sides
by 20) 3. 900 = 60 × events
4. 900/60 = 60 × events/60 (divide both sides by 60) = 15 events.
See Fundamentals of Sleep Technology, 2nd edition, Chapter 33, page 402.

25. 20 minutes. To calculate:


1. Apnea–hypopnea index (AHI) = 60 × events/total sleep time (TST) in
minutes 2. 30 = 60 × 10/TST in minutes
3. TST × 30 = TST × 60 × 10/TST (multiply both sides by TST) 4. TST × 30
= 60 × 10 (cancel TST)
5. TST = 600/30 (divide both sides by 30) = 20 minutes.
See Fundamentals of Sleep Technology, 2nd edition, Chapter 33, page 402.

26. 30 minutes. Apnea–hypopnea index (AHI) = (60 × events)/total sleep time


(TST) in minutes. Substituting, 20 = (60 × 10)/TST in minutes. Multiplying
both sides by TST, TST × 20 = TST × 60 × 10/TST = TST × 20 = 60 × 10.
Dividing both sides by 20, TST = 600/20 = 30 minutes.
See Fundamentals of Sleep Technology, 2nd edition, Chapter 33, page 402.

27. 8 events/hour. (60 minutes per hour × 6 events)/45 min = 8 events per hour.
See Fundamentals of Sleep Technology, 2nd edition, Chapter 33, page 402.

28. 2.4 events/hour. (60 minutes per hour × 2 events)/50 minutes = 2.4 events per
hour.
See Fundamentals of Sleep Technology, 2nd edition, Chapter 33, page 402.

29. 10 events/hour. In this question, TST is already presented in hours, and,


therefore, there is no need to convert from minutes. Substituting, number of
events/TST = 60/6 = 10 events per hour.
See Fundamentals of Sleep Technology, 2nd edition, Chapter 33, page 402.

30. SOL = 1.5 minutes. Sleep onset latency = sleep onset epoch – lights off
epoch/2 = (27 – 24)/2 = 3/2 = 1.5 minutes.
See Fundamentals of Sleep Technology, 2nd edition, Chapter 33, page 402.

31. RL = 0 minutes. Stage R latency = stage R onset epoch – sleep onset epoch/2
= (27 – 27)/2 = 0 minutes.
See Fundamentals of Sleep Technology, 2nd edition, Chapter 33, page 402.

32. Hour.
See Fundamentals of Sleep Technology, 2nd edition, Chapter 33, page 402.

33. D, Lowest saturation level.


See Fundamentals of Sleep Technology, 2nd edition, Chapter 33, page 402.

34. Total sleep time (TST).


See Fundamentals of Sleep Technology, 2nd edition, Chapter 33, page 402.

35. Total time NREM.


See Fundamentals of Sleep Technology, 2nd edition, Chapter 33, page 402.

36. Total sleep time (TST).


See Fundamentals of Sleep Technology, 2nd edition, Chapter 33, page 402.

37. Supine AHI = 60 events/hour. Supine AHI = 60 × (supine apneas + supine


hypopneas)/supine sleep time in minutes = 60 × (80 + 100)/180 = 60
events/hour.
See Fundamentals of Sleep Technology, 2nd edition, Chapter 33, page 402.

38. Left-side AHI = 25 events/hour. Left-side AHI = 60 × (left-side apneas +


left-side hypopneas)/left-side sleep time in minutes = 60 × (20 + 30)/120 = 25
events/hour.
See Fundamentals of Sleep Technology, 2nd edition, Chapter 33, page 402.

39. Right-side AHI = 20 events/hour. Right-side AHI = 60 × (right-side apneas


+ right-side hypopneas)/right-side sleep time in minutes. Hence, right-side
AHI = 60 × (10 + 10)/60 = 20 events/hour.
See Fundamentals of Sleep Technology, 2nd edition, Chapter 33, page 402.

40. AHI = 41.7 events/hour. AHI = 60 × (apneas + hypopneas)/sleep time in


minutes. Or AHI = 60 × (80 + 20 + 10 + 100 + 30 + 10)/(180 + 120 + 60) =
60 × 250/360 = 41.7 events/hour.
See Fundamentals of Sleep Technology, 2nd edition, Chapter 33, page 402.

41. Stage R AHI = 80 events/hour. Stage R AHI = 60 × (stage R apneas + stage


R hypopneas)/stage R sleep time in minutes. Therefore, stage R AHI = 60 ×
(80 + 40)/(90) = 60 × 120/90 = 80 events/hour.
See Fundamentals of Sleep Technology, 2nd edition, Chapter 33, page 402.

42. NREM AHI = 40 events/hour. NREM AHI = 60 × (NREM apneas + NREM


hypopneas)/NREM sleep time in minutes. For this question, NREM AHI = 60
× (120 + 60)/(270) = 60 × 180/270 = 40 events/hour.
See Fundamentals of Sleep Technology, 2nd edition, Chapter 33, page 402.

43. AHI = 50 events/hour. AHI = 60 × (apneas + hypopneas)/sleep time in


minutes. Substituting, AHI = 60 × (80 + 40 + 120 + 60)/(90 + 270) = 60 ×
300/360 = 50 events/hour.
See Fundamentals of Sleep Technology, 2nd edition, Chapter 33, page 402.
SECTION V
Interventions and Therapeutics
CHAPTER
28
Titration of Continuous Positive Airway
Pressure and Application and
Adjustment of Positive Airway Pressure
Devices
JOYCE BLACK

NOTE: This chapter corresponds to Chapter 34 in Fundamentals of Sleep


Technology, 2nd edition.

1. An American Academy of Sleep Medicine (AASM) literature review found


that appropriate settings for positive airway pressure (PAP) therapy can be
determined with mathematical models that use height, weight, neck size,
airway size, or other measurements. True or false?

2. It would be inappropriate to use a full-face mask:


A. With every patient
B. When the patient is a mouth breather
C. When the patient states he or she is a mouth breather
D. When the patient does not have teeth

3. According to the American Academy of Sleep Medicine (AASM)


Guidelines, an optimal positive airway pressure (PAP) titration occurs when
the selected setting keeps the Respiratory Disturbance Index (RDI) below
______________ for at least ______________ minutes.
A. 3, 10
B. 4, 12
C. 5, 15
D. 10, 20
4. When performing a positive airway pressure (PAP) titration on a pediatric
patient younger than 12 years of age, the PAP setting should be increased
when at least ______________ obstructive apnea(s) or hypopnea(s) are
seen.
A. 1
B. 2
C. 3
D. 4

5. It is possible to determine the proper starting size for a given mask using
measurements of the nose, but there is no way to predict the best mask type
for a patient. True or false?

6. The primary goals of positive airway pressure (PAP) titration include all of
the following EXCEPT:
A. Elimination of apneas
B. Elimination of hypopneas
C. Elimination of respiratory event–related arousals (RERAs)
D. Elimination of periodic limb movements

7. The basic American Academy of Sleep Medicine (AASM) recommendation


for starting the titration portion of a split-night study is after a diagnostic
study of:
A. 1 hour with an apnea–hypopnea index (AHI) of 20
B. 2 hours with an AHI of 30
C. 2 hours with an AHI of 40
D. None of the above

8. An inadequate PAP titration is defined by the American Academy of Sleep


Medicine (AASM) Guidelines as ≥______________ obstructive apneas, or
≥______________ hypopneas, or ≥______________ respiratory event–
related arousals (RERAs) or ≥3 minutes of loud snoring in adults.
A. 2, 3, 2
B. 1, 2, 3
C. 3, 5, 7
D. 2, 3, 5

9. The American Academy of Sleep Medicine (AASM) Guidelines for


continuous positive airway pressure (CPAP) therapy for obstructive sleep
apnea recommend a starting pressure of ______________ cm H2O with a
maximum pressure of ______________cm H2O for adults.
A. 4, 15
B. 4, 20
C. 5, 15
D. 5, 20

10. BPAP is useful for patients who cannot tolerate continuous positive airway
pressure (CPAP) or who complain of trouble exhaling during CPAP therapy.
True or false?

11. What is the most effective method for improving adherence to therapy?
A. Changing the mask
B. Decreasing pressure
C. Changing to bilevel positive airway pressure (BPAP)
D. Early intervention to address patient complaints

12. Continuous positive airway pressures (CPAPs) that are too high may cause:
A. Excessive sneezing
B. Arousals and unwanted changes in respiratory patterns
C. Increase in oxygen saturation
D. More obstructive apneas

13. In a pediatric patient younger than 12 years of age, the continuous positive
airway pressure (CPAP) can be increased by 1 cm H2O if unambiguous
snoring is observed for at least 1 minute. True or false?

14. It is the responsibility of a sleep technologist performing a titration study to


be sure that the study has no artifact. In addition, the technologist is also
responsible for:
A. Assisting the patient for the best outcome
B. Acting as the eyes and ears of the physician and recording and
commenting on important events that occur during the night
C. Ensuring the best pressure is identified for the best outcome for the
patient
D. All of the above
15. A respiratory event–related arousal (RERA) could be scored if there is a 10-
second period of signal flattening followed by an arousal or if there is
increased effort measured by inductance plethysmography. True or false?

16. The American Academy of Sleep Medicine (AASM) Guidelines recommend


waiting ______________ between changes in positive airway pressure
(PAP) settings.
A. A minimum of 2 minutes
B. A minimum of 5 minutes
C. A minimum of 10 minutes
D. Until five respiratory disturbances are seen

17. In changing from continuous (CPAP) to bilevel (BPAP) positive airway


pressure, the expiratory positive airway pressure (EPAP) should be set at the
pressure that resolved the obstructive events and the inspiratory positive
airway pressure (IPAP) should be at least ______________ cm H2O higher.
A. 3
B. 4
C. 5
D. 6

18. According to the American Academy of Sleep Medicine (AASM) Guidelines


for an adequate PAP titration, the respiratory disturbance index (RDI) may
be above 10 events per hour if it is reduced by 75% from baseline. True or
false?

19. Auto-titrating positive airway pressure (APAP) therapy is indicated for all of
the following EXCEPT:
A. Titration during attended studies
B. Patients with obstructive sleep apnea and severe chronic obstructive
pulmonary disease (COPD)
C. Patients requiring changes in pressure
D. To determine a fixed pressure

20. Ideally, patients should be recorded in a supine position during REM sleep
for at least ______________ minutes at the designated optimal positive
airway pressure (PAP) setting.
A. 5
B. 10
C. 15
D. 20

21. The AASM Positive Airway Pressure Titration Guidelines provide a critical
pathway that:
A. Should be used in all patients at all times regardless of patient response
to treatment
B. Is entirely evidence-based and required to be followed for center
accreditation
C. Is a recommended protocol that supplements technologist judgment
during titration
D. Is documented to be effective in 95% of patients studied

SCENARIO
Amy, a new sleep technologist, is preparing for her patient, Mr. Johnson, who
has severe OSA. Mr. Johnson's AHI is 75, and his oxygen saturation nadir is
69% during his polysomnography. His sleep efficiency is 37%, but he goes into
REM sleep for 15 minutes near the end of the night. On his morning
questionnaire, Mr. Johnson answers that he thought the night was pretty close to
his regular sleep pattern although it might have taken a little longer than usual
to fall asleep.

22. What conclusions can Amy make with this information? Provide
explanations for why or why not.
A. Mr. Johnson is so tired he will be easy to set up on continuous positive
airway pressure (CPAP).
B. He needs very high CPAP pressures and might possibly need BPAP.
C. He has COPD or another medical disorder that is causing his low oxygen
saturation.
D. Once his optimal pressure is reached, he will sleep all night and have the
best sleep in a long time.
E. He needs a full-face mask.

23. Place the following items in the correct order: i. Mr. Johnson is titrated to an
optimal pressure before leaving for home.
ii. Mr. Johnson is fitted with a mask that he helped to select.
iii. CPAP is increased to 11 cm H2O, due to one apnea and one hypopnea
each.
iv. Mr. Johnson is educated via a video on the diagnosis and treatment of
obstructive sleep apnea (OSA).
v. CPAP is started at 4 cm H2O and increased to 10 cm H2O.
A. iii, v, ii, i, iv
B. ii, iii, iv, v, i
C. v, ii, iv, i, iii
D. iv, ii, v, iii, i

24. Amy greets Mr. Johnson and they discuss his previous night in the sleep
center. Mr. Johnson is very positive about the treatment. He reports that his
doctor called to tell him how severe his sleep apnea is and informed him that
it will really help him if he uses CPAP. Amy gets out the three masks she
usually offers to patients and asks Mr. Johnson to see which among them he
feels is the most comfortable. This is a good policy to start getting a patient
adherent to therapy. True or false?

25. Amy increases the CPAP setting up to 11 cm H2O. Mr. Johnson wakes up to
go to the bathroom and when he returns, comments that he cannot tolerate
the CPAP pressure. What should Amy do? (Select all that apply.)
A. Tell Mr. Johnson to relax, as it will get easier to use CPAP with time.
B. Decrease the pressure to 4 or 5 cm H2O (per lab protocol).
C. Switch to bilevel positive airway pressure (BPAP) therapy.
D. Stop the study immediately.

26. Which of the following is a valid reason for decreasing CPAP pressure?
A. The patient cannot tolerate the higher pressure.
B. The patient is snoring loudly.
C. The patient is doing well, and the technologist simply wants to try
another CPAP setting.
D. Paradoxical effort is seen in the chest and abdominal channels.

27. Mr. Johnson has many hypopneas and respiratory event–related arousals
(RERAs) and continues to snore at a CPAP setting of 16 cm H2O. He wakes
up again and describes that he cannot tolerate the high pressures. He asks
Amy to stop the sleep study. What should the technologist do? Provide an
explanation.
A. Decrease the pressure again and reassure him.
B. Switch to BPAP.
C. Stop the study and let Mr. Johnson go home.
D. Call the on-call sleep physician.

28. Amy decides to switch to BPAP. What should the initial BPAP settings be if
the current CPAP setting is 8 cm H2O?
A. IPAP 8 cm H2O, EPAP 4 cm H2O
B. IPAP 10 cm H2O, EPAP 6 cm H2O
C. IPAP 12 cm H2O, EPAP 8 cm H2O
D. IPAP 14 cm H2O, EPAP 10 cm H2O

29. After starting BPAP therapy, Amy immediately notices two obstructive
apneas. What should she do next?
A. Increase IPAP only
B. Increase EPAP only
C. Increase both IPAP and EPAP
D. Continue monitoring the patient

30. Hypopneas and periods of 4% oxygen desaturation are present. What should
be done?
A. Increase IPAP only.
B. Increase EPAP only.
C. Increase both IPAP and EPAP.
D. Continue monitoring the patient.

31. Snoring and RERAs continue for 5 minutes. What should Amy do?
A. Increase IPAP only
B. Increase EPAP only
C. Increase both IPAP and EPAP
D. Continue monitoring the patient

ANSWERS
1. False.
See Fundamentals of Sleep Technology, 2nd edition, Chapter 34, page 411.

2. A, With every patient


See Fundamentals of Sleep Technology, 2nd edition, Chapter 34, page 413.

3. C, 5, 15.
See Fundamentals of Sleep Technology, 2nd edition, Chapter 34, page 414.

4. A, 1.
See Fundamentals of Sleep Technology, 2nd edition, Chapter 34, page 416.

5. True.
See Fundamentals of Sleep Technology, 2nd edition, Chapter 34, page 413.

6. D, Elimination of periodic limb movements.


See Fundamentals of Sleep Technology, 2nd edition, Chapter 34, page 414.

7. C, 2 hours with an AHI of 40.


See Fundamentals of Sleep Technology, 2nd edition, Chapter 34, page 414.

8. D, 2, 3, 5.
See Fundamentals of Sleep Technology, 2nd edition, Chapter 34, page 414.

9. B, 4, 20.
See Fundamentals of Sleep Technology, 2nd edition, Chapter 34, page 414.

10. True.
See Fundamentals of Sleep Technology, 2nd edition, Chapter 34, page 416.

11. D, Early intervention to address patient complaints.


See Fundamentals of Sleep Technology, 2nd edition, Chapter 34, page 420.

12. B, Arousals and unwanted changes in respiratory patterns.


See Fundamentals of Sleep Technology, 2nd edition, Chapter 34, page 417.

13. True.
See Fundamentals of Sleep Technology, 2nd edition, Chapter 34, page 416.

14. D, All of the above.


See Fundamentals of Sleep Technology, 2nd edition, Chapter 34, page 411.
15. True.
See Fundamentals of Sleep Technology, 2nd edition, Chapter 34, page 414.

16. B, A minimum of 5 minutes.


See Fundamentals of Sleep Technology, 2nd edition, Chapter 34, page 414.

17. B, 4.
See Fundamentals of Sleep Technology, 2nd edition, Chapter 34, page 416.

18. True.
See Fundamentals of Sleep Technology, 2nd edition, Chapter 34, page 414.

19. B, Patients with obstructive sleep apnea and chronic obstructive pulmonary
disease (COPD).

20. C, 15.
See Fundamentals of Sleep Technology, 2nd edition, Chapter 34, page 414.

21. C, Is a recommended protocol that supplements technologist judgment during


titration See Fundamentals of Sleep Technology, 2nd edition, Chapter 34,
page 414.

22. These answers use much of the chapter as a resource. The page numbers are
starting points.
A. Mr. Johnson is so tired he will be easy to set up on continuous positive
airway pressure (CPAP). This may not be true as Mr. Johnson may be
resistant to the chosen mask or have difficulty with the pressures. See
Fundamentals of Sleep Technology, 2nd edition, Chapter 34, page 414.
B. He needs very high CPAP pressures and might possibly need BPAP. This
also may be incorrect since assumptions regarding the pressure needed
for a patient cannot be made from the RDI, neck size, weight, or height
alone. See Fundamentals of Sleep Technology, 2nd edition, Chapter 34,
page 411.
C. He has COPD or another medical disorder that is causing his low oxygen
saturation. This may be true, but many patients can have very low
oxygen levels without any underlying lung problems. See Fundamentals
of Sleep Technology, 2nd edition, Chapter 34, pages 411 and 419.
D. Once his optimal pressure is reached, he will sleep all night and have the
best sleep in a long time. While some patients report feeling substantially
better after one night CPAP therapy, others may require several nights of
CPAP use to detect any improvements in sleep quality and daytime
symptoms. See Fundamentals of Sleep Technology, 2nd edition, Chapter
34, pages 413–414.
E. He needs a full-face mask. Not true. Full-face masks should only be used
on patients who are mouth breathers. See Fundamentals of Sleep
Technology, 2nd edition, Chapter 34, page 419.

23. D. iv, ii, v, iii, i.


See Fundamentals of Sleep Technology, 2nd edition, Chapter 34, page 412.

24. True.
See Fundamentals of Sleep Technology, 2nd edition, Chapter 34, page 413.

25. A. Tell Mr. Johnson to relax as it will get easier to use CPAP with time; and B.
Decrease the pressure to 4 or 5 cm H2O (per lab protocol).
See Fundamentals of Sleep Technology, 2nd edition, Chapter 34, pages 417–418.

26. A, The patient cannot tolerate the higher pressure.


See Fundamentals of Sleep Technology, 2nd edition, Chapter 34, page 417.

27. B. Switch to BPAP.


See Fundamentals of Sleep Technology, 2nd edition, Chapter 34, page 416.

28. C. IPAP 12 cm H2O, EPAP 8 cm H2O.


See Fundamentals of Sleep Technology, 2nd edition, Chapter 34, page 416.

29. C. Increase both IPAP and EPAP.


See Fundamentals of Sleep Technology, 2nd edition, Chapter 34, page 416.

30. A. Increase IPAP only.


See Fundamentals of Sleep Technology, 2nd edition, Chapter 34, page 416.

31. A. Increase IPAP only.


See Fundamentals of Sleep Technology, 2nd edition, Chapter 34, page 416.
CHAPTER
29
Developing and Maintaining
Therapeutic Compliance
JOYCE BLACK

NOTE: This chapter corresponds to Chapter 35 in Fundamentals of Sleep


Technology, 2nd edition.

1. Patient education is best described as the process that evolves from:


A. A physician telling a patient what to do
B. A team of health professionals telling a patient what to do
C. An exchange between health professionals and patients
D. An afternoon searching health-related Internet Web sites

2. The educational plan must include:


A. Results of the sleep study
B. Specific goals and outcomes
C. The type of device the patient will use
D. A schedule for payment for treatment devices

3. Asking the patient “What can you tell me about your condition?” is
important and allows the technologist to assess the patient's:
A. Basic knowledge and level of understanding
B. Family resources available to support treatment
C. Severity of sleep disorder
D. Motivation for improvement and treatment adherence

4. Asking the patient “What are your expectations?” is important and allows the
technologist to:
A. Predict future compliance with treatment
B. Remind the patient to expect minimal improvement with his or her
disorder
C. Set realistic goals
D. Limit the discussion to what the patient expects to discuss

5. Social factors that influence patient learning and adherence to treatment


include religious beliefs, cultural values, employers, and:
A. Medications
B. Preexisting medical conditions
C. Family
D. A ramp option on the patient's PAP device

6. Patient educational opportunities:


A. Begin when the patient receives his or her PAP device
B. Are limited to the clinic visits
C. Are available only when the physician is available
D. Begin with the initial clinic visit

7. Information overload may occur during the sleep test or provision of durable
medical equipment (DME) such as PAP equipment. A good way to begin to
alleviate the overload is to:
A. Motivate the patient by instilling a sense of fear if the device is not used
B. Speak loudly so the patient knows the information is important
C. Ask the patient to look at the technologist when the technologist is
speaking
D. Provide written material for the patient to review when the patient gets
home

8. Studies have shown that the most critical period for long-term treatment
adherence is:
A. The first 24 to 72 hours after initiation of treatment
B. The first 3 months of treatment
C. The “7-month itch”
D. After the patient has discontinued treatment for a week or more

9. The most frequently used definition of PAP compliance is:


A. At least 2 hours per night on 50% of nights
B. At least 4 hours per night on 70% of nights
C. At least 7 hours per night on 90% of nights
D. All night every night
10. A “true” compliance meter on a CPAP device:
A. Is not really necessary
B. Measures how long the machine has been running
C. Measures the time the patient breathes on the machine
D. Measures patient sleep time

11. The Functional Outcomes of Sleep Questionnaire (FOSQ) is a clinically


validated tool that measures a patient's:
A. Self-perception of sleepiness
B. Quality of life
C. Likelihood of obstructive sleep apnea
D. Cognitive functioning

12. The first step in addressing the patient who complains that the mask does not
fit well is to:
A. Adjust the straps and pads and ensure that they are not overtightened
B. Educate the patient in the use of the ramp feature
C. Tighten the straps to be sure that there are no instances of air leakage
D. Increase PAP pressure to ensure that the mask is maximally inflated

13. Patients who complain of claustrophobia or a feeling of suffocation may


benefit from:
A. A period of holding an unpressurized mask to the face while awake
B. Adaptive servo ventilation delivered with a full-face mask
C. Wearing earplugs during PAP use
D. An alarm that wakes the patient if the patient removes the PAP mask

14. Mouth breathing is a particular problem for PAP users with nasal masks
because it results in:
A. A significant drop in pressure
B. Stretching of the PAP straps
C. Cardiac arrhythmias
D. Inability to talk during PAP therapy

15. A common complaint from bed partners of patients starting on PAP therapy
is that:
A. They miss the snoring
B. The machine is too noisy
C. Movement is restricted
D. The patient frequently moves around in bed

16. When travelling, PAP therapy settings may not be adequate if the patient:
A. Is travelling overseas
B. Is travelling to warmer climates
C. Uses a humidifier at home
D. Travels to elevations over 5,000 ft.

17. It is recommended that the PAP device be placed:


A. Slightly above the patient's head
B. Within 6 inches of the bed
C. On the floor next to the patient's bed on a mat or thick carpet
D. Between 6 inches above the floor and the level of the patient's head

DISCUSSION QUESTIONS

18. Group sessions can be as effective in promoting PAP adherence as individual


sessions. Discuss the merits of group sessions in terms of patient outcomes
and financial impact.

19. Is an educational program sufficient to produce adequate PAP adherence?


Discuss elements of a comprehensive program to improve outcomes for
patients with OSA.

ANSWERS

1. C, An exchange between health professionals and patients.


See Fundamentals of Sleep Technology, 2nd edition, Chapter 35, page 421.

2. B, Specific goals and outcomes.


See Fundamentals of Sleep Technology, 2nd edition, Chapter 35, page 422.

3. A, Basic knowledge and level of understanding.


See Fundamentals of Sleep Technology, 2nd edition, Chapter 35, page 424.

4. C, Set realistic goals.


See Fundamentals of Sleep Technology, 2nd edition, Chapter 35, page 424.

5. C, Family.
See Fundamentals of Sleep Technology, 2nd edition, Chapter 35, page 425.

6. D, Begin with the initial clinic visit.


See Fundamentals of Sleep Technology, 2nd edition, Chapter 35, page 425.

7. D, Provide written material for the patient to review when the patient gets
home.
See Fundamentals of Sleep Technology, 2nd edition, Chapter 35, page 426.

8. A, The first 24 to 72 hours after initiation of treatment.


See Fundamentals of Sleep Technology, 2nd edition, Chapter 35, page 426.

9. B, At least 4 hours per night on 70% of nights.


See Fundamentals of Sleep Technology, 2nd edition, Chapter 35, page 426.

10. C, Measures the time the patient breathes on the machine.


See Fundamentals of Sleep Technology, 2nd edition, Chapter 35, page 428.

11. B, Quality of life.


See Fundamentals of Sleep Technology, 2nd edition, Chapter 35, page 428.

12. A, Adjust the straps and pads and ensure that they are not overtightened.
See Fundamentals of Sleep Technology, 2nd edition, Chapter 35, page 429.

13. A, A period of holding an unpressurized mask to the face while awake.


See Fundamentals of Sleep Technology, 2nd edition, Chapter 35, page 430.

14. A, A significant drop in pressure.


See Fundamentals of Sleep Technology, 2nd edition, Chapter 35, page 430.

15. B, The machine is too noisy.


See Fundamentals of Sleep Technology, 2nd edition, Chapter 35, page 431.

16. D, Travels to elevations over 5,000 ft.


See Fundamentals of Sleep Technology, 2nd edition, Chapter 35, page 432.

17. D, Between 6 inches above the floor and the level of the patient's head.
See Fundamentals of Sleep Technology, 2nd edition, Chapter 35, page 433.
18. “Despite less time allocated per patient, we found that a group educational
strategy resulted in improved acceptance of and adherence to CPAP therapy.
Using a group educational setting yielded a three- to fourfold increase in the
number of patients per unit of time. Further, we did not observe any detriment
in subsequent adherence. In fact, objective measures of CPAP use, rates of
regular use of CPAP, and discontinuation of therapy were all improved. In
addition, group clinics decreased the time between polysomnography and
initiation of therapy.” (Lettieri CJ, Walter RJ. Impact of group education on
continuous positive airway pressure adherence. J Clin Sleep Med.
2013;9(6):537–541.) See Fundamentals of Sleep Technology, 2nd edition,
Chapter 35, page 428.

19. “Although these data are promising, don't start celebrating yet. A close
examination of the results suggests that CPAP nonadherence remains a
problem. Unfortunately, neither study achieved levels of adherence that
would restore normal functioning. The mean use in both studies was less than
6 hour per night, leaving an unprotected airway the remainder of the sleep
period. Moreover, what remains unclear is the underlying mechanism that
promoted adherence. Both studies indicate that their interventions were based
on the need for education and improved self-efficacy. However, only one of
the studies systematically measured self-efficacy and did not find statistically
reliable differences between intervention groups.” (Weaver TE. Don't start
celebrating—CPAP adherence remains a problem. J Clin Sleep Med.
2013;9(6):551–552.)
CHAPTER
30
Oxygen Administration in the Sleep
Center
STEVEN H. LENIK

NOTE: This chapter corresponds to Chapter 36 in Fundamentals of Sleep


Technology, 2nd edition.

OXYGEN STORAGE AND DISTRIBUTION

1. What is the primary source of oxygen available to you in a freestanding sleep


center?
A. Compressed oxygen
B. Liquid oxygen
C. Piped gas
D. Oxygen concentrators

2. The sleep center is located on a floor in the hospital. What is the most likely
source of oxygen delivery?
A. Compressed oxygen
B. Liquid oxygen
C. Piped gas
D. Oxygen concentrators

OXYGEN DELIVERY DEVICES

3. What are the types of devices used to administer oxygen to your patient?
Name two.

INTERVENTION

4. Describe when to begin administering the oxygen to your patient.


TITRATION

5. Explain how to titrate oxygen during a sleep study.


PRECAUTIONS

6. How can oxygen be dangerous to a patient?


RECORDING AND DOCUMENTATION

7. What needs to be documented?


A. Current medications
B. Current use of oxygen
C. Oxygen liter flow
D. All of the above

8. How do I record oxygen use on the PSG?


A. PSI
B. cm/H2O
C. lpm
D. mm Hg
CASE STUDY NO. 1
A patient arrives at your sleep center for a polysomnogram, and the orders state,
“Titrate O2 if sats remain below 90%.” You begin to prepare for the procedure
by carefully reviewing the patient's medical history. You note that the patient
complains of shortness of breath with activity (exertional dyspnea) and at night,
with occasional waking with gasping. He also complains of constant fatigue. The
patient is slightly obese (BMI = 30), and is taking medication for hypertension
(lisinopril and hydrochlorothiazide) and type II diabetes (metformin and
glyburide). The patient had an overnight oximetry study (which is not in the
chart), and the lowest oxygen saturation of the night was 76%. There is no
mention of pulmonary disease or that the patient is currently using supplemental
oxygen.

9. What should you do to treat this patient?


CASE STUDY NO. 2


An elderly patient is sent for a polysomnogram with a long history of smoking
and smoke inhalation (>50 years). She is thin for her height (BMI = 18.1) and
rather frail looking. Her complaints include frequent waking with inability to
breathe at night, depression, panic attacks, coughing, wheezing, and daytime
sleepiness. There are no orders regarding oxygen use on this patient.
After beginning diagnostic study, you observe a few central breathing events
and even fewer obstructive breathing events in the first hour, along with many
periodic limb movements. In the second hour of the study, the patient enters
REM sleep and immediately begins to desaturate below 85%. Long periods of
decreased airflow are noted, but they do not meet the criteria for hypopneas (Fig.
30-1).
Figure 30-1 Example PSG for Case Study no. 2.

10. What do you do now?


CASE STUDY NO. 3


A pediatric patient undergoing polysomnography has a history of premature birth
with low birth weight and has developed bronchopulmonary dysplasia as a result
of respiratory distress syndrome and subsequent mechanical ventilation. The
polysomnogram has been ordered to rule out pediatric sleep apnea. The patient is
currently on continuous supplemental oxygen at 2.5 lpm.
The test is started (per doctor's orders) with the patient on usual oxygen flow
via nasal cannula. As the test progresses, the child demonstrates a large number
of respiratory events, both obstructive and central. There are also what appear to
be hypopneas, albeit without sufficient desaturation to score them.

11. What treatment is appropriate?


ANSWERS

1. D, Oxygen concentrators.

2. C, Piped gas.
Compressed oxygen comes in canisters (or tanks) of varying sizes. They are most useful for portable or
emergency oxygen delivery. If the patient arrives with one, it will be a smaller size (for instance, “E”
cylinder) that probably does not hold enough gas to last the length of the study. You will need to supply
your own supplemental oxygen. If your lab uses compressed gas, it is probably a larger tank (for
instance, “H” cylinder): These will last quite a while but are bulky, unwieldy, and not very practical.
They need to be secured to a wall or flooring, inspected, and replaced regularly. These tanks will have a
pressure and flow regulator, which must be set and adjusted appropriately, and you will need to run a
tube from the regulator to the patient's oxygen interface, which may be a significant distance from the
tank.
Liquid oxygen comes in tanks that are much smaller than compressed gas tanks. These are ideal for
portable use, but rather expensive for use in a sleep lab. If the patient brings one, it will have a built-in
flow regulator. Otherwise, you will have to provide a flow regulator. The advantage of liquid oxygen is
that a tank can hold much more liquid than it would gas.
Many hospitals and clinics provide piped oxygen, usually available in the patient's room via a wall
plate. This gas is most likely coming from a very large reservoir of liquid gas located elsewhere on the
facility campus. The liquid is checked and refilled regularly by maintenance personnel. This is one of
the most convenient ways to supply supplemental oxygen to your patients, if available. A specific wall
adapter regulator must be used to ensure only oxygen is delivered, since many of these wall plates are
right next to compressed air and vacuum outlets that look similar.
Most freestanding sleep laboratories resort to oxygen concentrators for regular use. These are hardy
and economical machines that extract oxygen from the air (by filtering out the nitrogen) and deliver
95% or greater concentrations of oxygen. The machines are slightly bulky, run on electricity (so you
need to be near an AC outlet), and tend to be fairly noisy. It is not uncommon to position the machine
outside the patient's room and run a fairly long oxygen hose into the room to keep the noise from
disturbing the patient during the test. The concentrator should come fitted with an adjustable flow
regulator (Fig. 30-2).
Figure 30-2 Oxygen concentrator.

See Fundamentals of Sleep Technology, 2nd edition, Chapter 36, pages 436–437.

3. Low-flow devices (e.g., nasal cannula) and high-flow devices (e.g., oxygen
mask, venture mask).
More common in the sleep laboratory is the nasal cannula or simple oxygen mask, used with a relatively
low flow of oxygen. The cannula can be fitted to the patient just like a nasal pressure transducer,
although the gas flow may interfere somewhat with the output signal. A better signal can be obtained if
you use a nasal or nasal/oral oxygen mask with a thermistor/thermocouple or pressure transducer
cannula beneath it. Special cannulas are also available that have two or more lumens to simultaneously
provide oxygen and measure breathing. Oxygen can also be bled directly into the CPAP circuit or into
most CPAP masks if they are being used. The flow can be regulated at the source, which usually
involves entering the patient's room every time you need to make an adjustment. A nasal cannula should
not be used if the patient requires more than 5 L of oxygen per minute (Fig. 30-3).
Figure 30-3 Dual lumen nasal cannula.

When a patient requires a high flow of oxygen (say >5 L/min) or high concentration (>30%), an
oxygen or Venturi mask or closed circuit system (e.g., endotracheal tube) must be used. For long-term
use, an in-line humidifier should be used as well. These interfaces are not very practical in the sleep
laboratory but may be encountered if working in a hospital or other health care facility. The flow of
oxygen and patient's breathing pattern may make it difficult or impossible to measure airflow during a
study, and the patient may be acutely unstable as well. It is important not to disrupt or interrupt the flow
of oxygen to the patient at any time, particularly during setup.
See Fundamentals of Sleep Technology, 2nd edition, Chapter 36, pages 439–440.

4. This is the whole crux of oxygen administration. When do you start therapy,
and how much oxygen do you give the patient?
First, a caveat you have probably heard before: Oxygen is a medication for the treatment of pulmonary
diseases, just like a bronchodilator. In most states and federal jurisdictions, as well as many hospitals,
clinics, and other facilities, its use is regulated by laws, statutes, and facility-specific protocols. Check to
be sure you are permitted to administer oxygen in your jurisdiction, and familiarize yourself with any
regulations or procedures that must be followed. Often, a physician's order is required.
Now, assuming you are permitted, begin by assessing need. What are the indications for
administering oxygen? Signs of respiratory failure include complaints of shortness of breath or air
hunger, rapid breathing rate, grunting, nasal flaring, sweating, palpitations, or gasping. In severe cases, a
bluish color to the skin and dangerous arrhythmias may occur. Also, the patient may experience
disorientation, agitation, or loss of consciousness (coma). Of course, there may be other causes for these
symptoms as well. This is why a medical history is so important.
In most cases (in the sleep lab), the major indication for the application of oxygen therapy is a
written order (or protocol) based on pulse oximetry readings. In this sample case, the order states
“Titrate O2 if sats remain below 90%.” Unfortunately, it does not state how long to wait to begin
therapy. A call to the ordering physician to clarify this matter might be in order. If she or he cannot be
reached, and in the absence of additional instructions, you can try the following: If the patient
demonstrates significant obstructive breathing events during sleep (apneas, hypopneas, RERAs), begin
traditional treatment (positive airway pressure) and titrate as instructed to eliminate obstructive events.
(You probably would not eliminate all events but should get the patient to five or less events per hour of
sleep.) If, once this is achieved, oxygen saturation remains below 90%, add oxygen to the patient's PAP
circuit and begin titration at a low-flow setting (e.g., 1 L/min).
If, on the other hand, the patient does not demonstrate significant obstructive breathing but oxygen
saturation remains below 90% (for at least 2 consecutive minutes), place a low-flow delivery device
(nasal cannula) on the patient and begin titration at 1 L/min.
Rationale: In order for oxygen to be effective, the patient must have a patent airway and functioning
respiratory system. If the airway is blocked, it must be opened first. In an emergency, this may involve
repositioning the patient's head or removing an obstacle from the mouth or throat. In the sleep lab, this
means establishing an open airway (usually with CPAP) prior to administration of oxygen. (If a patient
is experiencing obstructive apnea, oxygen will be unable to reach the alveoli. The airway must remain
open to provide adequate oxygen therapy.)
NOTE: This is just a guideline and no substitute for medical advice or instruction. Always check
with your ordering physician or medical director, or follow facility protocols when available.
See Fundamentals of Sleep Technology, 2nd edition, Chapter 36, pages 438–439.

5. If you have a facility protocol, follow this to the letter. If you do not have one,
you may adopt one like this:
Observe the patient's breathing and vital signs carefully for at least 5 minutes. If oxygen saturation is
consistently below 90%, increase oxygen flow by 1 L/min (1 to 2 lpm) and repeat. Allow at least 5
minutes for equilibration and complete circulation of oxygenated blood. (Typical cardiac output moves
about 5 L of blood per minute, or one body full in a healthy system. This allows for at least one
complete circuit of blood to see if therapy is effective.) Be aware that oxygen needs may increase with
increased patient activity.
If oxygen saturation is generally above 90% but shows serial dips below 90%, ascertain whether
these are due to body movement, respiratory disturbances, or other physiologic causes (e.g., cardiac
arrhythmias such as premature ventricular contractions). Make no adjustments for body movements or
artifacts, but increase PAP pressure for respiratory disturbances before increasing oxygen flow. For other
causes of desaturation, try increasing oxygen flow by 1 L/min. Repeat these steps as necessary until
oxygen saturation remains above 90%.
See Fundamentals of Sleep Technology, 2nd edition, Chapter 36, pages 438–439.

6. Since oxygen is a medication, it is possible to underdose or overdose a patient


with it. In people without respiratory disease, control of breathing depends
very much on the level of carbon dioxide in their blood. As CO2 increases,
so does depth and rate of breathing. A similar drop in CO2 results in
reduction of breathing to the point of apnea.
Those with chronic respiratory disease often respond differently, however. Since they are unable to
adequately ventilate, their CO2 levels are always high. In time, their body adapts to this and ceases to
respond in normal fashion. Instead, their breathing becomes regulated largely on the level of oxygen in
their arterial blood (which is usually fairly low).
When you administer oxygen to one of these compromised patients, the initial effect is a sudden rise
in blood oxygen pressure (pO2). While this does not affect the breathing of most people, it causes an
immediate reduction in breathing among these compromised patients. If you are running a PSG study on
such a patient, you may begin to see very long, slow breaths or repetitive lengthening central apneas.
While this is occurring, their ventilation becomes even more diminished resulting in ever-increasing
blood CO2 and accompanying respiratory acidosis. In a short time, they may develop acute respiratory
distress or even complete respiratory failure. If you see this begin to occur, consider reducing the
oxygen flow and getting immediate medical assistance (such as 911 or code blue, or call an attending
physician) (Fig. 30-4)

Figure 30-4 Repetitive central apneas.

See Fundamentals of Sleep Technology, 2nd edition, Chapter 36, page 438.

7. D, All of the above.

8. C, lpm.
As with all medications, administration of oxygen must be carefully and accurately documented. A good
rule of thumb is to record oxygen use just as you do PAP. A table or spreadsheet is useful for tracking
usage and changes, and all changes should be typed directly onto the PSG record. Unfortunately, no
commercially available PSG system (as of this date) automatically tracks oxygen levels as they do PAP
levels.
The most useful tool for the interpreting physician is the final report graphic page. This is a summary
of the entire study that often contains simultaneous graphics of the sleep hypnogram, average oxygen
saturation reading, heart or pulse rate, body position, and other recorded channels including PAP
pressure and leak. Adding supplemental oxygen flow to this graph makes their job immensely easier.
Unfortunately, this must be done manually; fortunately, it is a quick and simple task (Fig. 30-5).
Figure 30-5 Hypnogram and oxygen saturation summary.

See Fundamentals of Sleep Technology, 2nd edition, Chapter 36, pages 438–439.

9. As it turns out, this patient has several signs and symptoms of and risk factors
for obstructive sleep apnea syndrome (male, EDS, nocturnal gasping,
hypertension, etc.). The overnight oximetry demonstrated a low oxygen
saturation of 76%, but did not identify if this was a single event, continuous
event, or multiple desaturation events from baseline. After beginning the
study, you observe frequent obstructive apnea events with desaturation to the
mid 70s. After sufficient diagnostic time, you place the patient on PAP
therapy and titrate per protocol. At a pressure of 9 cm H2O, the patient's
respiratory events are largely eliminated, and his oxygen saturation remains
above 94%, with an occasional drop to 89% after an arousal and large body
movement. There is no need to apply supplemental oxygen therapy at this
point. Carefully document all steps taken, including the reason you did not
apply oxygen as initially instructed. Be objective and helpful; avoid being
defensive or censuring. (Resist writing “Patient didn't NEED oxygen.” Try
“Oxygen desaturation controlled with 9 cm H2O CPAP” instead. The doctors
will think a whole lot more of you.)

10. In this case, the desaturation is not caused by obstructed breathing, but is
probably the result of prolonged hypoventilation during REM. (Without
monitoring CO2, it is not possible to ascertain that true hypoventilation has
occurred.) The loss of accessory muscle use and altered response to blood
pCO2 during REM sleep can induce this in patients with a compromised
respiratory system. If protocols (and laws) allow, starting this patient on low
flow (1 lpm) oxygen via nasal cannula would seem indicated. If no
improvement is seen in 5 minutes, flow should be increased to 2 lpm with a
subsequent observation period. Titration should continue until baseline
saturation remains above 90%.
It is likely that the patient may arouse from REM sleep during this time and spontaneously resaturate
as her breathing normalizes. In this case, the cannula and oxygen should remain in place until her next
REM period, where the titration may be resumed, or flow maintained if it is still effective. Because of
the patient's pulmonary and smoking history, she should be observed carefully for the onset of
diminished breathing capacity or central apneas.
Again, the rationale for oxygen therapy should be carefully documented (oxygen applied via nasal
cannula at 1 lpm for persistent and continuous oxygen saturation below 90%). An account of the amount
of time spent below a certain SaO2 level (e.g., 87%) may also be necessary in order to qualify the
patient for reimbursement for oxygen therapy (especially for Medicare).

11. This is an extremely complicated case that ideally would involve a highly
trained pediatric pulmonologist's or sleep disorders specialist's direct input to
therapy. However, all too often, that is not available, and you must make the
best decision possible, given your background and training.
The patient is displaying obstructive breathing events, possibly many more than you can objectively
score. Due to the use of supplemental oxygen, she may not demonstrate significant enough
desaturations for the events to qualify as hypopneas. Nonetheless, the decreased volume, crescendo
snoring, and terminal arousals all indicate obstructed breathing. The best plan is to begin by treating
these anomalies, since the oxygen therapy will not be effective if the airway is closed.
Once CPAP is applied, the obstructive events begin to diminish and central events become more
prominent. Now a decision needs to be made about whether to initiate bilevel PAP (with or without a
backup rate) or advanced PAP or non-invasive ventilation or alter the oxygen setting. You may even
consider a trial of reducing the supplemental oxygen to see if there are any adverse effects.
Unfortunately, the ideal therapy for this type of patient is much too specialized and individualized for
any fixed protocol. It is also beyond the scope of this workbook. As a sleep technologist with these
types of patients, it is imperative to work closely with a highly skilled physician to determine the best
course of action. Therapy may vary considerably from patient to patient and may involve some trial and
error. As technologies improve, there may soon be a better solution; until then, caution, documentation,
and common sense are your best tools.
CHAPTER
31
Advanced PAP Therapies
LISA BOND

NOTE: This chapter corresponds to Chapter 37 in Fundamentals of Sleep


Technology, 2nd edition.

1. Adaptive servo ventilation devices provide varying amounts of inspiratory


support. The support is lowest during:
A. Hyperpneic phase of periodic breathing
B. Central sleep apnea
C. Obstructive sleep apnea
D. Episodes of snoring with mild-to-moderate flow limitation

2. During a sleep study, the technologist using adaptive servo ventilation should
increase expiratory positive airway pressure (EPAP) when which of the
following are present?
A. Periods of hypoventilation
B. Central apneas
C. Obstructive apneas
D. Hyperpneas

3. How does adaptive servo ventilation treat central apneas?


A. Provides auto-titrating expiratory positive airway pressures
B. Increases pressure support
C. Decreases pressure support
D. Follows predetermined backup breathing rate

4. Adaptive servo ventilation is an effective treatment for which of the


following disorders?
A. Obesity hypoventilation syndrome
B. Nocturnal asthma
C. Hunter-Cheyne-Stokes breathing pattern
D. Congenital central hypoventilation syndrome
5. Central apnea may emerge during PAP titration and may persist in some
patients, leading to a possible diagnosis of complex sleep apnea. If
overtitration of positive airway pressure is not suspected, consideration
should be given to which of the following?
A. Adaptive servo ventilation
B. Respiratory stimulants, such as theophylline
C. Recommendation for weight loss
D. Tracheostomy

CASE STUDY
A 42-year-old man has chronic low back pain that has worsened recently. He has
been taking over-the-counter pain medications but reports that they have stopped
working. His pain specialist starts him on oxycodone hydrochloride, a narcotic
analgesic. Since starting this new medication, he begins to have very loud
snoring and pauses in breathing along with daytime sleepiness and difficulty
concentrating at work. His blood pressure is 145/95 mm Hg, pulse is 82 per
minute and regular, and BMI is 32.5 kg/m2. The lungs are clear to auscultation,
and both cardiac and neurologic examinations are unremarkable. Match the
symptom with the most likely diagnosis:
6. Use of opiates A. OSA
7. Loud snoring B. CSA
8. Breathing pauses C. Both OSA and CSA
9. Hypertension
10. Obesity
The patient is referred for a sleep study. The initial portion of the study
shows severe obstructive sleep apnea (AHI = 42, lowest oxygen saturation =
68%) and occasional central apneas. A split-night protocol is used, and the
patient is titrated to CPAP of 11 cm H2O, which successfully resolves obstructive
apneas, hypopneas, and snoring, but central apneas become more frequent. The
technologist describes the central apnea pattern as irregular with a clear waxing
and waning amplitude pattern. Which of the following treatment options are
appropriate at this point, and which are inappropriate?
DISCUSSION QUESTION

16. Continuous positive airway pressure therapy is effective for many patients
with central sleep apnea. What other alternative therapies are available?
Which therapies may be useful for primary central sleep apnea, high altitude
periodic breathing, opiate-induced hypoventilation, and severe heart failure?

ANSWERS

1. A, Hyperpneic phase of periodic breathing.


See Fundamentals of Sleep Technology, 2nd edition, Chapter 37, page 445.

2. C, Obstructive apneas.
See Fundamentals of Sleep Technology, 2nd edition, Chapter 37, page 447.

3. D, Follows predetermined backup breathing rate.


See Fundamentals of Sleep Technology, 2nd edition, Chapter 37, page 446.

4. C, Hunter-Cheyne-Stokes breathing pattern.


See Fundamentals of Sleep Technology, 2nd edition, Chapter 37, page 448.

5. A, Adaptive servo ventilation.


See Fundamentals of Sleep Technology, 2nd edition, Chapter 37, pages 449–451.

6. C, Both OSA and CSA.


See Fundamentals of Sleep Technology, 2nd edition, Chapter 37, page 449.

7. A, OSA.
See Fundamentals of Sleep Technology, 2nd edition, Chapter 37, page 448.
8. C, Both OSA and CSA.
See Fundamentals of Sleep Technology, 2nd edition, Chapter 37, pages 448–449.

9. A, OSA.
See Fundamentals of Sleep Technology, 2nd edition, Chapter 16, page 171.

10. A, OSA.
See Fundamentals of Sleep Technology, 2nd edition, Chapter 16, page 171.

11. Inappropriate; the patient is in pain.


See Fundamentals of Sleep Technology, 2nd edition, Chapter 37, page 449.

12. Inappropriate.
See Fundamentals of Sleep Technology, 2nd edition, Chapter 16, page 176.

13. Inappropriate.
See Fundamentals of Sleep Technology, 2nd edition, Chapter 16, page 177.

14. Inappropriate.
See Fundamentals of Sleep Technology, 2nd edition, Chapter 37, page 448.

15. Appropriate.
See Fundamentals of Sleep Technology, 2nd edition, Chapter 37, page 447.

16. The following recommendations are included in the AASM Practice


Parameters: “CPAP therapy targeted to normalize the apnea–hypopnea index
(AHI) is indicated for the initial treatment of CSAS related to CHF.
(STANDARD)
Nocturnal oxygen therapy is indicated for the treatment of CSAS
related to CHF. (STANDARD)
Adaptive servo ventilation (ASV) targeted to normalize the apnea–
hypopnea index (AHI) is indicated for the treatment of CSAS related
to CHF. (STANDARD)
BPAP therapy in a spontaneous–timed (ST) mode targeted to
normalize the apnea–hypopnea index (AHI) may be considered for the
treatment of CSAS related to CHF only if there is no response to
adequate trials of CPAP, ASV, and oxygen therapies. (OPTION)
The following therapies have limited supporting evidence but may be
considered for the treatment of CSAS related to CHF after
optimization of standard medical therapy, if PAP therapy is not
tolerated, and if accompanied by close clinical follow-up:
acetazolamide and theophylline. (OPTION)
Positive airway pressure therapy may be considered for the treatment
of primary CSAS. (OPTION)
Acetazolamide has limited supporting evidence but may be considered
for the treatment of primary CSAS. (OPTION)
The use of zolpidem and triazolam may be considered for the
treatment of primary CSAS only if the patient does not have
underlying risk factors for respiratory depression. (OPTION)
The following possible treatment options for CSAS related to end-
stage renal disease may be considered: CPAP, supplemental oxygen,
bicarbonate buffer use during dialysis, and nocturnal dialysis.
(OPTION)” (Aurora RN, Chowdhuri S, Ramar K, et al. The treatment
of central sleep apnea syndromes in adults: practice parameters with an
evidence-based literature review and meta-analyses Sleep.
2012;35(1):17–49.)
See Fundamentals of Sleep Technology, 2nd edition, Chapter 37, pages 446–451.
CHAPTER
32
Dental Sleep Medicine
LAREE FORDYCE

NOTE: This chapter corresponds to Chapter 38 in Fundamentals of Sleep


Technology, 2nd edition.

1. After a patient with obstructive sleep apnea has been treated with an oral
appliance, sleep testing should be repeated. True or false?

2. Oral appliances or dental devices can be used to treat which type of sleep-
disordered breathing?
A. Mild-to-moderate obstructive sleep apnea
B. Cheyne-Stokes respiration
C. Central sleep apnea
D. Obesity hypoventilation syndrome

3. The most commonly used type of oral appliance is a:


A. Tongue-retaining device
B. Palate expanding device
C. Mandibular advancement device
D. Maxillomandibular advancement device

4. An oral appliance should ideally be fitted by a/an:


A. Family dentist
B. Sleep-trained dentist
C. Sleep medicine physician
D. Orthodontist

5. A 49-year-old male has a respiratory disturbance index (RDI) of 27 events


per hour and a past medical history of hypertension. His BMI is 29 kg/m2.
He tries continuous positive airway pressure (CPAP) therapy for 2 months
but reports that he could not wear it. Is he a suitable candidate for oral
appliance therapy (OAT)? Explain your answer.

6. Contraindications for using oral appliance therapy include all of the


following except:
A. Temporomandibular joint (TMJ) disorder
B. Central sleep apnea
C. Severe obstruction
D. Obesity hypoventilation syndrome

7. A 44-year-old male has a respiratory disturbance index (RDI) of 27 events


per hour. A majority of his respiratory events consist of Cheyne-Stokes
respiration. Is he an ideal candidate for oral appliance therapy? Explain your
answer.

8. Which of the following is an indication for oral appliance therapy?


A. Patients with primary snoring or mild obstructive sleep apnea who do
not respond to, or are not appropriate candidates for, treatment with
behavioral measures such as weight loss or sleep position change
B. Patients with severe central apnea
C. Patients with moderate-to-severe obstructive sleep apnea who are not
candidates for tonsillectomy and adenoidectomy, craniofacial operations, or
tracheostomy
D. All of the above

9. Dentists are often at the front line of screening patients for sleep disorders.
True or false?

10. Like continuous positive airway pressure (CPAP) therapy, long-term follow-
up is essential for compliance and efficacy of oral appliance therapy. True or
false?

11. Close collaboration between dentists and sleep professionals in treating


snoring and obstructive sleep apnea is important. True or false?

12. During follow-up visits for oral appliance therapy, dentists and physicians
pay special attention to:
A. Compliance
B. Evidence of recurrent obstructive sleep apnea
C. Dental complications
D. All of the above

ANSWERS

1. True, to ensure that the oral appliance is effectively treating sleep apnea
and/or snoring.
See Fundamentals of Sleep Technology, 2nd edition, Chapter 38, page 455.

2. A, Mild-to-moderate obstructive sleep apnea.


Oral appliances are indicated for the treatment of mild-to-moderate obstructive sleep apnea as well as
primary snoring.
See Fundamentals of Sleep Technology, 2nd edition, Chapter 38, page 456.

3. C, Mandibular advancement device.


See Fundamentals of Sleep Technology, 2nd edition, Chapter 38, page 454.

4. B, Sleep-trained dentist.
Although oral appliances can be fitted by any dentist, a sleep-trained dentist is generally better able to
provide airway management for patients with sleep-disordered breathing.
See Fundamentals of Sleep Technology, 2nd edition, Chapter 38, page 453.

5. Yes, oral appliances are indicated for patients with moderate obstructive sleep
apnea who are unable or unwilling to use CPAP devices.
See Fundamentals of Sleep Technology, 2nd edition, Chapter 38, page 456.

6. A, Temporomandibular joint disorder.


Although an improperly fitted oral appliance can worsen a preexisting TMJ disorder, the latter is not an
absolute contraindication for this therapy. Oral appliance therapy is not appropriate for patients who are
unable to breathe nasally or who have central sleep apnea or obesity hypoventilation syndrome.
See Fundamentals of Sleep Technology, 2nd edition, Chapter 38, page 455.

7. No. Cheyne-Stokes respiration is a contraindication for oral appliance therapy.


See Fundamentals of Sleep Technology, 2nd edition, Chapter 38, page 456.

8. A, Patients with primary snoring or mild obstructive sleep apnea who do not
respond to, or are not appropriate candidates for, treatment with behavioral
measures such as weight loss or sleep position change.
See Fundamentals of Sleep Technology, 2nd edition, Chapter 38, page 456.

9. True.
Many individuals see their dentist once or twice annually. Dentists can look for “dental wear” caused by
teeth grinding, or for “scalloping” of the tongue and/or cheeks, a physical examination finding in many
patients with obstructive sleep apnea. Enlarged tonsils, low-lying palate, large tongue, and long uvula, if
present, can also be visualized.
See Fundamentals of Sleep Technology, 2nd edition, Chapter 38, page 457.

10. True.
Any kind of treatment for sleep-disordered breathing should be followed to ensure that the patient is
using the therapy, that it is effective, and that it is not associated with any adverse complications.
See Fundamentals of Sleep Technology, 2nd edition, Chapter 38, page 455.

11. True.
See Fundamentals of Sleep Technology, 2nd edition, Chapter 38, pages 453 and 456.

12. D, All of the above.


To ensure effective therapy, all of these issues must be addressed.
See Fundamentals of Sleep Technology, 2nd edition, Chapter 38, pages 455–456.
CHAPTER
33
Treatment of Insomnia
ROXANNE TAYLOR

NOTE: This chapter corresponds to Chapter 39 in Fundamentals of Sleep


Technology, 2nd edition.

1. Some patients receive substantial attention, special consideration, and


sympathy for the misery associated with insomnia, influencing decisions
regarding treatment. What is this psychological process referred to?
A. The Hawthorne effect
B. Secondary gain
C. Transference
D. Empathy burnout

2. Depressed patients with insomnia benefit more from treatment of both


disorders than from treatments aimed at only one or the other disorder. In
these patients, insomnia and depression are said to be:
A. Comorbid
B. Causal
C. Complex
D. Chronic

3. Identify an effective way to instruct patients on proper sleep hygiene.


A. Identify specific sleep hygiene factors that affect the patient and
individualize care.
B. Detail all sleep hygiene measures and implement them all
simultaneously.
C. Provide a pamphlet and let the patient work it out on his or her own.
D. Skip sleep hygiene as a treatment because it is not effective.

4. Which of the following is a component of stimulus control therapy?


A. Imagining being on a beach in Hawaii drinking a piña colada from a
cocoanut
B. Keeping the bedroom as cold and dark as possible
C. Keeping a pet off the bed
D. Getting out of bed if unable to fall asleep within 15 minutes

5. Cognitive therapy focuses on:


A. Distracting the patient who obsesses about sleep
B. Thinking up ways to fool your brain into falling asleep
C. Using muscle relaxation to reduce the level of anxiety
D. Changing dysfunctional beliefs about sleep

6. Side effects of benzodiazepine use include memory impairment, accidents,


tolerance, dependence, abuse, withdrawal, and, in the case of agents with a
prolonged half-life:
A. Daytime sedation
B. Daytime anxiety
C. Increased incidence of rebound insomnia
D. Sleep terrors

7. Which of the following class of medications is commonly used for insomnia


but is not approved by the U.S. Food and Drug Administration (FDA) for
this indication?
A. Melatonin receptor agonists
B. Benzodiazepines
C. Selective serotonin reuptake inhibitors
D. Nonbenzodiazepine benzodiazepine receptor agonists

8. What is the definition of perpetuating factors for insomnia?


A. Vulnerability to biologic and psychological factors
B. Factors that maintain the insomnia once it is started
C. Factors that trigger the patient to develop insomnia
D. Intermittent use of hypnotics without physician supervision

9. An important instrument for monitoring the progress of the patient being


treated for insomnia is the:
A. Home sleep testing
B. Multiple sleep latency test
C. Epworth sleepiness scale
D. Sleep diary
10. Hypnotics are generally recommended for short-term use. What is a useful
strategy for helping patients to discontinue hypnotic use?
A. A slow taper of the medication
B. Abrupt discontinuation
C. Replacement of the hypnotic with an antidepressant
D. Using addiction support groups such as Narcotics Anonymous

Consider the following history of a sleep center patient: A 56-year-old male


has a long-standing history of anxiety prior to important events in his life. He
reports that he had been unable to sleep on the night before the 1st day of school,
graduation, prom, and marriage. He has had severe insomnia since he was
financially devastated by the 2008 recession, losing all of his savings and house
and nearly losing his company. He was diagnosed with depression at this time.
He is married with two children, and reports that he has considerable emotional
support from his family. They are very considerate of his insomnia, and his wife
often takes over driving the children around and doing household chores. His
finances have improved since then and he now describes himself as
“comfortable,” but the severe insomnia persists. He has grown to hate his
bedroom, mostly due to the long sleepless nights he has spent there. He will
occasionally fall asleep on the couch watching late night television. He is sure
that his failure to sleep at least 8 hours per night is reducing his ability to
function during the day. He first sought help from his internist, who
recommended a nightly dose of zolpidem. This enabled him to fall asleep
rapidly, but he continues to wake up in the middle of the night and be unable to
fall back to sleep. His internist has referred him to the sleep disorders center for
treatment.
Match the element of the patient's history with the appropriate terms
below.

11. He has always been unable to sleep A. Comorbid disorder


12. The 2008 recession B. Precipitating event
13. He was diagnosed with depression C. Pharmacotherapy
14. He has grown to hate his bedroom D. Perpetuating factor
15. Internist recommended zolpidem E. Predisposing factor

Indicate which of the following management strategies for insomnia are


indicated or not in this patient.

DISCUSSION QUESTION

21. Classification of insomnia has shifted from “primary” and “secondary” types
to recognition that insomnia and other disorders may be “comorbid.” How
would this new diagnostic approach influence the management of patients
with insomnia and medical or psychiatric disorders?

ANSWERS

1. B, Secondary gain
See Fundamentals of Sleep Technology, 2nd edition, Chapter 39, page 458.

2. A, Comorbid
See Fundamentals of Sleep Technology, 2nd edition, Chapter 39, page 459.

3. A, Identify specific sleep hygiene factors that affect the patient and
individualize care See Fundamentals of Sleep Technology, 2nd edition,
Chapter 39, page 459.

4. D, Getting out of bed if unable to fall asleep within 15 minutes See


Fundamentals of Sleep Technology, 2nd edition, Chapter 39, page 460.

5. D, Changing dysfunctional beliefs about sleep See Fundamentals of Sleep


Technology, 2nd edition, Chapter 39, page 460.

6. A, Daytime sedation See Fundamentals of Sleep Technology, 2nd edition,


Chapter 39, page 461.

7. C, Selective serotonin reuptake inhibitors See Fundamentals of Sleep


Technology, 2nd edition, Chapter 39, page 462.

8. B, Factors that maintain the insomnia once it is started See Fundamentals of


Sleep Technology, 2nd edition, Chapter 39, page 463.

9. D, Sleep diary
See Fundamentals of Sleep Technology, 2nd edition, Chapter 39, page 464.

10. A, A slow taper of the medication See Fundamentals of Sleep Technology,


2nd edition, Chapter 39, page 464.

11. E, Predisposing factor

12. B, Precipitating event

13. A, Comorbid disorder

14. D, Perpetuating factor

15. C, Pharmacotherapy

16. Not indicated

17. Indicated

18. Indicated

19. Not indicated

20. Indicated

21. “The strategy of augmenting an antidepressant medication with a brief,


symptom focused cognitive–behavioral therapy for insomnia is promising for
individuals with MDD and comorbid insomnia.” (Manber R, Edinger JD,
Gress JL, et al. Cognitive behavioral therapy for insomnia enhances
depression outcome in patients with comorbid major depressive disorder and
insomnia. Sleep. 2008;31(4):489–495.)
SECTION VI
Ancillary Procedures
CHAPTER
34
Multiple Sleep Latency Test and
Maintenance of Wakefulness Test
CHARLOTTE FROMER

NOTE: This chapter corresponds to Chapter 40 in Fundamentals of Sleep


Technology, 2nd edition.

1. Excessive daytime sleepiness is defined as:


A. Sleepiness occurring in a situation when one would be expected to be
awake and alert
B. Mean sleep onset latency during multiple sleep latency test of 14
minutes
C. A score of 6 on the Epworth Sleepiness Scale
D. A complaint of fatigue and lack of interest in things

2. The multiple sleep latency test (MSLT) provides supporting evidence for the
diagnosis of:
A. Obstructive sleep apnea
B. REM behavior disorder
C. Narcolepsy
D. Restless legs syndrome

3. The maintenance of wakefulness test (MWT) is indicated:


A. In the diagnosis of narcolepsy
B. To evaluate the efficacy of psychostimulant therapy
C. When a patient taking antidepressant medications complains of
sleepiness
D. For children suspected of attention deficit hyperactivity disorder

4. The standard multiple sleep latency test (MSLT) includes:


A. Vertical EOG monitoring
B. At least four nap opportunities
C. Temporal and parietal EEG monitoring
D. Use of a snoring microphone

5. Standard instructions to the patient during a multiple sleep latency test


(MSLT) include “please lie quietly, keep your eyes closed, and
______________.”
A. Do not eat anything
B. Try to stay awake as long as possible
C. Call for the technologist if you need to use the bathroom
D. Try to fall asleep

6. The clinical multiple sleep latency test (MSLT) is continued for 15 minutes
after the first:
A. Epoch of any stage of sleep
B. Sleep spindle
C. Three epochs of stage N2
D. REM sleep epoch

7. Normal mean sleep latency on the multiple sleep latency test (MSLT) for a 6-
year-old patient is:
A. Eight minutes
B. Ten minutes
C. Fifteen minutes
D. Not validated by normative data

8. In comparison to a 20-minute maintenance of wakefulness test (MWT) trial,


a 40-minute trial reduces the probability of:
A. A false-positive test
B. A ceiling effect
C. Experimenter bias
D. Misdiagnosis of narcolepsy

9. Sleep latency for the maintenance of wakefulness test (MWT) is the time
from sleep onset to the:
A. First sleep spindle
B. First epoch of sleep
C. First rapid eye movement
D. First slow eye movement
10. If no sleep occurs during a multiple sleep latency test (MSLT) nap trial, the
sleep latency is reported as:
A. The average for the patient
B. Zero minutes
C. Twenty minutes
D. Forty minutes

Consider the following history of a sleep center patient: A 20-year-old man


has a history compatible with narcolepsy. He falls asleep inappropriately, has
weakness in his knees when he laughs at a joke, and has vivid dreams. These
symptoms started 10 years ago and have worsened recently. He reports
occasional episodes of reflux, especially after a spicy meal. He also suffers from
depression for which he takes fluoxetine. He has classic migraine and describes
an aura of scintillating scotoma. His usual bedtime is 1 AM, and it typically takes
him 15 minutes to fall asleep. He wakes at 7 AM during schooldays and at 10 AM
on weekends. Which elements of the patient's history should be taken into
account when interpreting the results of the multiple sleep latency test (MSLT)?

DISCUSSION QUESTION

16. Why should the multiple sleep latency test (MSLT) and maintenance of
wakefulness test (MWT) be performed with strict adherence to standardized
protocols, including appropriate conditions, proper recording techniques,
and interpretation by qualified and experienced clinicians?

ANSWERS

1. A, Sleepiness occurring in a situation when one would be expected to be


awake and alert See Fundamentals of Sleep Technology, 2nd edition,
Chapter 40, page 467.

2. C, Narcolepsy
See Fundamentals of Sleep Technology, 2nd edition, Chapter 40, page 468.

3. B, To evaluate the efficacy of psychostimulant therapy See Fundamentals of


Sleep Technology, 2nd edition, Chapter 40, page 468.

4. B, At least four nap opportunities See Fundamentals of Sleep Technology,


2nd edition, Chapter 40, page 469.

5. D, Try to fall asleep See Fundamentals of Sleep Technology, 2nd edition,


Chapter 40, page 470.

6. A, Epoch of any stage of sleep See Fundamentals of Sleep Technology, 2nd


edition, Chapter 40, page 470.

7. D, Not validated by normative data See Fundamentals of Sleep Technology,


2nd edition, Chapter 40, page 470.

8. B, A ceiling effect See Fundamentals of Sleep Technology, 2nd edition,


Chapter 40, page 471.

9. B, First epoch of sleep See Fundamentals of Sleep Technology, 2nd edition,


Chapter 40, page 472.

10. C, Twenty minutes See Fundamentals of Sleep Technology, 2nd edition,


Chapter 40, page 470.

11. No

12. Yes

13. No
14. Yes

15. No

16. “Many factors can impact the MSLT and MWT, and thus strict clinical
protocols and interpretation by qualified experienced clinicians are essential
to assure clinical utility of these tests.” (Standards of Practice Committee of
the American Academy of Sleep Medicine. Practice parameters for clinical
use of the multiple sleep latency test and the maintenance of wakefulness test.
Sleep. 2005;28(1):113–121.) See Fundamentals of Sleep Technology, 2nd
edition, Chapter 40, page 468.
CHAPTER
35
Actigraphy
KAREN I. SMITH

NOTE: This chapter corresponds to Chapter 41 in Fundamentals of Sleep


Technology, 2nd edition.

1. Actigraphy is the:
A. Measurement and quantification of REM movements
B. Recording and analysis of the nondominant wrist movements
C. Recording of leg movements
D. Home sleep testing

2. Actigraphy can collect data:


A. For short periods of time
B. For extended periods of time
C. In the patient's natural environment
D. For all of the above

3. Actigraphy is indicated to assist with the evaluation and treatment of all


circadian rhythm sleep disorders. True or false?

4. What does an actigraph use to measure activity levels?


A. Gold cup electrode
B. Inductance plethysmography
C. Pressure cannula
D. Piezoelectric accelerometer

5. Actigraphy is best able to provide an accurate estimate of sleep–wake


patterns in which of the following populations?
A. Toddlers and young children
B. Adults with normal sleep
C. Adults with insomnia
D. Hospitalized patients

6. Actigraphy can substitute for polysomnography in diagnosing patients with


periodic limb movement disorder (PLMD). True or false?

7. Sleep efficiency (SE) in actigraphy should only be calculated if a (an)


______________ is included.
A. Video recording
B. Sleep diary
C. Bed partner interview
D. Event button

8. Which of the following is useful in the evaluation of a suspected insomnia


due to a circadian rhythm sleep disorder?
A. Polysomnography
B. Home sleep testing
C. Dim light melatonin onset level
D. Sleep diary and actigraphy

9. Actigraphy with a light sensor can monitor whether:


A. The patient is obtaining a sufficient amount of light in the morning
B. The patient is restricting light late in the day
C. The patient is getting the proper spectrum of light
D. A and B are correct

10. Actigraphy worn for 7 to 10 days prior to a multiple sleep latency test
(MSLT) ensures the patient is obtaining sufficient sleep. True or false?

DISCUSSION QUESTIONS

11. What instructions should be provided to patients undergoing actigraphy?


12. Would actigraphy help in the assessment of pediatric patients who cannot
tolerate an overnight polysomnography?

13. Discuss ways actigraphy can be used to evaluate sleep–wake patterns.


ANSWERS

1. B, Recording and analysis of the nondominant wrist movements.


See Fundamentals of Sleep Technology, 2nd edition, Chapter 41, page 474.

2. D, All of the above.


See Fundamentals of Sleep Technology, 2nd edition, Chapter 41, page 474.

3. False.
See Fundamentals of Sleep Technology, 2nd edition, Chapter 41, page 474.

4. D, Piezoelectric accelerometer.
See Fundamentals of Sleep Technology, 2nd edition, Chapter 41, page 474.

5. B, Adults with normal sleep.


See Fundamentals of Sleep Technology, 2nd edition, Chapter 41, page 474.

6. False.
See Fundamentals of Sleep Technology, 2nd edition, Chapter 41, page 478.

7. B, Sleep diary.
See Fundamentals of Sleep Technology, 2nd edition, Chapter 41, page 477.

8. D, Sleep diary and actigraphy.


See Fundamentals of Sleep Technology, 2nd edition, Chapter 41, page 478.

9. D, A and B are correct.


See Fundamentals of Sleep Technology, 2nd edition, Chapter 41, page 322.

10. True.
See Fundamentals of Sleep Technology, 2nd edition, Chapter 41, page 481.

11. Keeping a daily sleep diary is helpful. Patients are asked to record times of
wake and sleep, and periods when the actigraph is not being used. A diary
helps identify times with low activity during wakefulness as well as times
with high activity during sleep. Lastly, reported bedtimes and wake times
permit calculation of sleep onset latency and sleep efficiency.
See Fundamentals of Sleep Technology, 2nd edition, Chapter 41, page 475.

12. Actigraphy is not a useful tool in pediatric patients, and is a poor measure of
periodic limb movements in children.
See Fundamentals of Sleep Technology, 2nd edition, Chapter 41, page 478.

13. Actigraphy can be used to identify possible causes of daytime sleepiness, to


monitor the patient's adherence to treatment recommendations (e.g.,
maintaining a consistent sleep schedule), and to support the validity of a
multiple sleep latency test (MSLT) evaluation for narcolepsy or hypersomnia.
See Fundamentals of Sleep Technology, 2nd edition, Chapter 41, page 478.
CHAPTER
36
Portable Monitoring
LAREE FORDYCE

NOTE: This chapter corresponds to Chapter 42 in Fundamentals of Sleep


Technology, 2nd edition.

1. Which of the following is NOT a guideline of the American Academy of


Sleep Medicine (AASM) for unattended portable sleep apnea monitoring?
A. Comprehensive sleep evaluation not required
B. No comorbid medical conditions, such as heart failure or neuromuscular
disease
C. Clinical suspicion of periodic limb movement disorder or narcolepsy
D. No clinical features of obesity hypoventilation syndrome (OHS)

2. List three advantages of portable sleep apnea monitoring compared to


polysomnography.

3. List three disadvantages of portable sleep apnea monitoring compared to


polysomnography.

4. Portable sleep apnea monitoring includes which of the following tests?


Identify all of the correct choices.
A. Level 1 = Attended PSG
B. Level 2 = Unattended PSG
C. Level 3 = Cardiorespiratory monitoring
D. Level 4 = Continuous single or dual bioparameter recording

5. In portable sleep apnea monitoring, a minimum of four channels are


recorded. These channels include:
A. Chest effort, heart rate, oxygen saturation, snoring
B. Chest effort, abdominal effort, airflow, electrocardiography
C. Chest effort, airflow, oxygen saturation, electrocardiography
D. Oxygen saturation, heart rate, airflow, snoring

CASE STUDY NO. 1 FOR QUESTIONS 6 AND 7

A 57-year-old man with hypertension and depression is referred for possible


obstructive sleep apnea. His BMI is 48 kg/m2. A portable sleep apnea study
shows that the patient has Cheyne-Stokes respiration (Fig. 36-1).

Figure 36.1

6. What should you do for this patient?


A. Start auto-titrating positive airway pressure (APAP) therapy.
B. Repeat polysomnography with positive airway pressure (PAP) titration
study.
C. Start supplemental oxygen therapy.
D. Start adaptive servo ventilation.

7. This patient is a good candidate for portable sleep apnea monitoring. True or
false?

CASE STUDY NO. 2 FOR QUESTION 8


A 23-year-old woman presents with snoring and possible obstructive sleep apnea
(OSA). Past medical history is unremarkable. Her BMI is 24 kg/m2. Results of
her portable sleep apnea test are shown in Figures 36-2 and 36-3 and Table 36-1.

Figure 36.2
Figure 36.3

Table 36-1
8. From these data, what is the next best step for this patient?
A. Continuous positive airway pressure trial at a setting of 10 cm H2O
B. Start auto-titrating positive airway pressure (APAP) therapy
C. Repeat polysomnography with positive airway pressure (PAP) titration
study
D. Start supplemental oxygen therapy

CROSSWORD PUZZLE

ACROSS
1. Type of sleep-disordered breathing that is a contraindication for portable
sleep apnea testing (acronym)

3. Subjective test of daytime sleepiness (acronym)


4. Gold standard test for diagnosing sleep apnea (acronym)



7. Portable sleep apnea monitors should ideally be able to display this (two
words)

10. Type of portable sleep apnea test that includes measures of ventilation, heart
rate or electrocardiography (ECG), and oxygen saturation (SaO2)

11. Studies completed at home are most often inadequate in this regard

12. Performed by a trained sleep medicine physician


DOWN

2. This must occur prior to portable monitoring setup (two words)


5. Therapy for sleep-disordered breathing (acronym)


6. Type of portable sleep apnea test that includes measures of


electroencephalography (EEG), electrooculography (EOG), chin
electromyography (EMG), electrocardiography (ECG) or heart rate, airflow,
respiratory effort, and oxygen saturation (SaO2)

8. Portable home sleep testing is also known as ______________ (acronym)


9. Portable home sleep testing is indicated when there is a high probability of


this sleep disorder (acronym)

ANSWERS

1. C, Clinical suspicion of periodic limb movement disorder or narcolepsy.


See Fundamentals of Sleep Technology, 2nd edition, Chapter 42, page 493.

2. Less complex; not limited to number of laboratory beds; less expensive to


perform.
See Fundamentals of Sleep Technology, 2nd edition, Chapter 42, pages 487, 490–491.

3. Lacks direct observation by sleep technologists; higher failure rate; does not
measure sleep quality.
See Fundamentals of Sleep Technology, 2nd edition, Chapter 42, pages 487, 490–491.

4. B, Level 2 = Unattended PSG and, C, Level 3 = Cardiorespiratory monitoring.


See Fundamentals of Sleep Technology, 2nd edition, Chapter 42, page 483.

5. C, Chest effort, airflow, oxygen saturation, electrocardiography.


See Fundamentals of Sleep Technology, 2nd edition, Chapter 42, page 483.

6. B, Repeat polysomnography with positive airway pressure (PAP) titration


study.
Cheyne-Stokes respiration (CSR) is a contraindication for auto-titrating positive airway pressure
(APAP) therapy. Although CSR may improve with oxygen therapy and adaptive servo ventilation, these
are not first-line therapies for this disorder.
See Fundamentals of Sleep Technology, 2nd edition, Chapter 42, pages 493 and 499.

7. False.
Given his severe obesity, this patient is at risk for obstructive sleep apnea (OSA) and obesity
hypoventilation syndrome (OHS). Therefore, a laboratory-based polysomnography is preferred.
Additionally, monitoring CO2 levels during sleep is recommended to assess for possible OHS.
See Fundamentals of Sleep Technology, 2nd edition, Chapter 42, pages 493, 499.

8. B, Start auto-titrating positive airway pressure (APAP) therapy.


A trial of auto-titrating positive airway pressure (APAP) is safe for patients with no significant medical
comorbidities. In-laboratory titration should be considered for APAP failures.
See Fundamentals of Sleep Technology, 2nd edition, Chapter 42, page 499.

CROSSWORD PUZZLE SOLUTION

ACROSS
1. CSA—central sleep apnea.
See Fundamentals of Sleep Technology, 2nd edition, Chapter 42, page 499.

2. ESS—Epworth Sleepiness Scale.


See Fundamentals of Sleep Technology, 2nd edition, Chapter 42, page 639.

3. PSG
See Fundamentals of Sleep Technology, 2nd edition, Chapter 42, page 644.

4. Raw data.
See Fundamentals of Sleep Technology, 2nd edition, Chapter 42, page 494.

5. Cardiorespiratory.
See Fundamentals of Sleep Technology, 2nd edition, Chapter 42, page 483.

6. Technically.
See Fundamentals of Sleep Technology, 2nd edition, Chapter 42, page 491.

7. Interpretation.
See Fundamentals of Sleep Technology, 2nd edition, Chapter 42, page 494.

DOWN

8. Sleep evaluation.
See Fundamentals of Sleep Technology, 2nd edition, Chapter 42, page 493.

9. CPAP.
See Fundamentals of Sleep Technology, 2nd edition, Chapter 42, page 638.

10. Level II.


See Fundamentals of Sleep Technology, 2nd edition, Chapter 42, page 484.

11. OCST.
See Fundamentals of Sleep Technology, 2nd edition, Chapter 42, page 482.

12. OSA.
See Fundamentals of Sleep Technology, 2nd edition, Chapter 42, page 482.
SECTION VII
Pediatrics
CHAPTER
37
Pediatric Polysomnography
LAREE FORDYCE

NOTE: This chapter corresponds to Chapter 43 in Fundamentals of Sleep


Technology, 2nd edition.

1. The technique that familiarizes patients to the sleep center experience is


referred to as:
A. Visitation
B. Sensitization
C. Desensitization
D. Exposure

2. A 3-year-old child refuses the nasal cannula and thermistor during the hook-
up portion of the polysomnography. What can a sleep technologist do to
address this?
A. Run the study without nasal sensors
B. Discontinue the study and send the patient home
C. Start the study without sensors, but apply them once the child is asleep
D. Ignore the protest and forcibly apply the sensors

3. Precautions to assure safety for both staff and patients are essential in the
sleep laboratory. Name four safety measures that are particularly important
during a pediatric sleep study.

4. The American Academy of Sleep Medicine (AASM) recommends using the


same technical specifications when performing adult and pediatric
polysomnography. True or false?

5. Capnography is recommended when polysomnography is conducted to


evaluate for sleep-disordered breathing in children. True or false?

6. In addition to video recording, which of the following should be documented


during a pediatric polysomnography?
A. Body position
B. Behavior of the patient
C. Behavior of the parents
D. All of the above

7. Which of the following will help ensure a successful pediatric


polysomnography? Identify all correct answers.
A. Preparing for the child and parent needs based on the patient's age and
development and special requests
B. Engaging both the child and parents in a family-centered approach
C. Understanding age-specific differences in the application of sensors
D. Replacing dislodged sensors only when the patient is awake

8. Polysomnography remains the gold standard test for diagnosing sleep-


disordered breathing in children. True or false?

9. There are different criteria for acquiring polysomnographic data and scoring
sleep stages in infants younger than 2 months of age and older children.
True or false?

10. It is helpful to involve the patient's family when performing a pediatric


polysomnography. True or false?

CROSSWORD PUZZLE
ACROSS
1. What a sleep technologist might have to do before setting up a
polysomnography (PSG) for a pediatric patient

2. Children with obstructive sleep apnea (OSA) may do poorly at this place.

3. Performed to observe and monitor behavior and activity

4. The ______________ should be synchronized for audio–video recording and


polysomnographic data.

5. Another term for bedwetting


6. A disorder that can develop in school-aged children with sleep disturbance


(acronym)

7. Monitoring required during pediatric polysomnography to assess respiratory


patterns

8. Type of montage used when a nocturnal seizure disorder is suspected


9. A subjective measure of daytime sleepiness (acronym)


10. Characteristic of sensors used in pediatric polysomnography


DOWN

2. What you will do to the sampling rate to improve visual resolution of EEG

5. What sleep technologists who perform pediatric polysomnography should be


7. A type of parasomnia that most commonly occurs in children



9. Treatment for obstructive sleep apnea in children (acronym)

10. Abnormal respiratory effort pattern


11. The percent distance between C3 and Cz


15. Treatment for sleep-disordered breathing (acronym)


ANSWERS

1. B, Sensitization.
See Fundamentals of Sleep Technology, 2nd edition, Chapter 43, pages 504–505.

2. C, Start the study without sensors, but apply them once the child is asleep.
This option will allow the placement of sensors without a fight.
See Fundamentals of Sleep Technology, 2nd edition, Chapter 43, page 504.

3. Remove sharp objects; cover all electrical outlets; identify and remove all
choking hazards; place rails in beds and cribs.
See Fundamentals of Sleep Technology, 2nd edition, Chapter 43, page 505.

4. True
See Fundamentals of Sleep Technology, 2nd edition, Chapter 43, page 507.

5. True
See Fundamentals of Sleep Technology, 2nd edition, Chapter 43, page 505.

6. D, All of the above.


See Fundamentals of Sleep Technology, 2nd edition, Chapter 43, pages 506 and 509.

7. A, Preparing for the child and parent needs based on the patient's age,
development, and special requests; B, Engaging both the child and parents in
a family-centered approach; and C, Understanding age-specific differences
in the application of sensors.
See Fundamentals of Sleep Technology, 2nd edition, Chapter 43, pages 504–505.
8. True
See Fundamentals of Sleep Technology, 2nd edition, Chapter 43, page 509.

9. True
See Fundamentals of Sleep Technology, 2nd edition, Chapter 43, page 508.

10. True
See Fundamentals of Sleep Technology, 2nd edition, Chapter 43, pages 503–504.

CROSSWORD PUZZLE SOLUTION

ACROSS
1. Desensitization.
See Fundamentals of Sleep Technology, 2nd edition, Chapter 43, pages 493 and 499.

2. School
See Fundamentals of Sleep Technology, 2nd edition, Chapter 43, page 527.

3. Video
See Fundamentals of Sleep Technology, 2nd edition, Chapter 43, pages 493 and 499.

4. Time
See Fundamentals of Sleep Technology, 2nd edition, Chapter 43, pages 493 and 499.

5. Enuresis
See Fundamentals of Sleep Technology, 2nd edition, Chapter 43, page 539.

6. ADHD
See Fundamentals of Sleep Technology, 2nd edition, Chapter 43, page 527.

7. Capnography
See Fundamentals of Sleep Technology, 2nd edition, Chapter 43, page 505.

8. Extended
See Fundamentals of Sleep Technology, 2nd edition, Chapter 43, page 508.

9. ESS
See Fundamentals of Sleep Technology, 2nd edition, Chapter 43, page 639.

10. Small
See Fundamentals of Sleep Technology, 2nd edition, Chapter 43, pages 493 and 499.
DOWN

11. Increase
See Fundamentals of Sleep Technology, 2nd edition, Chapter 43, page 508.

12. Trained
See Fundamentals of Sleep Technology, 2nd edition, Chapter 43, page 504.

13. Night terrors


See Fundamentals of Sleep Technology, 2nd edition, Chapter 43, page 548.

14. TA
See Fundamentals of Sleep Technology, 2nd edition, Chapter 43, pages 526–527.

15. Paradoxical
See Fundamentals of Sleep Technology, 2nd edition, Chapter 43, page 509.

16. Twenty
See Fundamentals of Sleep Technology, 2nd edition, Chapter 43, pages 314–316.

17. CPAP
See Fundamentals of Sleep Technology, 2nd edition, Chapter 43, page 527.
CHAPTER
38
Pediatric Scoring
TIM A. STATZA

NOTE: This chapter corresponds to Chapter 44 in Fundamentals of Sleep


Technology, 2nd edition.

1. In contrast to alpha rhythm in adults, the dominant posterior rhythm in


infants is:
A. Slower
B. Faster
C. About the same
D. Sharp and spiky rather than smooth and sinusoidal

2. Stage N can be used in scoring NREM sleep in:


A. Infants without spindles, K complexes, or slow waves
B. Children without alpha rhythm activity
C. Children without vertex sharp waves
D. Infants without sawtooth waves

3. Hypnagogic hypersynchrony is:


A. A vivid visual phenomenon at sleep onset
B. Interictal activity
C. A pediatric equivalent of spindle activity
D. Bursts of theta waves during drowsiness

4. In infants less than 2 months of age, a distinctive EEG pattern associated


with NREM or quiet sleep and consisting of high-voltage slow waves
separated by 4 to 8 seconds of lower-voltage mixed frequency waves is
called:
A. Burst suppression
B. Frontal intermittent rhythmic delta activity
C. Flat and wave activity
D. Trace alternant
5. The duration of an obstructive apnea in a pediatric patient is at least:
A. 10 seconds
B. 20 seconds
C. Two missed breaths
D. Long enough to cause a 3% oxygen desaturation

6. In children, central apneas lasting less than 20 seconds and not accompanied
by a 3% oxygen desaturation or arousal are:
A. Scored as a hypopnea
B. Extremely rare
C. Normal
D. Scored as postarousal breathing pauses

7. CO2 monitoring, using either end-tidal or transcutaneous sensors, is


mandatory in children for the scoring of:
A. Apnea
B. Hypopnea
C. Central events
D. Hypoventilation

8. In infants, episodes of three or more central apneas lasting more than 3


seconds and separated by less than 20 seconds of normal breathing are
called:
A. Ataxic breathing
B. Periodic breathing
C. Central breathing
D. Klein-Levin breathing

9. Scoring arousals in children is:


A. Based on changes in heart rate
B. The same as in adults
C. Not required
D. Based on behavior and movement

10. In a limited channel study in infants, apneas associated with severe oxygen
desaturation may lead the scorer to infer that the infant:
A. Has had a “near miss” episode
B. Is awake
C. Is in REM sleep
D. Is supine

DISCUSSION QUESTIONS

11. Age is critically important to the scoring of sleep stages in infants. What is
conceptional age, and why is it so important for infant sleep scoring?

12. The American Academy of Sleep Medicine (AASM) 2012 Scoring Manual
Task Force considered changing the rule to recommend that all children
younger than 18 years be scored using pediatric rules, as opposed to the
current rule that allows physicians to score children between 13 and 18
years using adult criteria. What would be the effect of this rule change? Do
you support the change?

ANSWERS

1. A, Slower.
See Fundamentals of Sleep Technology, 2nd edition, Chapter 44, page 512.

2. A, Infants without spindles, K complexes, or slow waves.


See Fundamentals of Sleep Technology, 2nd edition, Chapter 44, page 513.

3. D, Bursts of theta waves during drowsiness.


See Fundamentals of Sleep Technology, 2nd edition, Chapter 44, page 514.

4. D, Trace alternant.
See Fundamentals of Sleep Technology, 2nd edition, Chapter 44, page 514.

5. C, Two missed breaths.


See Fundamentals of Sleep Technology, 2nd edition, Chapter 44, page 519.

6. C, Normal.
See Fundamentals of Sleep Technology, 2nd edition, Chapter 44, page 519.
7. D, Hypoventilation.
See Fundamentals of Sleep Technology, 2nd edition, Chapter 44, page 520.

8. B, Periodic breathing.
See Fundamentals of Sleep Technology, 2nd edition, Chapter 44, page 522.

9. B, The same as in adults.


See Fundamentals of Sleep Technology, 2nd edition, Chapter 44, page 522.

10. C, Is in REM sleep.


See Fundamentals of Sleep Technology, 2nd edition, Chapter 44, page 524.

11. “The importance of defining an infant's conceptional age arises because the
EEG (or a PSG) of a normal infant is more dependent upon the age of the
brain following conception than the number of days following birth. Except
when stressed, or in situations involving encephalopathy or medication-
related factors, the EEG or PSG of a neonate reflects the actual
developmental age of the brain. The brain, EEG, and PSG of an infant
continue to develop and mature at a similar rate, independent of whether the
infant is in utero or postdelivery. An EEG or PSG of a normal premature
infant born at 32 weeks of gestational age whose chronologic age is 8 weeks
should resemble that of a normal infant born at 40 weeks of gestational age 2
days earlier. The EEG and PSG patterns observed in infants 6 months or
younger correlate most closely with the infant's CA; after that the number of
months in age postterm birth usually suffices.” (Grigg-Damberger M, Gozal
D, Marcus CL, et al. The visual scoring of sleep and arousal in infants and
children: Development of polygraphic features, reliability, validity, and
alternative methods. J Clin Sleep Med. 2007;3(2):201–240.)

12. “The task force considered a change in the current rule to one recommending
that pediatric rules be used for all children younger than 18 years. When the
AASM pediatric scoring rules were developed, there were no data available
specifically pertaining to adolescents; therefore, it was suggested that
adolescents aged 13 to 18 years could be scored using either pediatric or adult
criteria. Since then, two studies have shown significant differences in
respiratory parameters when the PSGs of adolescents aged 13 to 18 years
were scored using pediatric versus adult criteria. A study of normal
adolescents showed that they had a significantly higher AHI when pediatric
scoring rules were used. Another study of adolescents with suspected OSA
also showed a significant difference in AHI using pediatric versus adult
scoring rules, especially between the pediatric rule and the recommended
adult rule for hypopneas. In addition, significantly more children would have
been diagnosed with OSA using pediatric versus adult rules.” (Berry RB,
Budhiraja R, Gottlieb DJ, et al. Rules for scoring respiratory events in sleep:
Update of the 2007 AASM Manual for the Scoring of Sleep and Associated
Events. J Clin Sleep Med. 2012;8(5):597–619.)
CHAPTER
39
Sleep-Related Breathing Disorders in
Children
CHRISTOPHER A. COOK

NOTE: This chapter corresponds to Chapter 45 in Fundamentals of Sleep


Technology, 2nd edition.

1. What is the most common indication for polysomnography among children?


A. Respiratory problems
B. Headaches
C. Difficulty falling asleep
D. Nocturnal enuresis

2. Which of the following statements regarding apnea of prematurity (AOP) is


correct?
A. A cardiovascular consequence of obstructive sleep apnea (OSA)
B. Often seen in infants of less than 33 weeks' gestation.
C. Cannot be treated by medications
D. Related to the infant's sleep position

3. It is advisable to place babies and infants in this sleep position in order to


reduce the risk of sudden infant death syndrome (SIDS).
A. Upright
B. Supine
C. Side
D. Prone

4. Which of the following is not associated with snoring and obstructive sleep
apnea (OSA) in pediatric patients?
A. Nocturnal enuresis
B. Parasomnia
Cognitive impairment
D. Colic

5. Which of the following is the treatment of choice for childhood obstructive


sleep apnea (OSA) if there are no contraindications?
A. Upper airway surgery
B. Continuous positive airway pressure (CPAP)
C. Medication
D. Oxygen therapy

DISCUSSION QUESTIONS

6. Discuss the possible causes and diagnostic evaluation of apparent life-


threatening events (ALTEs).

7. Discuss the risk factors, symptoms, and treatments of pediatric obstructive


sleep apnea.

CASE STUDY

Charlie, a 2-month-old infant, is found making high-pitched noises during a nap.


He is lying in a semiprone position and has a purplish-blue skin tone. Fearing an
apparent life-threatening event (ALTE), his parent calls emergency services and
begins infant resuscitation. When emergency services arrive, Charlie has begun
breathing and his skin color returns to normal.

8. Which of the following a risk factor for his apparent life-threatening event
(ALTE)?
A. Prematurity
B. Congenital heart disease
C. Positive family history of ALTE
D. All of the above
9. Apparent life-threatening event (ALTE) increases the risk of Sudden Infant
Death Syndrome (SIDS). True or false?

10. Charlie was stable when the emergency services arrived. What further action
should be taken at this time?
A. Set up an appointment with Charlie's pediatrician within 7 days.
B. Take Charlie to the emergency room for evaluation.
C. Have the patients stay up all night to observe any abnormal events
D. Order an overnight oximetry to measure nighttime oxygen levels

ANSWERS

1. A, Respiratory problems.
See Fundamentals of Sleep Technology, 2nd edition, Chapter 45, page 525.

2. B, Often seen in infants of less than 33 weeks' gestation.


See Fundamentals of Sleep Technology, 2nd edition, Chapter 45, page 525.

3. B, Supine.
See Fundamentals of Sleep Technology, 2nd edition, Chapter 45, page 527.

4. D, Colic.
See Fundamentals of Sleep Technology, 2nd edition, Chapter 45, page 526.

5. A, Upper airway surgery.


See Fundamentals of Sleep Technology, 2nd edition, Chapter 45, page 527.

6. Gastroesophageal reflux and lower respiratory tract infection can give rise to
apparent life-threatening events (ALTEs). Other possible causes include
seizures, cardiac arrhythmias, cardiomyopathy, laryngomalacia,
tracheomalacia, and child abuse. A cause for the ALTE is not identified in
about half of patients (idiopathic ALTE). Diagnosis is aided by laboratory
tests (blood count, metabolic panel) chest films, electrocardiography, and
physiologic monitoring during sleep, usually including esophageal pH, chest
wall movement, airflow, heart rate, and oximetry. If no clear cause is found,
cardiorespiratory monitoring is often continued in the home setting.
See Fundamentals of Sleep Technology, 2nd edition, Chapter 45, pages 525–526.
7. Risk factors for obstructive sleep apnea (OSA) in a child include obesity,
craniofacial anomalies, enlarged tonsils, and neuromuscular disorders. The
child can present with snoring, nonrestorative sleep, failure to thrive, and
abnormal daytime behavior. In addition to snoring, he or she may be
observed to snort and gasp during sleep, and may have nighttime
awakenings, enuresis, diaphoresis, and apnea. Daytime symptoms include
irritability, hyperactivity, poor school performance, mouth breathing,
headaches, and sleepiness. Obstructive sleep apnea increases the likelihood
of diabetes and the metabolic syndrome, independent of obesity.
Adenotonsillectomy is the most common treatment for children with OSA.
See Fundamentals of Sleep Technology, 2nd edition, Chapter 45, page 526.

8. D, All of the above.


See Fundamentals of Sleep Technology, 2nd edition, Chapter 45, page 525.

9. False.
See Fundamentals of Sleep Technology, 2nd edition, Chapter 45, page 526.

10. B, Take Charlie to the emergency room for evaluation.


See Fundamentals of Sleep Technology, 2nd edition, Chapter 45, page 526.
CHAPTER
40
Nonrespiratory Pediatric Sleep
Disorders
CHRISTOPHER A. COOK

NOTE: This chapter corresponds to Chapter 46 in Fundamentals of Sleep


Technology, 2nd edition.

1. Insufficient quantity or quality of sleep can result in all of the following


EXCEPT:
A. Nocturnal enuresis
B. Temper tantrums
C. Poor concentration
D. Hyperactivity

2. Match the normal sleep duration per 24 hours with the pediatric age group:
Newborn (0 to 2 months) A. 9 to 12 hours and naps 2 to 4.5
hours
Infant (2 to 12 months) B. 10 to 11 hours
Toddler (Age 12 months to 3 C. 12 to 13 hours
Years)
Preschool-aged (Age 3 to 5) D. 11 to 12 hours
School-aged (Age 5 to 11) E. 9 to 9.5 hours
Adolescents (Age 12 to 18) F. 16 to 20 hours


3. Match the sleep disorder with the pediatric age group:
Newborn and infants A. Narcolepsy
Toddler and preschool-aged B. Sleepwalking and Sleep terrors
School-aged C. Sleep-onset association disorder
Adolescents D. Colic


4. The initial assessment of a pediatric sleep disturbance will include all of the
following EXCEPT:
A. Detailed sleep history, from the parents
B. Polysomnography
C. Physical examination
D. Medication history

DISCUSSION QUESTIONS

5. Specific sleep disorders are more prevalent in different age groups. List the
sleep disorders that are most commonly associated with each of the various
pediatric age groups.

6. Clinical observations made by the sleep technologist during


polysomnography are valuable in the diagnosis of pediatric sleep disorders.
Describe the different observation techniques required for the various
pediatric age groups.

CASE STUDY NO. 1

An 11-year-old girl arrives at the sleep clinic with her parents. She has difficulty
falling asleep until after midnight and has to be out of bed by 6:30 AM to get to
school on time. On weekends, she sleeps until about 8:30 AM and awakens
spontaneously. She has a long history of behavioral problems and hyperactivity,
fairly well controlled by medication. She is a very restless sleeper and has
complained about leg discomfort at night for 8 years. Her parents describe a
clear circadian pattern with increased restlessness in the evening. She also
admits that she has difficulty keeping her legs still in class, reporting that she
“just has to move.” Similar symptoms have been described in her mother, aunt,
and grandmother. She is diagnosed with attention deficit hyperactivity disorder
(ADHD) and is prescribed a stimulant medication. Her serum ferritin level is 52
ng/mL (within normal limits).

7. What is the primary sleep complaint?


A. Restless legs syndrome (RLS)
B. Attention deficit hyperactivity disorder (ADHD)
C. Obstructive sleep apnea (OSA)
D. Insomnia

8. The patient's family history includes which of the following sleep disorders?
A. Restless legs syndrome (RLS)
B. Attention deficit hyperactivity disorder (ADHD)
C. Circadian rhythm sleep disorder
D. Insomnia

9. What sleep disorder needs to be treated to address her difficulty falling


asleep?
A. Attention deficit hyperactivity disorder (ADHD)
B. Irregular sleep–wake disorder
C. Delayed sleep phase syndrome
D. Restless legs syndrome (RLS)

CASE STUDY NO. 2

A 3-year-old girl presents with complaints of daytime sleepiness. She has a


regular bedtime at 8 PM but only falls asleep when watching her favorite movie.
If she wakes up during the night, her mother restarts the movie, and she returns
to sleep after watching for 20 to 30 minutes. This pattern may repeat itself
several times during the night. She stayed with her grandmother during the
previous summer. Unfortunately, the grandmother does not have the movie, and
she hardly slept during the 1st week with her. She slept better by the 2nd week.
The patient returns to her previous routine after returning home, and her
complaints of daytime sleepiness continued.

10. What sleep disorder needs to be treated to address her difficulty falling
asleep?
A. Attention deficit hyperactivity disorder
B. Delayed sleep phase syndrome
C. Inadequate sleep hygiene
D. Sleep-onset association disorder

11. What has caused this sleep disorder to develop?


A. Absence of the grandmother
B. Mother's refusal to show the movie
C. Sleep routine was established for the patient
D. Separation anxiety

ANSWERS

1. A, Nocturnal enuresis.
See Fundamentals of Sleep Technology, 2nd edition, Chapter 46.
2.

Newborn (0 to 2 months) F, 16 to 20 hours


Infant (2 to 12 months) A, 9 to 12 hours and naps 2 to 4.5 hours
Toddler (Age 12 months to 3 years) C, 12 to 13 hours
Preschool-aged (Age 3 to 5) D, 11 to 12 hours
School-aged (Age 5 to 11) B, 10 to 11 hours
Adolescents (Age 12 to 18) E, 9 to 9.5 hours
See Fundamentals of Sleep Technology, 2nd edition, Chapter 46, pages 531–533.
3.

Newborn and infants D, Colic


Toddler and preschool-aged C, Sleep-onset association disorder
School-aged B, Sleepwalking and sleep terrors
Adolescents A, Narcolepsy
See Fundamentals of Sleep Technology, 2nd edition, Chapter 46, pages 531–533.

4. B, Polysomnography.
See Fundamentals of Sleep Technology, 2nd edition, Chapter 46, page 534.
5. Some of the most frequently encountered causes of sleep-onset insomnia and
prolonged nocturnal awakenings in children are:
1. Sleep-onset association disorder (toddlers)
2. Delayed sleep phase circadian rhythm disorder (adolescents)
3. Inadequate sleep hygiene (adolescents)
4. Periodic limb movement disorder (PLMD) and restless legs syndrome
(RLS) (school-aged children)
5. Limit-setting behaviors (toddlers)
6. Obstructive sleep apnea syndrome (OSAS) (infants to adolescents)
See Fundamentals of Sleep Technology, 2nd edition, Chapter 46, pages 531–533.

6. The technologist serves as the eyes and ears of the sleep physician; thus,
thorough documentation of all events and behavior is essential. The
technologist must review the initial patient assessment, including sleep
history, medical history, family history, psychosocial history, physical
examination, and behavioral assessment. This information will direct the
sleep technologist to focus on the observations most valuable during the
sleep study. The sleep technologist's written observations should provide
information regarding what the child feels, believes is happening, and needs,
and whether the family can support these needs. The technologist should
document anything out of the ordinary such as unusual behavior, head
banging or rocking, sleepwalking, prolonged crying, hypersomnolence,
hypersexuality, hallucinations, sleep paralysis or cataplexy, unusual behavior
of a family member (swearing, yelling, and rough handling), unusual eating
habits (eating during sleep time and overeating), unusual sleeping positions
(arched back and sitting up), and unusual verbalization (screaming,
prolonged crying, and words inappropriate for age). Signs of abuse (bruises,
black eyes), enuresis (sleep stage and parent and child's reactions), and
numerous visits to the bathroom or resistance to bedtime should also be
reported.
See Fundamentals of Sleep Technology, 2nd edition, Chapter 46, pages 533–534.

7. D, Insomnia.
See Fundamentals of Sleep Technology, 2nd edition, Chapter 46, page 535.

8. A, Restless legs syndrome (RLS).


See Fundamentals of Sleep Technology, 2nd edition, Chapter 46, page 535.

9. D, Restless legs syndrome (RLS).


See Fundamentals of Sleep Technology, 2nd edition, Chapter 46, page 535.
10. D, Sleep-onset association disorder.
See Fundamentals of Sleep Technology, 2nd edition, Chapter 46, page 534.

11. C, Sleep routine was established for the patient.


See Fundamentals of Sleep Technology, 2nd edition, Chapter 46, page 534.
CHAPTER
41
Interventions in the Pediatric Sleep
Laboratory
LAREE FORDYCE

NOTE: This chapter corresponds to Chapter 48 in Fundamentals of Sleep


Technology, 2nd edition.

CASE STUDY NO. 1

A 10-year-old boy with severe obstructive sleep apnea (OSA) has enuresis for 8
months. He is not taking any medications. BMI is 33 kg/m2. Polysomnographic
tracing in Figure 41-1 shows continuous positive airway pressure (CPAP)
titration at 13 cm H2O.
Figure 41-1

1. What is the best course of action at this time?


A. Stay at 13 cm H2O CPAP setting.
B. Switch to bilevel positive airway pressure (BPAP) titration.
C. Increase CPAP setting to 14 cm H2O.
D. Add supplemental O2.

2. In switching to bilevel positive airway pressure (BPAP) titration, which


setting should be chosen?
A. EPAP 13, IPAP 17 cm H2O
B. EPAP 4, IPAP 8 cm H2O
C. EPAP 5, IPAP 9 cm H2O
D. EPAP 9, IPAP 13 cm H2O

CASE STUDY NO. 2

A 13-year-old girl with a BMI of 47 kg/m2 demonstrates a respiratory pattern


similar to the polysomnographic tracing in Figure 41-2.
Figure 42-1

3. What is the respiratory pattern in this tracing?


A. Biot's breathing
B. Tachypnea
C. Cheyne-Stokes respiration
D. Paradoxical respiration

4. What is the most likely diagnosis?


A. Obstructive sleep apnea
B. Central sleep apnea
C. Hypoventilation
D. Diaphragm paralysis

5. In Figure 41-2, continuous positive airway pressure (CPAP) has been titrated
to 15 cm H2O CPAP pressure. What is the best intervention at this time?
A. Increase CPAP setting to 16 cm H2O.
B. Switch to bilevel positive airway pressure (BPAP) titration.
C. Add supplemental oxygen.
D. Continue at current CPAP setting.
MULTIPLE CHOICE QUESTIONS

6. Which of the following is a desired goal of positive airway pressure (PAP)


titration?
A. Respiratory disturbance index (RDI) < 5 events per hour for ≥ 10
minutes including an REM supine period
B. RDI < 5 events per hour for ≥ 15 minutes including an REM supine
period
C. RDI < 5 events per hour for ≥ 5 minutes including an REM period
D. RDI < 5 events per hour for ≥ 15 minutes

7. Which of the following is a desired goal of positive airway pressure (PAP)


titration?
A. Respiratory disturbance index (RDI) > 10 events per hour but RDI
reduced by 75% from baseline
B. RDI > 10 events per hour but RDI reduced by 85% from baseline
C. RDI > 5 events per hour but RDI reduced by 90% from baseline
D. RDI > 5 events per hour but RDI reduced by 95% from baseline

8. Which of the following is the most common problem experienced with


children on continuous positive airway pressure (CPAP) therapy?
A. Pressure intolerance
B. Poor-fitting mask
C. Noise from device
D. Compliance

9. Noninvasive ventilation is indicated in children with which of the following?


A. Obstructive sleep apnea
B. Hypoxia
C. Hypoventilation
D. Central sleep apnea

10. In a child with high transcutaneous carbon dioxide (ptcCO2), what should be
done during noninvasive ventilation?
A. Increase expiratory positive airway pressure (EPAP).
B. Increase inspiratory positive airway pressure (IPAP).
C. Decrease EPAP.
D. Decrease IPAP.
11. Setting a backup respiratory rate is used to improve ventilation and decrease
PaCO2. How should a backup rate be set compared to a child's spontaneous
respiratory rate?
A. 1 to 2 breaths per minute faster
B. 1 to 2 breaths per minute slower
C. 2 to 3 breaths per minute faster
D. Same as spontaneous rate

12. What is the best method of delivering oxygen therapy for a child requiring
greater than 3 L/min?
A. Nasal cannula
B. Face mask
C. Full-face mask
D. O2 entrainment port

DISCUSSION QUESTION

13. Can a large nasal mask be used as a substitute for a full-face mask for a small
child? Explain your answer.

ANSWERS

1. C, Increase CPAP setting to 14 cm H2O. The patient needs higher pressures to


keep the airway open.
See Fundamentals of Sleep Technology, 2nd edition, Chapter 48, pages 555–556.

2. C, EPAP 5, IPAP 9 cm H2O. Minimum delta pressure (difference between


inspiratory (IPAP) and expiratory (EPAP) positive airway pressure) is 4 cm
H2O. Titrate EPAP for obstructive events, and increase IPAP for hypopneas
and residual events.
See Fundamentals of Sleep Technology, 2nd edition, Chapter 48, page 559.

3. B, Tachypnea.
See Fundamentals of Sleep Technology, 2nd edition, Chapter 48, page 562.

4. A, Obstructive sleep apnea.


See Fundamentals of Sleep Technology, 2nd edition, Chapter 48.

5. A, Increase CPAP setting to 16 cm H2O.


See Fundamentals of Sleep Technology, 2nd edition, Chapter 48, page 556.

6. B, RDI < 5 events per hour for ≥ 15 minutes including an REM supine period.
See Fundamentals of Sleep Technology, 2nd edition, Chapter 48, page 556.

7. A, Respiratory disturbance index (RDI) > 10 events per hour but RDI reduced
by 75% from baseline.
See Fundamentals of Sleep Technology, 2nd edition, Chapter 48, page 556.

8. B, Poor-fitting mask. There is a limited choice of pediatric masks. Mask


fitting is especially difficult in children with facial anomalies.
See Fundamentals of Sleep Technology, 2nd edition, Chapter 48, page 557.

9. C, Hypoventilation.
See Fundamentals of Sleep Technology, 2nd edition, Chapter 48, page 558.

10. B, Increase inspiratory positive airway pressure (IPAP).


See Fundamentals of Sleep Technology, 2nd edition, Chapter 48, page 556.

11. B, 1 to 2 breaths per minute slower. A slower backup rate allows a child to
trigger positive airway pressure (PAP) device with his or her own breaths.
See Fundamentals of Sleep Technology, 2nd edition, Chapter 48, page 560.

12. B, Face mask. Nasal cannula can be used for administration of oxygen up to 2
lpm; at higher liter flow, a face mask is preferred.
See Fundamentals of Sleep Technology, 2nd edition, Chapter 48, page 562.

13. No. A full-face mask has an antiasphyxiation valve. Nasal masks do not have
this valve, and using a nasal mask could be dangerous to a small child.
See Fundamentals of Sleep Technology, 2nd edition, Chapter 48, page 557.
APPENDIX I ALGORITHMS
RICHARD S. ROSENBERG

1. Identify the artifact present in the frontal EEG channel of Figure I-1.

2. Identify the artifact present in all EEG channels in Figure I-2.


3. Identify the artifact present in all EEG, EOG, and ECG channels in Figure I-
3.

4. Identify the artifact present in all EEG channels in Figure I-4.


5. Identify the artifact present in the right EOG (topmost) channel of Figure I-5.

6. Identify the arrhythmia in the box (10-second sample) in Figure I-6.


7. A 77-year-old woman with severe obstructive sleep apnea returns to the sleep
center for CPAP titration. Her oxygen saturation is at 93% while she was
awake but falls to 82% at sleep onset. Signal quality is adequate.
Intermittent apnea persists during the initial stages of titration but resolves at
a pressure of 9 cm H2O. At this pressure level, snoring is absent, and the
CPAP flow waveform is well rounded and shows no signs of flow limitation.
Her saturation averages 85% during NREM sleep and decreases to 81%
during a brief episode of REM sleep. In accordance with center policy, you
begin oxygen therapy at 1 L/min and observe changes in oxygen saturation.
At this level, saturation increases to 86%. What are your next steps?
8. A 57-year-old man with episodes of moving and shouting during sleep is
accompanied by his wife during the clinic visit. His wife notes that he
thrashes about during the night and had hit her on several occasions. Once,
he got out of bed, ran to the dresser and slammed it with his fists several
times. The mirror fell on him and broke. He suffered several lacerations and
had to brought to an emergency room. The events generally occur late at
night, typically around 3:00 AM. He has some recall of the events, and in this
instance, he remembered vaguely a dream in which he is at a hat shop and is
enraged because the salesman refuses to sell him a particularly attractive
Stetson. He has no other sleep complaints. On rare occasions, he has noted
daytime sleepiness. He is otherwise alert and functions well at work. He has
recently begun complaining of difficulty getting out of his reading chair. His
usual bedtime is 10:30 PM, and he falls asleep quickly. He wakes at 6:00 AM
with an alarm. He does not snore, and no symptoms consistent with restless
legs syndrome, narcolepsy, or insomnia. He is not obese. What is the most
likely diagnosis?
9. A 7-year-old boy has frequent episodes of waking with a piercing shriek.
These episodes begin shortly after sleep onset and can recur two or three
times before he settles down for the night. They began at age 3 and have
worsened over time. They generally do not occur after midnight. At the start
of each episode, he sits up in bed, looks frightened, and may wave his hands
in front of his face. He then screams for about a minute followed by several
minutes of uncontrollable sobbing. He does not get out of bed during these
episodes. He is inconsolable, and attempts to restrain him only result in
more thrashing and greater agitation. The events end as abruptly as they
begin with a rapid return to sleep. In the morning, he has no recall of the
events. What is the diagnosis?
10. A 27-year-old woman complains of excessive daytime sleepiness. She is a
junior partner at a prestigious law firm and is concerned about a lack of
energy at work. She takes naps in her car during lunch hour, but this has not
resolved her problem. Her symptoms started with her promotion 6 months
ago. Her usual bedtime is 1:00 AM, and she frequently works in the
evenings. At times, she wakes during the night and reads a brief or
researches on the Internet to help her become drowsy again. She wakes at
6:00 AM with an alarm. She denies snoring, restless legs, cataplexy, vivid
dreams, and hypnagogic hallucinations. She is not obese and has a
Mallampati grade of 2. What is responsible for this patient’s symptoms?
11. A 42-year-old man presents with complaint of excessive daytime sleepiness.
He is a salesman and frequently takes naps in-between calling on clients.
His problem began in high school and has worsened with age. He uses
caffeine and occasional energy drinks with little relief in his sleepiness. His
bedtime is at 10:00 PM, and he falls asleep quickly. He wakes at 7:00 AM
with an alarm. He does not snore. His wife says that he is a quiet sleeper and
he does not move much during the night. He has several episodes of
excessive sleepiness during the day and tries to time his appointments
around these. His naps are somewhat refreshing, although he feels he is
rarely fully alert during the day. He does not have symptoms of restless legs
syndrome, and he denies cataplexy, vivid dreams, and hypnagogic
hallucinations. He is not obese. A polysomnogram is performed and shows
normal breathing during sleep, no periodic limb movements, and high sleep
efficiency. REM onset occurs at 102 minutes. An MSLT shows an average
sleep latency of 4.5 minutes with no sleep-onset REM episodes. What is the
most likely cause for his daytime sleepiness?

ANSWERS
1. Use the algorithm in Figure I-7 to identify the artifact in the frontal EEG
channel.
See Fundamentals of Sleep Technology, 2nd edition, Algorithms, page 621.

2. Use the algorithm in Figure I-8 to identify the artifact in all EEG channels.

See Fundamentals of Sleep Technology, 2nd edition, Algorithms, page 621.

3. Use the algorithm in Figure I-9 to identify the artifact in all EEG channels,
EOG and ECG.

See Fundamentals of Sleep Technology, 2nd edition, Algorithms, page 621.

4. Use the algorithm in Figure I-10 to identify the artifact in all EEG channels.
See Fundamentals of Sleep Technology, 2nd edition, Algorithms, page 621.

5. Use the algorithm in Figure I-11 to identify the artifact in the right EOG
(topmost) channel.

See Fundamentals of Sleep Technology, 2nd edition, Algorithms, page 623.

6. Use the algorithm in Figure I-12 to identify the arrhythmia in the box (10-
second sample).
See Fundamentals of Sleep Technology, 2nd edition, Algorithms, page 625.

7. Use the algorithm in Figure I-13 to determine the appropriate approach for
this patient.
See Fundamentals of Sleep Technology, 2nd edition, Algorithms, page 628.

8. Use the algorithm in Figure I-14 to determine the appropriate diagnosis for
this patient.
See Fundamentals of Sleep Technology, 2nd edition, Algorithms, page 629.

9. Use the algorithm in Figure I-15 to determine the appropriate diagnosis for
this patient.
See Fundamentals of Sleep Technology, 2nd edition, Algorithms, page 629.

10. Use the algorithm in Figure I-16 to determine the appropriate diagnosis for this patient.
See Fundamentals of Sleep Technology, 2nd edition, Algorithms, page 630.

11. Use the algorithm in Figure I-17 to determine the appropriate diagnosis for this patient.
See Fundamentals of Sleep Technology, 2nd edition, Algorithms, page 630.
APPENDIX II Glossary
RITA BROOKS

DISEASE STATES

Match the disease state to the definition.

1. Acromegaly A. A chronic metabolic disorder in which there is


excessive growth hormone production after the
skeleton and other organs have finished growing.
2. Metabolic B. A rare, chronic disorder caused by adrenal gland
syndrome failure resulting in insufficient production of adrenal
hormones including cortisol, aldosterone, androgens
(male), and estrogen (female).
3. Gastroesophageal C. An inflammatory disease of the airway characterized
reflux by episodic dyspnea and wheezing, reversible
episodes of bronchoconstriction, and airway
hyperreactivity to a variety of specific and
nonspecific stimuli.
4. Hyperthyroidism D. Collapse of a lobe or segment of the lung or the entire
lung that prevents the exchange of oxygen and carbon
dioxide.
5. Parkinson disease E. Occurs when cardiac dysfunction requires the body to
make compensatory changes to maintain adequate
cardiac output; failure of the heart to pump
effectively.
6. Addison disease F. A chronic condition associated with abnormally high
levels of sugar (glucose) in the blood that results from
defects in the body's ability to produce and/or use
insulin.
7. Enuresis G. Recurrent, involuntary bedwetting occurring during
sleep in a child older than 5 years.
8. Asthma H. A backflow of gastric acid and other gastric contents
into the esophagus due to incompetent barriers at the
gastroesophageal junction.
9. Neuromuscular I. Results from an overactive thyroid and excessive
disorder thyroid hormone production. Symptoms include
fatigue, difficulty sleeping, mood impairment,
difficulty concentrating, muscle weakness,
tachycardia, and heat intolerance.
10. Atelectasis J. A constellation of abnormalities with symptoms that
include central obesity with fat deposits centered on
the abdomen, glucose intolerance/insulin resistance,
hypertension, hypercholesterolemia, and blood-
clotting abnormalities.
11. Congestive heart K. Commonly known as a heart attack.
failure
12. Restrictive lung L. A group of disorders including muscular dystrophy,
disease myotonic dystrophy, amyotrophic lateral sclerosis,
poliomyelitis, and myasthenia gravis.
13. Diabetes mellitus M. A progressive degenerative movement disorder with
features that include resting tremor, bradykinesia
(slowness of movement), rigidity, and gait
disturbance.
14. Seizure N. A condition in which the kidneys fail to adequately
filter toxins and waste products from the blood.
15. Myocardial O. A disorder characterized by reduced lung volumes,
infarction either because of an alteration in the lung parenchyma
or due to diseases of the chest wall, pleura, or thoracic
neuromuscular apparatus.
16. Renal disease P. A sudden abnormal discharge of electrical activity in
the brain that usually affects how a person acts or
feels for a short time.

PEDIATRICS

Fill in the blank:

17. A sleep state in infants that is distinguished by eye movements, fairly rapid
irregular breathing, occasional body movements, vocalizations and facial
movements in the way of smiles, frowns, and sucking is known as
______________.

18. A childhood parasomnia that is classified as a rhythmic movement disorder,


______________ typically involves the entire body while on the hands and
knees with movements that may persist into NREM sleep or may occur
following arousals or during waking from sleep.

19. The weeks from the day of conception to the day of delivery determine
______________.

20. The weeks from the 1st day of the last normal menstrual period to the day
of delivery determine ______________.

21. A transitional sleep state seen in infants at sleep onset, during arousals, or
when the infant is transitioning between active and quiet sleep is called
______________.

22. ______________ is a childhood disorder characterized by difficulty


initiating sleep where the child stalls or refuses to go to sleep at the set
bedtime.

23. A sleep state in infants that is distinguished by slower, more rhythmic


breathing, little body movement, and no eye movements is called
______________.

TECHNICAL

Match the term to its definition.
24. Common A. A measure of a material's ability to conduct an electric
mode rejection current.
25. Impedance B. A unit of measure of electric current.
26. Conductivity C. The ratio of input voltage (μV) and output amplitude
(mm) (V/A).
27. Biocalibrations D. Opposition to the flow of alternating current by the
combination of resistance and capacitance in an
electrical circuit.
28. Notch filter E. The vertical height of a wave, representing the electrical
voltage of the wave.
29. Ampere F. Prevents line-frequency interference (50 or 60 Hz) by
providing a conductive pathway from the patient to
ground via the recording system.
30. Patient G. A series of exercises performed prior to initiating a study
grounding and at the end of a study, to verify correct input
derivations and signal quality.
31. Sensitivity H. The cancellation of unwanted voltages common to both
input electrodes.
32. Amplitude I. A specialized cutoff filter that attenuates or eliminates a
designated frequency, usually 50 or 60 Hz.
33. Frequency J. The period or width of the wave expressed as Hertz (Hz)
or cycles per second (CPS).

TECHNICAL

Fill in the blank:

34. A theory that states that the minimum sampling rate must be twice the rate of
the highest frequency sampled in order to adequately resolve the signal and
prevent aliasing is known as the ______________.

35. A ______________ filter is used to attenuate signals below the cutoff


frequency. At a particular setting, frequencies at or below the cutoff
frequency will be increasingly attenuated depending on the frequency
response curve.

36. An ______________ is a measure of the resistance to the flow of electricity.


37. A noninvasive method for assessing alveolar hypoventilation by monitoring


of the concentration of CO2 in the exhaled breath is known as
______________.

38. A ______________ filter is used to attenuate signals above the cutoff


frequency. At a particular setting, frequencies at or above the cutoff
frequency will be increasingly attenuated depending on the frequency
response curve.

39. An automatic, peak flow– or minute ventilation–targeted device that


performs breath-to-breath analysis and adjusts pressure settings accordingly
to maintain a regular breathing pattern is called ______________.

CARDIAC

Fill in the blank:
40. The time from the onset of the P wave to the beginning of the QRS complex
is known as the ______________

41. The ______________ is a rhythmic pattern that is initiated when an


electrical impulse is conducted through the heart muscle.

42. The ECG waveform that represents ventricular depolarization is the


______________.

43. The active contracting phase of the heartbeat is known as ______________.

44. The ECG waveform that represents ventricular repolarization is the


______________.

45. The time when the heart is in a state of relaxation is known as


______________.

46. Any variation from the normal rhythm in the heartbeat is called a(an)
______________.

47. The ECG waveform that represents atrial depolarization is the


______________.

CARDIAC

Match the term to its definition.

48. Escape beat A. A heart arrhythmia in which an abnormal heartbeat


occurs with every other normal concurrent beat.
49. Bigeminy B. A rhythm lasting for greater than 3 consecutive beats at
a rate greater than 100 per minute with a QRS duration
of ≥120 ms.
50. Sinus pause C. An interruption of cardiac rhythm lasting more than 3
seconds.
51. Narrow D. Conditions where the heart rhythm is irregular, either
complex too fast (tachycardia) or too slow (bradycardia).
tachycardia
52. Junctional E. An irregularly irregular ventricular rhythm associated
arrhythmias with replacement of consistent P waves by rapid
electrical oscillations.
53. Bradycardia F. A cardiac arrhythmia characterized by more atrial
contractions than ventricular beats with atrial rates of
between 240 and 400 beats per minute.
54. Asystole G. A rhythm lasting for greater than 3 consecutive beats at
a rate greater than 100 per minute with a QRS duration
of ≥120 ms.
55. Atrial flutter H. A missed pacing stimulus from the SA node producing
a pause during which the heart is electrically silent.
56. Dysrhythmias I. Occurs when atrial depolarizations fail to reach the
ventricles or when atrial depolarization is conducted
with a delay.
57. Ventricular J. Occur if the AV nodal tissue fires prematurely or if the
fibrillation atrial mechanism for initiating the cardiac cycle fails.
These arrhythmias feature normal-appearing QRS
complexes; when P waves are seen, they will be
inverted and can occur before or after the QRS
complex.
58. Atrial K. An EKG rhythm of less than 60 beats per minute during
fibrillation wake and less than 40 beats per minute during sleep for
ages 6 years through adult.
59. Premature atrial L. A delayed heartbeat originating from an ectopic focus
contraction instead of the sinoatrial node.
60. Wide complex M. A condition in which there is uncoordinated contraction
tachycardia of the cardiac muscle of the ventricles in the heart,
making the contractions ineffective.
61. Atrioventricular N. A common arrhythmia that produces premature P
(AV) block waves, with normal QRS morphology and normal T
waves.
62. Sinus O. An ECG rhythm during sleep that reflects a sustained
tachycardia sinus heart rate of greater than 90 beats per minute for
adults.
63. Ventricular P. A common arrhythmia that is produced when a
tachycardia contraction signal originates in the ventricle of the
heart. This arrhythmia has an absent P wave and an
early and wide and/or aberrant QRS with a
compensatory pause.
64. Premature Q. A faster than normal rhythm that is associated with the
ventricular generation of electrical impulses within the ventricles
contraction and is characterized by an electrocardiogram having a
broad QRS complex.

ANATOMY AND PHYSIOLOGY



Match the term to its definition.

65. Pharynx A. Portion of the brain where most respiratory and


cardiovascular control networks reside. Located just
anterior to the spinal cord and composed of the
medulla and pons.
66. Melatonin B. Small hollow sacs at the end of the respiratory tree that
are involved in gas exchange.
67. Alveoli C. A hormone released by the cortex of the adrenal gland
when an individual experiences stress.
68. Hypoxic drive D. The part of the brain that integrates sensory input and
stimulates arousal.
69. Cortisol E. The primary respiratory muscle; a dome-shaped sheet
of skeletal muscle that separates the thoracic cavity
from the abdominal cavity and is innervated by the
phrenic nerve.
70. Ventilation F. A section of the upper airway behind the nasal
turbinates and the tongue. It is a common site of airway
collapse during sleep.
71. Hypoxemia G. The innermost layer of the heart that extends outward
to include the valves of the heart; a sheet of
endothelium resting on a thin layer of connective
tissue.
72. Reticular H. A form of respiratory drive, which regulates the
formation respiratory cycle.
73. Inspiration I. Controls the circadian rhythm. These nuclei are located
in the anterior hypothalamus.
74. Diaphragm J. The outermost layer of the heart that contains an outer
fibrous connective tissue and an inner serous
pericardium that functions as a protective layer.
75. Endocardium K. The process of carbon dioxide (CO2) removal and
replenishment of oxygen (O2) in the body.
76. Suprachiasmatic L. A secretory product of the pineal gland that maintains
nucleus the circadian rhythm and regulates other hormones.
77. Tidal volume M. Unusually slow and shallow breathing that can lead to
an increase of CO2 in the blood.
78. Epicardium N. The process of moving air in and out of the lungs.
79. Hypoxia O. An abnormally low amount of oxygen in the blood
(usually refers to arterial blood).
80. Gas exchange P. The area of the upper airway between the trachea and
the pharynx.
81. Hypoventilation Q. The movement of atmospheric air into the airways and
lungs.
82. Larynx R. Oxygen levels below the control point, signaling the
respiratory control system to compensate with
increased ventilation. Also a deficiency of oxygen
reaching the tissues of the body.
83. Brainstem S. The lowermost anatomical region of the brain, just
anterior to the spinal cord. It is the location of the main
respiratory and cardiovascular control systems.
84. Medulla T. The volume of a single normal breath. Normal values
in an adult are 400 to 600 mL.

SLEEP AND SLEEP DISORDERS



Match the term to its definition.

85. Somniloquy A. A breathing pattern of rhythmic waxing and


waning of depth of breaths and regularly
recurring apneic periods.
86. Complex sleep apnea B. Individuals with this disorder go to bed later
and awaken later than what is considered
normal.
87. Somnambulism C. A sudden, abrupt, striking, and frightening
arousal from slow-wave sleep associated
with profound autonomic activity and
behavioral manifestations of intense fear.
Episodes usually begin suddenly with the
individual sitting upright in bed and
emitting a powerful, piercing scream. The
individual is unresponsive, and efforts to
restrain or console the individual tend to
exacerbate autonomic and motor activity.
88. Cheyne-Stokes respiration D. Ambulation that occurs during sleep.
89. Advanced sleep phase E. The experience of persistent sleepiness that
disorder is not resolved with adequate sleep.
90. Delayed sleep phase F. Individuals with this disorder go to sleep
disorder before the normal bedtime and then awaken
earlier than what is considered normal.
91. Stanford Sleepiness Scale G. A disease state characterized by airflow
limitation that is relatively irreversible.
92. Sleep terrors H. A chronic neurologic disorder characterized
by excessive daytime sleepiness (EDS) and
sudden and uncontrollable attacks of sleep.
93. Excessive daytime I. Respiratory pattern in which the application
sleepiness of CPAP for the treatment of obstructive
sleep apnea elicits central apneas in the
patient.
94. REM sleep behavior J. Neurosensorimotor disorder that
disorder significantly impacts sleep in the first half
of the night as patients often must stretch,
move, or walk to provide relief of
unpleasant sensations in the legs, resulting
in significant disruption to sleep quality.
95. Chronic obstructive K. Characterized by a paradoxical increase in
pulmonary disease muscle tone and frequent limb movements
during REM sleep, elaborate, purposeful
movements, vocalizations, and occasionally
violent behaviors such as punching,
kicking, and/or leaping out of bed. This
motor activity is associated with dream
recall as patients seem to be acting out their
dreams.
96. Uvulopalatopharyngoplasty L. Surgical procedure that involves removal of
the uvula, tonsils, and tonsillar pillars along
with the lower part of soft palate.
97. Restless legs syndrome M. A subjective measure of sleepiness rating
that uses a 7-point Likert rating scale to
assess immediate changes in the patient's
level of sleepiness. The rating is from “Not
sleepy at all” to “Extremely sleepy.”
98. Narcolepsy N. Sleep talking; not generally associated with
pathology.

SLEEP AND SLEEP DISORDERS


Fill in the blank:
99. The study of circadian rhythms on physiologic and pathologic events is
known as ______________.

100. A REM period that occurs during an MSLT nap is known as a(an)
______________.

101. ______________ is defined as grinding the teeth and clenching the jaw
during wakefulness and sleep; it is often associated with arousals during
sleep.

102. Sudden and transient episodes of loss of muscle tone triggered by emotion
are called ______________.

103. A sudden shift in EEG frequency lasting for at least 3 seconds with at least
10 seconds of stable sleep preceding the change in N1, N2, N3, or with an
accompanying increase in chin EMG in REM sleep is known as a(an)
______________.

104. A(an) ______________ is an innate, 24-hour cycle of fluctuation in
physiologic and behavioral functions.

105. A self-reported measurement designed to assess improvement of sleep,


based on multiple activities of daily living is known as the
______________.

106. The portion of the total recording time spent asleep, usually expressed as a
percentage, is called ______________.

107. A self-administered questionnaire used to assess a patient's own perception


of their sleepiness, which assists sleep professionals to recognize and
measure excessive daytime sleepiness, is known as the ______________.

108. A rough estimate of tongue size relative to the oral cavity used to assess the
risk of upper airway obstruction is called the ______________.

109. A(an) ______________ can be used to treat snoring and mild-to-moderate


obstructive sleep apnea.

110. The time from lights out to sleep onset (first of any epoch of sleep) is called
______________.

111. Rhythmic movements of the extremities that are measured during sleep that
may or may not have clinical relevance are called ______________.

112. ______________ is an irrational fear of being in a confined or enclosed


space, which can make one feel like they are unable to breathe.

113. Cycles that occur at recurrent intervals of less than 24 hours are defined as
a(an) ______________.

114. The behavioral patterns that are consistent with sleeping well are defined as
______________.

115. A brief occurrence of alpha activity during a stage of sleep is called


______________.

116. ______________ is a treatment for insomnia that uses a paradoxical


approach where less time is spent in bed in an effort to improve sleep.

117. The first epoch of sleep is defined as ______________.


118. ______________ are vivid dreamlike images or sounds that occur at sleep
onset during the progression from wakefulness to sleep.

119. A surgical procedure that moves the upper and lower jaws forward and
enlarges the airway so that soft tissues in the soft palate and tongue are
pulled forward is called ______________.

120. An environmental time cue that sets the body clock is known as a(an)
______________.

121. An insufficient duration of sleep (quantitative), a fragmented or interrupted


sleep period (qualitative), or a combination of both factors is known as
______________.

122. ______________ are vivid dreamlike images or sounds that occur during
the progression from sleep to wakefulness.

123. An inability to move or speak, sometimes accompanied by the sensation of
inability to breathe for a few seconds or a few minutes that occurs during the
transition from wakefulness to sleep or from sleep to wakefulness is called
______________.

ANSWERS

1. A, A chronic metabolic disorder in which there is excessive growth hormone


production after the skeleton and other organs have finished growing.
See Fundamentals of Sleep Technology, 2nd edition, Glossary, page 636.

2. J, A constellation of abnormalities with symptoms that include central obesity


with fat deposits centered on the abdomen, glucose intolerance/insulin
resistance, hypertension, hypercholesterolemia, and blood-clotting
abnormalities.
See Fundamentals of Sleep Technology, 2nd edition, Glossary, page 642.

3. H, A backflow of gastric acid and other gastric contents into the esophagus
due to incompetent barriers at the gastroesophageal junction.
See Fundamentals of Sleep Technology, 2nd edition, Glossary, page 640.

4. I, Results from an overactive thyroid and excessive thyroid hormone


production. Symptoms include fatigue, difficulty sleeping, mood impairment,
difficulty concentrating, muscle weakness, tachycardia, and heat intolerance.
See Fundamentals of Sleep Technology, 2nd edition, Glossary, page 640.

5. M, A progressive degenerative movement disorder with features that include


resting tremor, bradykinesia (slowness of movement), rigidity, and gait
disturbance.
See Fundamentals of Sleep Technology, 2nd edition, Glossary, page 643.

6. B, A rare, chronic disorder caused by adrenal gland failure resulting in


insufficient production of adrenal hormones including cortisol, aldosterone,
androgens (male), and estrogen (female).
See Fundamentals of Sleep Technology, 2nd edition, Glossary, page 636.

7. G, Recurrent, involuntary bedwetting occurring during sleep in a child older


than 5 years.
See Fundamentals of Sleep Technology, 2nd edition, Glossary, page 639.

8. C, An inflammatory disease of the airway characterized by episodic dyspnea


and wheezing, reversible episodes of bronchoconstriction, and airway
hyperreactivity to a variety of specific and nonspecific stimuli.
See Fundamentals of Sleep Technology, 2nd edition, Glossary, page 636.

9. L, A group of disorders including muscular dystrophy, myotonic dystrophy,


amyotrophic lateral sclerosis, poliomyelitis, and myasthenia gravis.
See Fundamentals of Sleep Technology, 2nd edition, Glossary, page 642.

10. D, Collapse of a lobe or segment of the lung or the entire lung that prevents
the exchange of oxygen and carbon dioxide.
See Fundamentals of Sleep Technology, 2nd edition, Glossary, page 637.

11. E, Occurs when cardiac dysfunction requires the body to make compensatory
changes to maintain adequate cardiac output; failure of the heart to pump
effectively.
See Fundamentals of Sleep Technology, 2nd edition, Glossary, page 638.

12. O, A disorder characterized by reduced lung volumes, either because of an


alteration in the lung parenchyma or due to diseases of the chest wall, pleura,
or thoracic neuromuscular apparatus.
See Fundamentals of Sleep Technology, 2nd edition, Glossary, page 644.

13. F, A chronic condition associated with abnormally high levels of sugar


(glucose) in the blood that results from defects in the body's ability to produce
and/or use insulin.
See Fundamentals of Sleep Technology, 2nd edition, Glossary, page 639.

14. P, A sudden abnormal discharge of electrical activity in the brain that usually
affects how a person acts or feels for a short time.
See Fundamentals of Sleep Technology, 2nd edition, Glossary, page 645.

15. K, Commonly known as a heart attack.


See Fundamentals of Sleep Technology, 2nd edition, Glossary, page 642.

16. N, A condition in which the kidneys fail to adequately filter toxins and waste
products from the blood.
See Fundamentals of Sleep Technology, 2nd edition, Glossary, page 644.

17. A sleep state in infants that is distinguished by eye movements, fairly rapid
irregular breathing, occasional body movements, vocalizations and facial
movements in the way of smiles, frowns, and sucking is known as active
sleep.
See Fundamentals of Sleep Technology, 2nd edition, Glossary, page 636.

18. A childhood parasomnia that is classified as a rhythmic movement disorder,


body rocking typically involves the entire body while on the hands and knees
with movements that may persist into NREM sleep and may occur following
arousals or during waking from sleep.
See Fundamentals of Sleep Technology, 2nd edition, Glossary, page 637.

19. The weeks from the day of conception to the day of delivery determine
conceptional age.
See Fundamentals of Sleep Technology, 2nd edition, Glossary, page 638.

20. The weeks from the 1st day of the last normal menstrual period to the day of
delivery determine gestational age.
See Fundamentals of Sleep Technology, 2nd edition, Glossary, page 640.

21. A transitional sleep state seen in infants at sleep onset, during arousals, or
when the infant is transitioning between active and quiet sleep is called
indeterminate sleep.
See Fundamentals of Sleep Technology, 2nd edition, Glossary, page 641.

22. Limit setting is a childhood disorder characterized by difficulty initiating sleep


where the child stalls or refuses to go to sleep at the set bedtime.
See Fundamentals of Sleep Technology, 2nd edition, Glossary, page 641.

23. A sleep state in infants that is distinguished by slower, more rhythmic


breathing, little body movement, and no eye movements is called quiet sleep.
See Fundamentals of Sleep Technology, 2nd edition, Glossary, page 644.

24. H, The cancellation of unwanted voltages common to both input electrodes.


See Fundamentals of Sleep Technology, 2nd edition, Glossary, page 638.

25. D, Opposition to the flow of alternating current by the combination of


resistance and capacitance in an electrical circuit.
See Fundamentals of Sleep Technology, 2nd edition, Glossary, page 641.

26. A, A measure of a material's ability to conduct an electric current.


See Fundamentals of Sleep Technology, 2nd edition, Glossary, page 638.

27. G, A series of exercises performed prior to initiating a study and at the end of
a study, to verify correct input derivations and signal quality.
See Fundamentals of Sleep Technology, 2nd edition, Glossary, page 637.

28. I, A specialized cutoff filter that attenuates or eliminates a designated


frequency, usually 50 or 60 Hz.
See Fundamentals of Sleep Technology, 2nd edition, Glossary, page 642.

29. B, A unit of measure of electric current.


See Fundamentals of Sleep Technology, 2nd edition, Glossary, page 636.

30. F, Prevents line-frequency interference (50 or 60 Hz) by providing a


conductive pathway from the patient to ground via the recording system.
See Fundamentals of Sleep Technology, 2nd edition, Glossary, page 643.

31. C, The ratio of input voltage (μV) and output amplitude (mm) (V/A).
See Fundamentals of Sleep Technology, 2nd edition, Glossary, page 645.

32. E, The vertical height of a wave, representing the electrical voltage of the
wave.
See Fundamentals of Sleep Technology, 2nd edition, Glossary, page 636.

33. J, The period or width of the wave expressed as Hertz (Hz) or cycles per
second (CPS).
See Fundamentals of Sleep Technology, 2nd edition, Glossary, page 640.

34. A theory that states that the minimum sampling rate must be twice the rate of
the highest frequency sampled in order to adequately resolve the signal and
prevent aliasing is known as the Nyquist sampling theory.
See Fundamentals of Sleep Technology, 2nd edition, Glossary, page 642.

35. A low-frequency filter is used to attenuate signals below the cutoff frequency.
At a particular setting, frequencies at or below the cutoff frequency will be
attenuated with increasing degree dependent on the frequency response curve.
See Fundamentals of Sleep Technology, 2nd edition, Glossary, page 641.

36. An ohm is a measure of the resistance to the flow of electricity.


See Fundamentals of Sleep Technology, 2nd edition, Glossary, page 642.

37. A noninvasive method for assessing alveolar hypoventilation by monitoring of


the concentration of CO2 in the exhaled breath is known as capnography.
See Fundamentals of Sleep Technology, 2nd edition, Glossary, page 638.

38. A high-frequency filter is used to attenuate signals above the cutoff frequency.
At a particular setting, frequencies at or above the cutoff frequency will be
attenuated with increasing degree dependent on the frequency response curve.
See Fundamentals of Sleep Technology, 2nd edition, Glossary, page 640.

39. An automatic, peak flow– or minute ventilation–targeted device that performs


breath-to-breath analysis and adjusts pressure settings accordingly to maintain
a regular breathing pattern is called adaptive servoventilation.
See Fundamentals of Sleep Technology, 2nd edition, Glossary, page 636.

40. The time from the onset of the P wave to the beginning of the QRS complex is
known as the P–R interval.
See Fundamentals of Sleep Technology, 2nd edition, Glossary, page 644.

41. The cardiac cycle is a rhythmic pattern that is initiated when an electrical
impulse is conducted through the heart muscle.
See Fundamentals of Sleep Technology, 2nd edition, Glossary, page 638.

42. The ECG waveform that represents ventricular depolarization is the QRS
complex.
See Fundamentals of Sleep Technology, 2nd edition, Glossary, page 644.

43. The active contracting phase of the heartbeat is known as systole.


See Fundamentals of Sleep Technology, 2nd edition, Glossary, page 646.

44. The ECG waveform that represents ventricular repolarization is the T wave.
See Fundamentals of Sleep Technology, 2nd edition, Glossary, page 646.

45. The time when the heart is in a state of relaxation is known as diastole.
See Fundamentals of Sleep Technology, 2nd edition, Glossary, page 639.

46. Any variation from the normal rhythm in the heartbeat is called a(an)
arrhythmia.
See Fundamentals of Sleep Technology, 2nd edition, Glossary, page 637.

47. The ECG waveform that represents depolarization is the P wave.


See Fundamentals of Sleep Technology, 2nd edition, Glossary, page 644.

48. L, A delayed heartbeat originating from an ectopic focus instead of the


sinoatrial node.
See Fundamentals of Sleep Technology, 2nd edition, Glossary, page 639.

49. A, A heart arrhythmia in which an abnormal heartbeat occurs with every other
normal concurrent beat.
See Fundamentals of Sleep Technology, 2nd edition, Glossary, page 637.

50. H, A missed pacing stimulus from the SA node producing a pause during
which the heart is electrically silent.
See Fundamentals of Sleep Technology, 2nd edition, Glossary, page 645.

51. G, A rhythm lasting for greater than 3 consecutive beats at a rate greater than
100 per minute with a QRS duration of ≥120 ms.
See Fundamentals of Sleep Technology, 2nd edition, Glossary, page 642.

52. J, Occur if the AV nodal tissue fires prematurely or if the atrial mechanism for
initiating the cardiac cycle fails. These arrhythmias feature normal-appearing
QRS complexes; when P waves are seen, they will be inverted and can occur
before or after the QRS complex.
See Fundamentals of Sleep Technology, 2nd edition, Glossary, page 641.

53. K, An EKG rhythm of less than 60 beats per minute during wake and less than
40 beats per minute during sleep for ages 6 years through adult.
See Fundamentals of Sleep Technology, 2nd edition, Glossary, page 637.

54. C, An interruption of cardiac rhythm lasting more than 3 seconds.


See Fundamentals of Sleep Technology, 2nd edition, Glossary, page 637.

55. F, A cardiac arrhythmia characterized by more atrial contractions than


ventricular beats with atrial rates of between 240 and 400 beats per minute.
See Fundamentals of Sleep Technology, 2nd edition, Glossary, page 637.

56. D, Conditions where the heart rhythm is irregular, either too fast (tachycardia)
or too slow (bradycardia).
See Fundamentals of Sleep Technology, 2nd edition, Glossary, page 639.

57. M, A condition in which there is uncoordinated contraction of the cardiac


muscle of the ventricles in the heart, making the contractions ineffective.
See Fundamentals of Sleep Technology, 2nd edition, Glossary, page 646.

58. E, An irregularly irregular ventricular rhythm associated with replacement of


consistent P waves by rapid electrical oscillations.
See Fundamentals of Sleep Technology, 2nd edition, Glossary, page 637.

59. N, A common arrhythmia that produces premature P waves, with normal QRS
morphology and normal T waves.
See Fundamentals of Sleep Technology, 2nd edition, Glossary, page 644.
60. B, A rhythm lasting for greater than 3 consecutive beats at a rate greater than
100 per minute with a QRS duration of ≥120 ms.
See Fundamentals of Sleep Technology, 2nd edition, Glossary, page 647.

61. I, Occurs when atrial depolarizations fail to reach the ventricles or when atrial
depolarization is conducted with a delay.
See Fundamentals of Sleep Technology, 2nd edition, Glossary, page 637.

62. O, An ECG rhythm during sleep that reflects a sustained sinus heart rate of
greater than 90 beats per minute for adults.
See Fundamentals of Sleep Technology, 2nd edition, Glossary, page 645.

63. Q, A faster than normal rhythm that is associated with the generation of
electrical impulses within the ventricles and is characterized by an
electrocardiogram having a broad QRS complex.
See Fundamentals of Sleep Technology, 2nd edition, Glossary, page 646.

64. P, A common arrhythmia that is produced when a contraction signal originates


in the ventricle of the heart. This arrhythmia has an absent P wave and an
early and wide and/or aberrant QRS with a compensatory pause.
See Fundamentals of Sleep Technology, 2nd edition, Glossary, page 644.

65. F, A section of the upper airway behind the nasal turbinates and the tongue. It
is a common site of airway collapse during sleep.
See Fundamentals of Sleep Technology, 2nd edition, Glossary, page 643.

66. L, A secretory product of the pineal gland that maintains the circadian rhythm
and regulates other hormones.
See Fundamentals of Sleep Technology, 2nd edition, Glossary, page 642.

67. B, Small hollow sacs at the end of the respiratory tree that are involved in gas
exchange.
See Fundamentals of Sleep Technology, 2nd edition, Glossary, page 636.

68. H, A form of respiratory drive, which regulates the respiratory cycle.


See Fundamentals of Sleep Technology, 2nd edition, Glossary, page 641.

69. C, A hormone released by the cortex of the adrenal gland when an individual
experiences stress.
See Fundamentals of Sleep Technology, 2nd edition, Glossary, page 638.

70. N, The process of moving air in and out of the lungs.


See Fundamentals of Sleep Technology, 2nd edition, Glossary, page 646.
71. O, An abnormally low amount of oxygen in the blood (usually refers to
arterial blood).
See Fundamentals of Sleep Technology, 2nd edition, Glossary, page 641.

72. D, The part of the brain that integrates sensory input and stimulates arousal.
See Fundamentals of Sleep Technology, 2nd edition, Glossary, page 644.

73. Q, The movement of atmospheric air into the airways and lungs.
See Fundamentals of Sleep Technology, 2nd edition, Glossary, page 641.

74. E, The primary respiratory muscle; a dome-shaped sheet of skeletal muscle


that separates the thoracic cavity from the abdominal cavity and is innervated
by the phrenic nerve.
See Fundamentals of Sleep Technology, 2nd edition, Glossary, page 639.

75. G, The innermost layer of the heart that extends outward to include the valves
of the heart; a sheet of endothelium resting on a thin layer of connective
tissue.
See Fundamentals of Sleep Technology, 2nd edition, Glossary, page 639.

76. I, Controls the circadian rhythm. These nuclei are located in the anterior
hypothalamus.
See Fundamentals of Sleep Technology, 2nd edition, Glossary, page 646.

77. T, The volume of a single normal breath. Normal values in an adult are 400 to
600 mL.
See Fundamentals of Sleep Technology, 2nd edition, Glossary, page 646.

78. J, The outermost layer of the heart that contains an outer fibrous connective
tissue and an inner serous pericardium that functions as a protective layer.
See Fundamentals of Sleep Technology, 2nd edition, Glossary, page 639.

79. R, Oxygen levels below the control point, signaling the respiratory control
system to compensate with increased ventilation. Also a deficiency of oxygen
reaching the tissues of the body.
See Fundamentals of Sleep Technology, 2nd edition, Glossary, page 641.

80. K, The process of carbon dioxide (CO2) removal and replenishment of oxygen
(O2) in the body.
See Fundamentals of Sleep Technology, 2nd edition, Glossary, page 640.

81. M, Unusually slow and shallow breathing that can lead to an increase of CO2
in the blood.
See Fundamentals of Sleep Technology, 2nd edition, Glossary, page 641.

82. P, The area of the upper airway between the trachea and the pharynx.
See Fundamentals of Sleep Technology, 2nd edition, Glossary, page 641.

83. A, Portion of the brain where most respiratory and cardiovascular control
networks reside. Located just anterior to the spinal cord and composed of the
medulla and pons.
See Fundamentals of Sleep Technology, 2nd edition, Glossary, page 637.

84. S, The lowermost anatomical region of the brain, just anterior to the spinal
cord. It is the location of the main respiratory and cardiovascular control
systems.
See Fundamentals of Sleep Technology, 2nd edition, Glossary, page 642.

85. N, Sleep talking; not generally associated with pathology.


See Fundamentals of Sleep Technology, 2nd edition, Glossary, page 645.

86. I, Respiratory pattern in which the application of CPAP for the treatment of
obstructive sleep apnea elicits central apneas in the patient.
See Fundamentals of Sleep Technology, 2nd edition, Glossary, page 638.

87. D, Ambulation that occurs during sleep.


See Fundamentals of Sleep Technology, 2nd edition, Glossary, page 645.

88. A, A breathing pattern of rhythmic waxing and waning of depth of breaths and
regularly recurring apneic periods.
See Fundamentals of Sleep Technology, 2nd edition, Glossary, page 638.

89. F, Individuals with this disorder go to sleep before the normal bedtime and
then awaken earlier than what is considered normal.
See Fundamentals of Sleep Technology, 2nd edition, Glossary, page 636.

90. B, Individuals with this disorder go to bed later and awaken later than what is
considered normal.
See Fundamentals of Sleep Technology, 2nd edition, Glossary, page 638.

91. M, A subjective measure of sleepiness rating that uses a seven-point Likert


rating scale to assess immediate changes in the patient's level of sleepiness.
The rating is from “Not sleepy at all” to “Extremely sleepy.”
See Fundamentals of Sleep Technology, 2nd edition, Glossary, pages 645–646.
92. C, A sudden, abrupt, striking, and frightening arousal from slow-wave sleep
associated with profound autonomic activity and behavioral manifestations of
intense fear. Episodes usually begin suddenly with the individual sitting
upright in bed and emitting a powerful, piercing scream. The individual is
unresponsive, and efforts to restrain or console the individual tend to
exacerbate autonomic and motor activity.
See Fundamentals of Sleep Technology, 2nd edition, Glossary, page 645.

93. E, The experience of persistent sleepiness that is not resolved with adequate
sleep.
See Fundamentals of Sleep Technology, 2nd edition, Glossary, page 639.

94. K, Characterized by a paradoxical increase in muscle tone and frequent limb


movements during REM sleep, elaborate, purposeful movements,
vocalizations, and occasionally violent behaviors such as punching, kicking,
and/or leaping out of bed. This motor activity is associated with dream recall
as patients seem to be acting out their dreams.
See Fundamentals of Sleep Technology, 2nd edition, Glossary, page 644.

95. G, A disease state characterized by airflow limitation that is relatively


irreversible.
See Fundamentals of Sleep Technology, 2nd edition, Glossary, page 638.

96. L, Surgical procedure that involves removal of the uvula, tonsils, and tonsillar
pillars along with the lower part of soft palate.
See Fundamentals of Sleep Technology, 2nd edition, Glossary, page 646.

97. J, Neurosensorimotor disorder that significantly impacts sleep in the first half
of the night as patients often must stretch, move, or walk to provide relief of
unpleasant sensations in the legs, resulting in significant disruption to sleep
quality.
See Fundamentals of Sleep Technology, 2nd edition, Glossary, page 644.

98. H, A chronic neurologic disorder characterized by excessive daytime


sleepiness (EDS) and sudden and uncontrollable attacks of sleep.
See Fundamentals of Sleep Technology, 2nd edition, Glossary, page 642.

99. The study of circadian rhythms on physiologic and pathologic events is known
as chronobiology.
See Fundamentals of Sleep Technology, 2nd edition, Glossary, page 638.

100. A REM period that occurs during the first 15 minutes of an MSLT nap is
known as a(an) sleep-onset REM period.
See Fundamentals of Sleep Technology, 2nd edition, Glossary, page 645.

101. Bruxism is defined as grinding the teeth and clenching the jaw during
wakefulness and sleep; it is often associated with arousals during sleep.
See Fundamentals of Sleep Technology, 2nd edition, Glossary, page 637.

102. Sudden and transient episodes of loss of muscle tone triggered by emotion are
called cataplexy.
See Fundamentals of Sleep Technology, 2nd edition, Glossary, page 638.

103. A sudden shift in EEG frequency lasting for at least 3 seconds with at least 10
seconds of stable sleep preceding the change in N1, N2, N3, or with an
accompanying increase in chin EMG in REM sleep is known as a(an)
arousal.
See Fundamentals of Sleep Technology, 2nd edition, Glossary, page 637.

104. A(an) circadian rhythm is an innate, 24-hour cycle of fluctuation in


physiologic and behavioral functions.
See Fundamentals of Sleep Technology, 2nd edition, Glossary, page 638.

105. A self-reported measurement designed to assess improvement of excessive


sleepiness, based on multiple activities of daily living is known as the FOSQ
(functional outcomes of sleep questionnaire).
See Fundamentals of Sleep Technology, 2nd edition, Glossary, page 640.

106. The portion of the total recording time spent asleep, usually expressed as a
percentage, is called sleep efficiency.
See Fundamentals of Sleep Technology, 2nd edition, Glossary, page 645.

107. A self-administered questionnaire used to assess a patient's own perception of


their sleepiness, which assists sleep professionals to recognize and measure
excessive daytime sleepiness, is known as the ESS (Epworth Sleepiness
Scale).
See Fundamentals of Sleep Technology, 2nd edition, Glossary, page 639.

108. A rough estimate of tongue size relative to the oral cavity used to assess the
risk of upper airway obstruction is called the Mallampati airway
classification.
See Fundamentals of Sleep Technology, 2nd edition, Glossary, page 642.

109. A(an) oral appliance or dental device can be used to treat snoring and mild-
to-moderate obstructive sleep apnea.
See Fundamentals of Sleep Technology, 2nd edition, Glossary, pages 639, 643.

110. The time from lights out to sleep onset (first of any epoch of sleep) is called
sleep latency.
See Fundamentals of Sleep Technology, 2nd edition, Glossary, page 645.

111. Rhythmic movements of the lower extremities that are measured during sleep
that may or may not have clinical relevance are called periodic leg
movements in sleep.
See Fundamentals of Sleep Technology, 2nd edition, Glossary, page 643.

112. Claustrophobia is an irrational fear of being in a confined or enclosed space,


which can make one feel like they are unable to breathe.
See Fundamentals of Sleep Technology, 2nd edition, Glossary, page 638.

113. Cycles that occur at recurrent intervals of less than 24 hours are defined as
a(an) ultradian rhythm.
See Fundamentals of Sleep Technology, 2nd edition, Glossary, page 646.

114. The behavioral patterns that are consistent with sleeping well are defined as
sleep hygiene.
See Fundamentals of Sleep Technology, 2nd edition, Glossary, page 645.

115. A brief occurrence of alpha activity during a stage of sleep is called alpha
intrusion.
See Fundamentals of Sleep Technology, 2nd edition, Glossary, page 636.

116. Sleep restriction therapy is a treatment for insomnia that uses a paradoxical
approach where less time is spent in bed in an effort to improve sleep.
See Fundamentals of Sleep Technology, 2nd edition, Glossary, page 645.

117. The first epoch of sleep is defined as sleep onset.


See Fundamentals of Sleep Technology, 2nd edition, Glossary, page 645.

118. Hypnagogic hallucinations are vivid dreamlike images or sounds that occur at
sleep onset during the progression from wakefulness to sleep.
See Fundamentals of Sleep Technology, 2nd edition, Glossary, page 640.

119. A surgical procedure that moves the upper and lower jaws forward and
enlarges the airway so that soft tissues in the soft palate and tongue are pulled
forward is called maxillomandibular advancement.
See Fundamentals of Sleep Technology, 2nd edition, Glossary, page 642.
120. An environmental or time cue that sets the body clock is known as a(an)
zeitgeber.
See Fundamentals of Sleep Technology, 2nd edition, Glossary, page 647.

121. An insufficient duration of sleep (quantitative), a fragmented or interrupted


sleep period (qualitative), or a combination of both factors is known as sleep
deprivation.
See Fundamentals of Sleep Technology, 2nd edition, Glossary, page 645.

122. Hypnopompic hallucinations are vivid dreamlike images or sounds that occur
at awakening during the progression from sleep to wakefulness.
See Fundamentals of Sleep Technology, 2nd edition, Glossary, page 640.

123. An inability to move or speak, sometimes accompanied by the sensation of


inability to breathe for a few seconds or a few minutes that occurs during the
transition from wakefulness to sleep or from sleep to wakefulness is called
sleep paralysis.
See Fundamentals of Sleep Technology, 2nd edition, Glossary, page 645.
INDEX

A
Actigraphy
activity levels measure assessment, of pediatric patients collect data
evaluate sleep–wake patterns indication
insomnia evaluation
light sensor
MSLT
PLMD
SE
sleep–wake patterns
Adult sleep scoring
activity identify
alpha activity
alpha rhythm oscillate amplitude necessary
apnea
identify
arousal
diagnostic study
EEG
epoch scoring
epoch stage
K complexes
predominant EEG rhythm sleep stage
slow eye movement
slow wave
activity
oscillate
vertex sharp waves
waveform
Advanced PAP therapy
adaptive servo ventilation devices central apnea pattern
EPAP
symptom and diagnosis
Advanced sleep phase disorder Alertness
ALMA. Alternating leg muscle activation (ALMA) Alternating leg muscle
activation (ALMA) American Academy of Sleep Medicine (AASM) CPAP
guidelines
pediatric polysomnography pediatric scoring
positive airway pressure (PAP) devices scoring manual
sleep manual guidelines Amplitude, digital polysomnography Amy
after BPAP therapy
CPAP setting
RERA
snoring and RERAs
Apnea index (AI)
Apnea–hypopnea index (AHI) Apparent life-threatening events (ALTEs)
diagnostic evaluation
emergency services
risk factor
Artifact
ballistocardiographic
muscle
sweat
Attended video PSG (VPSG) Attention deficit hyperactivity disorder (ADHD)
Auto-titrating positive airway pressure (APAP) therapy B
Ballistocardiographic artifact Benign epilepsy of childhood (BECT) Bilevel
positive airway pressure (BPAP) Biocalibrations
Biopotentials of sleep anatomy and physiology brain nuclei involved
depolarization
EEG activity
EEG waveforms
electrodes application during electroencephalography gas exchange
heart rate
P wave represents
peripheral sensors
QRS complex
retinohypothalamic tract skin, cleaning and preparation specialized
muscle fibers recording
artifacts
calibration
CPAP machine
EOG
mouth, breathe
muscle artifact
physiologic calibrations QRS complex
sweat artifact
Bipolar disorder
manic phase
Blinking
Body, oxygen and gas exchange CaO2 calculation gas transfer
hypoxemia/hypoxia
hypoxia correction
oxygen availability
oxygen content
oxygen-hemoglobin dissociation curve partial pressure
respiration/ventilation Breathing disorders, in children ALTE
AOP
OSA
pediatric obstructive sleep apnea polysomnography
SIDS
C
Capnography
Carbon dioxide production Cardiac anatomy and physiology adapted, from
WPClipart.com afterload
cardiac cycle phases
diastolic pressure
electrical conducting pathway external neural influences gap junctions
heart rate
acceleration of
slow
speed
intercalated discs
intrinsic mechanism
layers of
myocardial contraction normal blood pressure
output
preload
systolic pressure
Cardiac arrhythmias
AASM
scoring manual
sleep manual guidelines atrial flutter
atrioventricular block atrioventricular node passes bundle branch block
cardiac cycle
ECG
estimate heart rate
heart rhythm
heartbeats
junctional rhythm
PVC
QRS complex
scoring arrhythmias
sinus arrhythmia
sinus heart rhythm
ventricular fibrillation Central sleep apnea
adaptive servo ventilation arousals
cause of
central nervous system dysfunction congestive heart failure CPAP
CSR
diagnosis
hypercapnia
hypoventilation
syndromes
positive airway pressure therapy respiratory events
respiratory patterns
treatment-emergent apnea ventilatory response
Cheyne-Stokes respiration (CSR) Chronotherapy
Circadian rhythms and disorders advanced sleep phase disorder amplitude
brain regions
cause
chronotherapy
delayed sleep phase disorder endocrine and body temperature regulation
entraining rhythms
examples of human
free-running disorder
intrinsically photosensitive retinal jet lag disorder
light exposure
light-blocking devices light–dark cycle
master circadian clock location melatonin
natural sunlight exposure night shift worker
pathways
patient's sleep and wakefulness phase
delay and advance
markers
response curve
physiologic patterns
problems
retinal ganglion cells shift work disorder
sleep disorders
sleep quality and quantity sleep–wake
diary
pattern
suprachiasmatic nucleus tau
wrist actigraphy
zeitgeber
influence
Conductivity, digital polysomnography Continuous positive airway pressure
(CPAP) AASM
BPAP
cause
pediatric patient
Continuous positive airway pressure therapy Continuous spike and slow-wave
sleep (CSWS) Cushing syndrome
D
Daytime sleepiness
Delayed sleep phase disorder, circadian rhythms Dental sleep medicine
obstructive sleep apnea oral appliance therapy RDI
sleep disorders
treating snoring
Diastolic pressure, cardiac anatomy and physiology Diffusion
Digital polysomnography AASM
amplifier voltages
amplitude
analog-to-digital
bioelectrical potential signal conductivity
EEG
signals
voltages
electrode
frequency
high-frequency filter
impedance
low-frequency filter
metals
notch filter
Ohm's law
polarity
power line electrical noise sensitivity
time constant
Disturbance index (RDI) E
EFM. Excessive fragmentary myoclonus (EFM) Electrical conducting pathway
Electrical status epilepticus of sleep (ESES) Electrocardiography (ECG)
Electrode disconnection Electroencephalographic–Polysomnographic
montages Electroencephalography (EEG) activity
biopotentials of sleep, anatomy and physiology generation of, slow-wave
activity high-frequency filter setting low-frequency filter setting
minimal recommended sampling rate pattern
rhythm
Enuresis
ESES. Electrical status epilepticus of sleep (ESES) Excessive fragmentary
myoclonus (EFM) Expiratory positive airway pressure (EPAP) F
Fatal familial insomnia (FFI) Functional Outcomes of Sleep Questionnaire
(FOSQ) G
Gap junctions
Gas exchange, respiratory system Generalized anxiety disorder (GAD) H
HFT. Hypnagogic foot tremor (HFT) High-frequency filter
Human physiology, for sleep technologist circadian rhythmicity
enuresis
gastroesophageal reflux disorder during kidney function
melatonin suppression
narcolepsy
slow-wave activity
Hypnagogic foot tremor (HFT) Hypnic jerks
Hypopnea index (HI)
Hypopneas, criteria for Hypoxemia/hypoxia
I
Impedance, digital polysomnography Insomnia
benzodiazepine side effects classification
cognitive therapy
depression
diagnostic criteria for hypnotics
management strategies
medications
mood and personality disorder assessment pathophysiology
patient's history perpetuating factors
possibility of comorbid psychological process
sleep hygiene
stimulus control therapy subtypes
transient or short-term treatment
Inspiratory positive airway pressure (IPAP) Intercalated discs
L
Life cycle, sleep
age-related changes
chin electromyogram
circadian pattern
circadian rhythms, older adults in EEG
activity
pattern
rhythm
electroencephalogram
homeostatic sleep process kindergarten, in fall
neonates
NREM–REM sleep cycle
occipital region
process model
sleep stage scoring, adults slow-wave activity
Low-frequency filter, digital polysomnography M
Maintenance of wakefulness test (MWT) indication
sleep latency
trial
Major depressive disorder (MDD) Maxillomandibular advancement surgery
Movement disorders
actigraphy
ALMA
choreic movements
history and physical
hypnagogic foot tremor hypnic jerks
motor disorder
nonepileptic seizures
panic disorder
placement for electrodes pseudoseizures
role of, sleep technologist seizure type
sleep study
sleep-related events
stage of sleep
stay calm and protect
VPSG
Multiple sleep latency test (MSLT) actigraphy
diagnosis
excessive daytime sleepiness mean sleep latency
nap trial
standardization
Muscle artifact
N
Narcolepsy
cataplexy
dose of modafinil
excessive daytime sleepiness vs. general population hypnagogic
hallucinations hypocretin analysis
imipramine hydrochloride medications
for cataplexy
monitor efficacy
modafinil
MSLT
polysomnography
sleep
hygiene
paralysis
structure
standard diagnostic evaluation symptoms
total amount of sleep
treatment of
vivid dreams
NFLE. Nocturnal frontal lobe epilepsy (NFLE) Nocturnal frontal lobe epilepsy
(NFLE) Notch filter, digital polysomnography NREM sleep, in infants
NREM–REM sleep cycle, in young adults Nyquist theorem
O
Obsessive–compulsive disorder (OCD) Obstructive sleep apnea (OSA) breathing
disorders, in children cardiac arrhythmia
cornerstone
and daytime sleepiness and diabetes
evaluation
hypopnea index
mallampati scale
maxillomandibular advancement surgery mild obstruction
and mood disorders
oral appliances
PAP therapy
pattern of cardiac variability reduce time spent sleeping risk factors
severity
smoking
tidal volume
upper airway
occlusion
surgical management
Obstructive sleep apnea syndrome (OSAS) Oral appliance therapy
contraindications
efficacy
follow-up
indications
OSAS. Obstructive sleep apnea syndrome (OSAS) Overlap syndrome
Oxygen
availability
content
hemoglobin dissociation curve Oxygen and gas exchange, body. Body,
oxygen and gas exchange Oxygen desaturation index (ODI) Oxygen
storage and distribution primary source
sleep center
Oxygen therapy
P
Panic disorder (PD)
Parasomnias
clinical vignettes
confusional arousals
evaluation of
medications
NREM
patients experience
REM sleep
sleep terrors
sleepwalking
Patient and employee safety. Safety, patient and employee Patient calibrations
Patient interview and assessment clinical vignettes
end of study
evaluation, of daytime sleepiness facility orientation
functions, elderly patient history and physical portion possible PAP titration
responses
sleep study
sleep technologist actions with special physical needs strategy for
Patient preparation
A1 and A3 distance
AASM
biocalibrations
electrode placement
electrodes apply
electromyogram
ground electrode
locations identify
MSLT
nasal air pressure transducer polysomnogram
precautions
scalp and face electrodes sensitivity
sensors
sleep center
Pediatric obstructive sleep apnea ALTE
risk factors
symptoms
treatments
Pediatric polysomnography AASM
crossword puzzle solution diagnose
hook-up portion
pediatric polysomnography safety measures
sleep center experience sleep stages
Pediatric scoring
AASM
alpha rhythm
arousal scoring
central apneas
channel study
CO2 monitoring EEG pattern
hypnagogic hypersynchrony normal breathing
NREM sleep
obstructive apnea
sleep stages
Pediatric sleep disorders, nonrespiratory ADHD
assessment
clinical observation
develop
family history
normal sleep duration
primary complaint
quality of sleep
Pediatric sleep laboratory, interventions BPAP titration
course of action
CPAP
diagnosis
respiratory pattern
Periodic leg movements (PLMS) Periodic limb movement disorder (PLMD)
Periodic limb movement index (PLMI) periodic limb movement with
arousal index (PLMArI) Periodic limb movements in sleep (PLMS)
Polarity, digital polysomnography Polysomnogram
narcolepsy
patient preparation
recording
AASM scoring manual
artifacts
attenuates
bio-calibration assesses channel selection
diagnose
electrical contact
electrode application
electrode popping effect excessive filter
M1 and M2 electrodes
mode rejection
modern system
montage
muscle artifact
quality
requirements
sleep study
sleep technologist performing slow-frequency artifact system reference
termination
treatment
Portable sleep apnea monitoring AASM
advantages
channels
Cheyne-Stokes respiration crossword puzzle solution disadvantages
results
unattended PSG tests
Positive airway pressure (PAP) devices
AASM
APAP therapy
full-face mask
goals
improving adherence to therapy inadequate pressure
mask type
REM sleep
RERA
sleep technologist performing titration
titration
CPAP therapy
goal
nasal mask
noninvasive ventilation oxygen therapy
spontaneous respiratory rate Posttraumatic stress disorder (PTSD)
Premature ventricular complexes (PVC) Propriospinal myoclonus
(PSM) Psychiatric disorders
alcohol use
antidepressant medications bipolar disorder
depression
GAD
MDD
mood disorders
OCD
panic disorder
PD
phobia
PTSD
REM sleep
deprivation
SAD
safety issues
schizophrenia
sleep architecture
sleep-disordered breathing total or partial deprivation typical healthy adult
about Pulmonary disease
Q
QRS complex, cardiac arrhythmias R
R latency (RL)
RBD. REM behavior disorder (RBD) REM behavior disorder (RBD) Report
generation
AHI
AI
ArI
HI
ODI
oxygen saturation nadir PLMArI
PLMI
RL
SOL
TRT
TST
Respiration/ventilation Respiratory event–related arousal (RERA) Respiratory
system, anatomy and physiology basic rhythm
carbon dioxide increased levels gas exchange
hemoglobin saturation vs. oxygen pressure motor control, abnormalities
motor nerve control
respiratory rate
robust response of
sensory signals
sigh or hyperventilation during sleep, neural output sleep stage with
tidal volume, of lung
upper airway
collapse
functions
work of breathing
Restless legs syndrome (RLS) clinical criteria for
diagnosis
dopaminergic treatment dysfunction
impact of
treatments for
S
Safety, patient and employee behaviors
evacuation policy
hand washing
lifting techniques
medical emergencies
medical setting
nondisposable item
precautions
purpose of
seizure disorders
semicritical equipment sleep center
protects
Seasonal affective disorder (SAD) Seizures
benign focal epilepsy
clinical distinctions
discharges and clinical with epilepsy
nocturnal
episodes
patients suspected
with primary generalized myoclonic protection
recommendations
sleep deprivation
spike wave
discharges
sudden abnormal discharge treatment
Sensitivity, digital polysomnography Sleep and disorders, medications and effect
benzodiazepines vs. nonbenzodiazepine hypnotics efficacy of melatonin
first-generation antihistamines medications
modafinil
opiates impact sleep
sleep center history
sleep disorders, medications for sleep pattern
tricyclic antidepressants Sleep and medical disorders with acromegaly
asthma
cortisol
Cushing syndrome
fatal familial insomnia medical comorbidities
muscular dystrophy
myasthenia gravis
with neuromuscular disorders nonspecific EEG pattern Overlap syndrome
oxygen desaturation
polysomnography
during sleep study
with symptomatic gastroesophageal reflux Sleep center, oxygen
administration intervention
oxygen delivery devices oxygen storage and distribution precautions
recording and documentation titration
Sleep efficiency (SE)
Sleep onset latency (SOL) Sleep-related eating disorder (SRED) Sleep-waking
SRED. Sleep-related eating disorder (SRED) Sudden infant death syndrome
(SIDS) Sweat artifact
Systolic pressure, cardiac anatomy and physiology T
Therapeutic compliance, developing and maintaining bed partners, complaint
claustrophobia
CPAP device
educational
plan
program
FOSQ
long-term treatment
mouth breathing
PAP
compliance
device
therapy
patient
complains
condition
education
expectations
outcomes
sleep test
social factors
Total recording time (TRT) Total sleep time (TST) V
Video polysomnography (VPSG) W
Willis-Ekbom disease. Restless legs syndrome (RLS)

You might also like